Young Christian: Volume 3, 1913

Table of Contents

1. Inspiration of the Scriptures: The New Testament, Part 4
2. Nearly Ninety, Sir!
3. Scripture Study: Matthew 11
4. Truths for Young Christians: Anger as a Sin
5. Christ the Center: Or, Why Christians Should Meet in His Name Alone, Part 5
6. The Coming and Reign of Our Lord Jesus Christ: The Appearing, Part 4
7. Teach Me Thy Way, O Lord
8. Correspondence: Mark 4:1; Heb. 2:1-4; Rom. 5:10;John 1:10, 14, 51
9. Inspiration of the Scriptures: The New Testament, Part 5
10. Being Let Go
11. Scripture Study: Matthew 12
12. Truths for Young Christians: Anger Caused by Pride
13. Christ the Center: Or, Why Christians Should Meet in His Name Alone, Part 6
14. The Coming and Reign of Our Lord Jesus Christ: The Appearing, Part 5
15. Correspondence: Luke 16:9; Heb. 2:10 and 5:9; Luke 9:60
16. Inspiration of the Scriptures: The New Testament, Part 6
17. I Could Not Do Without Thee
18. A Wise and Safe Thing to Do
19. Scripture Study: Matthew 13, Part 1
20. Truths for Young Christians: Covetousness
21. Christ the Center: Or, Why Christians Should Meet in His Name Alone, Part 7
22. The Coming and Reign of Our Lord Jesus Christ: The Appearing, Part 6
23. Correspondence: Luke 9:21; 1 John 2:16; 1 Cor. 15:42-44
24. Inspiration of the Scriptures: The New Testament, Part 7
25. What God Is Doing
26. Go out Quickly
27. Scripture Study: Matthew 13, Part 2
28. Truths for Young Christians: Deceit and Lying
29. Hezekiah: Brief Lessons on Church Truth, Part 1
30. The Coming and Reign of Our Lord Jesus Christ: The Age to Come, or the Millenium, Part 1
31. Correspondence: Heathen Lost; Luke 14:26; Matt. 6:9
32. Inspiration of the Scriptures: Quotations from the Old Testament
33. What God Is Doing
34. Scripture Study: Matthew 13, Part 3
35. Truths for Young Christians: Worldly Amusements
36. Hezekiah: Brief Lessons on Church Truth, Part 2
37. Correspondence: John 14:12; Young Christians
38. Inspiration of the Scriptures: The Importance of Bowing to the Authority of Scripture
39. Sought and Saved
40. Richer Than the Rich
41. Scripture Study: Matthew 13, Part 4
42. Truths for Young Christians: An Appeal to Christian Young Men
43. Hezekiah: Brief Lessons on Church Truth, Part 3
44. The Coming and Reign of Our Lord Jesus Christ: The Age to Come, or the Millenium, Part 2
45. Correspondence: Fifty Days; Mark 4:26-29; Passion Play; John 14:12
46. Inspiration of the Scriptures: Alleged Inaccuracies of the New Testament
47. An Anecdote of John Knox
48. Scripture Study: Matthew 14
49. Truths for Young Christians: On Our Relationship with the World
50. Hezekiah: Brief Lessons on Church Truth, Part 4
51. Excuses
52. The Coming and Reign of Our Lord Jesus Christ: The Age to Come, or the Millennium, Part 3
53. What Doest Thou Here?
54. Correspondence: Grave; Brotherly Kindness and Charity
55. Inspiration of the Scriptures: The Passover and Lord's Supper
56. Noah's Carpenters
57. Truths for Young Christians: On a Start in Life (for Young Men)
58. Pleasure
59. Scripture Study: Matthew 15
60. Hezekiah: Brief Lessons on Church Truth, Part 5
61. The Coming and Reign of Our Lord Jesus Christ: The Age to Come, or the Millennium, Part 4
62. Correspondence: Hyssop Dipped in Blood
63. Inspiration of the Scriptures: The Call of Abraham
64. Rest
65. Truths for Young Christians: To Young Men in Business
66. Not Heaven But Christ
67. Scripture Study: Matthew 16, Part 1
68. Hezekiah: Brief Lessons on Church Truth, Part 6
69. The Coming and Reign of Our Lord Jesus Christ: The Age to Come, or the Millennium, Part 5
70. Correspondence: Earthly Blessing and Earthly Inhabitation
71. Inspiration of the Scriptures: Holding Jesus by the Feet
72. Thank You for Telling Me
73. Truths for Young Christians: On a Start in Life (for Young Women)
74. Scripture Study: Matthew 16, Part 2
75. Hezekiah: Brief Lessons on Church Truth, Part 7
76. The Coming and Reign of Our Lord Jesus Christ: The Age to Come, or the Millennium, Part 6
77. Correspondence: Psalm 91:5 Explained
78. Inspiration of the Scriptures: The Sixth Hour in John 19:14
79. The Shut Door
80. Truths for Young Christians: What the Word of God Forbids for Young Women
81. She Obeyed His Voice
82. Scripture Study: Matthew 17, Part 1
83. Hezekiah: Brief Lessons on Church Truth, Part 8
84. The Coming and Reign of Our Lord Jesus Christ: The Age to Come, or the Millennium, Part 7
85. Correspondence: Synagogue of Satan; Matt. 16:19
86. Inspiration of the Scriptures: The Bible - its Unity, Part 1
87. Grace Triumphant
88. Truths for Young Christians: What the Word of God Enjoins for Young Women
89. Scripture Study: Matthew 17, Part 2
90. Hezekiah: Brief Lessons on Church Truth, Part 9
91. The Coming and Reign of Our Lord Jesus Christ: The Eternal State
92. Correspondence: Church; John 14:2

Inspiration of the Scriptures: The New Testament, Part 4

THE NEW TESTAMENT
THE GOSPELS
In turning to the gospels, we find them written by four servants of the Lord, Matthew, Mark, Luke and John. But we must dismiss from our minds at once the thought, that they were written by men who recorded what they had known as facts, each one according as his memory served him. This is how men generally write biographies; but it is quite certain that the writers of the gospels often omitted to mention facts with which they were most conversant, and recorded at length other circumstances of which they had no personal knowledge. Let us turn to a few examples. In the raising of the daughter of Jairus from the dead, we are distinctly told that no one was present with the Lord but Peter, James, and John; and yet, most striking as the fact is, and most important, too, in the history of our Lord’s ways, neither of them refer to it; but Matthew, Mark and Luke, who were not present, all name it, and enter into particulars of it.
Again, at the time when the Lord’s compassion was so drawn out toward the widow of Nain, who was attending the funeral of her only son, that He then and there raised him from the dead in the presence of crowds of people, and as we might expect, would be talked of far and near, yet neither of the apostles refer to it, and Luke only has recorded it. Whether he was present or not we are not told; but such a marvelous miracle, and so full of tenderness to a bereaved widow in raising her son to life and delivering him to his mother, if the writers were merely recording facts from memory, as people naturally would do, it would certainly not have been omitted.
In the transfiguration also, we find that only Peter, James, and John were favored to be present, to behold the pattern of the coming kingdom. But neither of them have recorded in their writings the particulars of it. Peter alludes to it in his second epistle, and says, “We were eye-witnesses of His Majesty” (2 Peter 1:16); but neither John nor James tell us anything about it, important beyond all expression as the event was.
Then again, look at our Lord in Gethsemane. One would have thought that every one there present with the Lord in that time of bitter agony and exercise, as anticipating the sorrows of the cross, then immediately before Him, would have fully described the scene. But it is not so. Perhaps no one entered into it more feelingly than that disciple whom Jesus loved, and who leaned on His bosom at the supper; but he gives no details of it, and only makes the briefest allusion to it. Peter, so heavy with sleep there, tells us nothing about it. James, also, is entirely silent; but Matthew, Mark, and Luke, who might not have been near enough to have seen the Lord in such sorrow bowed down to the ground in earnest prayer, each give us a detailed report of it; and yet more remarkable still, after leaving the garden, and going over Cedron with His disciples, when they met Judas and his band of men, and, as the effect of the Lord’s words, “they went backward and fell to the ground” (John 18:6), John only mentions this.
Now these examples are surely enough to convince us, that the gospels were not written as men write a biography, by merely gathering all the well-attested facts they could, and putting them together, for it must be clear to every fair mind this was not the case, but that each wrote according to the direction and guidance of the Spirit who inspired him.
Instead, therefore, of regarding the four gospels as the writings merely of four biographers however true, and trying, as people say, to study “the harmonies of the gospel,” we find that each writer had a distinct and definite line of truth to convey, in no wise contradictory to each other, but each gospel written with a definite object. This, when seen, enhances each of the four gospels immensely, and our interest in them becomes largely intensified. It is intelligible enough that four architects might give us the plans of a square building, each taking a separate side; and although they were all of them different in some particulars, so that no one could understand them till he knew that each represented a different side of the same building, yet then, and not till then, would he get the true idea of what the building really was. So with the gospels. Matthew clearly sets forth the Messiah in relation to God’s ancient people, the Jews; Mark, the Son of God as a perfect Servant; Luke, Son of God born of Mary, yet Son of Man; and John, the Son who came forth from the Father, came into the world, and went back to the Father.
Because MATTHEW presents Him to us as the Messiah promised to Israel, to set up His kingdom on earth, as predicted by prophets, He is at once introduced as “the Son of David, the Son of Abraham.” His genealogy is traced, not on Mary’s side, but in connection with Joseph, as legal Heir to the throne. In Matthew 1, He is called Jesus, meaning Jehovah our Savior; and though born of Mary, is really surnamed-God with us. In Matthew 2, He is said to be “born King of the Jews,” and the prophet Micah is referred to as to Bethlehem being the place of His birth into this world; and there it is added, to show the infinite glory of His person “whose goings forth have been from of old, from everlasting” (Mic. 5:2). In Matthew 3, John, His forerunner according to Isaiah and Malachi, called on the nation to “repent,” because the King was there, and ready to set up the kingdom of heaven. It was therefore “at hand.” In Matthew only we have the expression, “kingdom of heaven,” for this the Jewish people were taught to expect by the prophets; and Moses especially spoke of “the days of heaven upon earth” (Deut. 2:21). The expression, “kingdom of heaven,” occurs about twenty-eight times in this Gospel. John the Baptist had to seal his testimony of “the gospel of the kingdom,” first by imprisonment, and then by death.
Jesus, however, takes up the same testimony, and adds to it the signs of His being the Messiah, by miraculous power, and gives in the sermon on the mount, the principles on which the kingdom must be set up, then touches a leper and by His word heals him, and also a palsied man. The healing of the one showed, that however degraded and unclean the nation might be, there was grace and power in Him beyond all ordinances, however good, for healing; and on the other hand, however helpless the people, He could not only heal the body but forgive sins. In Matthew 10, He gives power to His apostles also to work miracles, and preach the glad tidings of the kingdom, which was a further testimony of His Messiahship. Afterward we find Him feeding thousands once and again on a few loaves and fishes, and baskets of fragments remaining after every one was filled. Now why was this? It was a further testimony to His being the Messiah, because it had been written in Psalm 132:15, “I will abundantly bless her provision, I will satisfy her poor with bread.” So we might go through the Gospel if our space permitted, only we cannot fail to see that it sets before us a line of instruction found nowhere else in Scripture, and yet in perfect keeping with all that had gone before or came after. It is well to notice, that in the twelfth chapter, the Messiah is so entirely rejected by the Jews, that they take counsel to destroy Him (Matt. 12:14); in Matthew 28 He is seen risen from the dead, but not ascended, a risen Man on the earth. And why is this? Because the Messiah’s sphere as such is not ascension glory as His church will have with Him as Bridegroom, Head of His body and Lord of all; but His earthly people, while they will know Him as having died for that nation, and therefore risen, will know Him as reigning here on earth, before His ancients gloriously, and sitting in David’s throne; thus fulfilling all the prophecies of Him, and all the promises to Abraham and his seed. Then of the Jewish people it will be truly said, “The inhabitant shall not say, I am sick; the people that dwell therein shall be forgiven their iniquity.” This they will know and rejoice in, when they sing, “Bless Jehovah, O my soul; and all that is within me, bless His holy name” “Who forgiveth all thine iniquities; who healeth all thy diseases” (Isa. 33:24; Psa. 103:1,3). Such a truly Jewish character has this Gospel, that here only the expression of the wicked people, “His blood be on us, and on our children,” (Matt. 27:25), is recorded; and in neither of the other gospels have we such particular and prophetic instruction as to the great tribulation, the coming of the Lord to Israel, and His judging the nations.
(Continued from Volume 2, page 314)
(To be Continued).

Nearly Ninety, Sir!

The writer of these lines, along with a friend, found themselves in O— with the object of distributing tracts and preaching the gospel. We had a wagon with us in which we traveled about from place to place. One evening we came to a fairly large town, where a number of people had gathered together out of curiosity to see us as we drove up in our wagon and offered some tracts. Their surprise was greater, when one of the strange visitors to their town stood up and cried out to them, “We are ambassadors for Christ, as though God did beseech you by us: we pray you in Christ’s stead, be ye reconciled to God” (2 Cor. 5:20). And taking his Testament out of his pocket, he read with a loud voice, the two following passages: “For God so loved the world, that He gave His only begotten Son, that whosoever believeth in Him should not perish, but have everlasting life” (John 3:16), and: “Be it known unto you, therefore, men and brethren, that through this Man is preached unto you the forgiveness of sins, and by Him all that believe are justified from all things from which ye could not be justified by the law of Moses” (Acts 13:38-39).
When he had read the passages, he connected them together and directed his remarks to those standing around, who all listened with great attention, others also coming to the doors of their houses to hear what the stranger had to say.
A few minutes thus went by, when an old white-haired woman came slowly down the street and stopped motionless opposite the wagon. Bent forward, the whole weight of her body supported on her staff, she stood there and listened attentively to the words of eternal life being preached.
When the speaker had ended, he stepped down from the wagon, as he wished to speak further with this old woman, apparently eager for salvation, and taking her by the hand said: “You support yourself firmly on your staff, I would like to know if you support yourself like that on Christ?”
The eyes of the old lady brightened, and lifting her eyes to heaven, she said, with great expression, “Yes, sir; that’s just what I do. I lean on Christ.”
“Then, is it also true of you, ‘Thy rod and Thy staff they comfort me’?”
“Yes, sir; it is. He gives me comfort, rich comfort.”
“Then you possess a strong staff on which you can rely entirely. And you have need of such a staff. You are old and feeble, I can see.”
“Yes, I am no longer young; nearly ninety, sir.”
We were astonished to hear that. Although the outward appearance of the old woman indicated a great age, we would not have believed her to be that old. In order to prove her a little more, the following question was put to her: “Now, since you have experienced the goodness and faithfulness of the Lord for so long, would you ever think of giving up your Savior in your old age?”
I wish my readers could have seen the old woman answer that question. Raising her bent body to its full height, she replied with an earnestness, which was mixed with a sacred indignation: “What? Give up my Lord? Separate from my Savior? Not for the whole world, sir! What am I saying? Not for the world? No, not for ten thousand worlds!”
We gazed on this old Christian, with heartfelt joy and wonder. There she stood before us, a poor, simple woman, nearly 90 years old; but what sublimity was expressed in her wrinkled countenance — in her whole bearing! How her eyes sparkled with almost youthful fire. During a long life she had experienced the truth of the promises of God and her Savior was of more value to her than “ten thousand worlds.” O, how real, how overpowering is a true, upright Christianity, a living belief in the invisible God and in Him whom He has sent — Jesus Christ. Whosoever believeth on Him shall not be ashamed. And should his days reach ninety, or even a hundred years, he will always experience that his belief will be richly rewarded. After a while one of us asked this old lady, now become so dear to us, “Then you have given your whole heart to Christ, haven’t you?”
“Yes, my heart, my soul, my body — all have I entrusted to Him; all belongs to Him.”
Happy the one who can speak thus! Body, soul and spirit — all, all had she entrusted to Him; who is faithful and able to preserve them blameless unto the coming of our Lord Jesus Christ. (1 Thess. 5:23).

Scripture Study: Matthew 11

Matthew 11
Matthew 11:1. The Lord departs to teach and to preach in the cities of Israel.
Matthew 11:2-11. John the Baptist sends from the prison where he is, two disciples who ask, “Art Thou He that should come, or do we look for another?” It must have been perplexing to his mind that the forerunner of the Great King should be rejected, and that the King should remain in such lowly guise. John is not bearing testimony now to the Son of God, but the Son of God bears testimony to John. The Lord does not explain the change of dispensation that is about to take place, but reminds him by the evidence of his senses that Immanuel is among His people. “Go show John again those things which ye do hear and see. The blind receive their sight, and the lame walk; the lepers are cleansed; and the deaf hear; the dead are raised up, and the poor have the gospel preached unto them.” Isaiah 61:1, was now being fulfilled; the Messiah of Israel had come. And then he adds, “Blessed is he, whosoever, shall not be offended in Me.” He was the rejected one.
John must rest on the word alone without explanation. John’s disciples depart, then the Lord speaks of him as one that looks like “a reed shaken by the wind,” nothing grand in his appearance. Yet he was a prophet and more than a prophet. He was the messenger of Malachi 3:1, for faith. And there was none born of women greater than he, notwithstanding the least in the kingdom of heaven is greater than he. This excepts the King who was speaking of John. The kingdom of heaven is not heaven, but the blessing belonging to the time when the Lord is absent in heaven and the Holy Spirit is dwelling in His people on the earth. The least in the kingdom would, therefore, refer to the position the believer has now, the least is higher positionally than any Old Testament saint could be. Now, believers are children of God, members of the body of Christ; their bodies are temples of the Holy Ghost. And when the Lord reigns as King, they will reign with Him. These blessings belong to those believers who lived since Pentecost (Acts 2), till the Lord shall descend to meet them in the air. (1 Thess. 4:16-17).
Matthew 11:12-13. “And from the days of John the Baptist until now the kingdom of heaven suffereth violence and the violent take it by force.” The law and the prophets prophesied until John; this indicates a change. The energy of the Spirit would impel men to force their way through every difficulty to the kingdom of a rejected King; they had to do violence to all their former training and now believe in and take part with One rejected by the nation who had been, up to this time, called Jehovah’s people. It was, indeed, a strait gate. Had the kingdom appeared in power and glory, this would be just what they expected but instead they must suffice with a rejected One. Thus John was to faith, the Elias that was to come, and those who had ears to hear received his testimony. To the unbelieving nation, Elias is still to come, the many continued in unbelief.
Matthew 11:16-19. There was no response in their hearts Godward. John came in righteousness, Jesus came in grace. Every testimony they reject, so the Lord pronounces judgment upon them.
Matthew 11:20-24. Tire and Sidon would have repented under such privileges, but Chorazin, Bethsaida and Capernaum have “Woe unto thee,” “Woe unto thee,” pronounced by the Lord, exalted to heaven with privileges, yet cast down to Hades, and in the day of judgment it will be more tolerable for Tire and Sidon and Sodom than for those favored cities. (Luke 12:47-48). The judgment of sin is eternal (Rev. 21:8), and this would come upon them. Wisdom’s children justify God, and condemn themselves, and God justifies them; the publicans receive the grace the Lord Jesus can bestow. John came separate from the sins of the nation, and they said, “He has a devil.” Jesus came full of love and pity for wretched sinners, and they called Him a gluttonous man and a winebibber. He was, indeed, a friend of publicans and sinners.
Tire and Sidon had abused the advantages which their Creator — God in His providence, had freely bestowed. They were guilty, but these Jews, possessing the Word of God and the promises, adds to their other sins, rejection and contempt for God’s blessed Son. What judgment they deserve, yet do we not see in them what our hearts are capable of?
The Lord felt the contempt of His people. Unrequited love is painful, but as the obedient Man on earth, He submits to the will of His Father, owning Him Sovereign, Lord of heaven and earth, who, acting in divine wisdom, can make no mistakes. Faith ever submits to His perfection. Jesus accepts His Father’s will as the very best. Man’s wisdom is blind to this the little child spirit can rest in it. “Even so, Father: for so it seemed good in Thy sight,” unquestioning submission of perfect confidence.
Matthew 11:27-30. The person of this blessed One, the Incarnate Son of God, was too glorious to be understood by man. His words and works being refused by the nation, left them without excuse. The Father’s will was everything to Jesus, yet He felt the pain and sorrow His rejection would bring on the Jews, fresh glories shine out in Him because of this rejection. All things are delivered unto Me of My Father, and no man knoweth the Son, but the Father. What a mystery! “Poor, yet making many rich: having nothing, yet possessing all things” (2 Cor. 6:10). All things are in His hand, yet He is a dependent Man on earth and rejected by His people. His person is inscrutable. The eternal Son, yet a man on earth; only the Father can comprehend such a thine.
“The Father only, Thy blest name of Son can comprehend.” He further says, “No man knoweth the Father save the Son and he to whomsoever the Son will reveal Him.” Yes, He can reveal the Father and can reveal Him to whomsoever He pleases — this takes in the poor Gentile dog. Wonderful grace that can bring ruined, sinful men to know God as their Father! And now His invitation goes out. God’s grace is like a river flowing on; if obstacles seem to hinder, it rises and flows over and around them. Man’s failure and resection make it seek new channels. The multitude may reject it, but He knows there are many laboring, heavy-laden hearts, unsatisfied souls, longing for what they do not know. His invitation is to them. “Come unto Me,” “and I will give you rest.” It is the Savior’s voice; the infinite, omniscient One who can give, promising rest to the weary soul that conies to Him. Come, cease your laboring, lay down your burden at His feet; let Him take it. What kind of rest does He give? The kind a guilty sinner needs, the kind a soul disappointed with the world, and Satan’s false promises and allurements needs. You will find pleasures forevermore, peace that passes all understanding, riches that are unsearchable, love that passes knowledge. “I will give you rest.” He has made peace through the blood of His cross. Everyone that trusts Him, finds everything righteously and divinely settled there, and such is the effect of that work, that believers are brought nigh to God. In the above passage, Jesus is the revealer of the Father, so that, included in this rest, is the blessed truth that we know the Father; His God is our God, His Father is our Father.
We can sing:
“Thou great and good! Thou just and wise,
Hail! as our Father and our God!
For we are thine by sacred ties,
Thy sons and daughters bought with blood.”
And again:
“Thou gav’st us in eternal love,
To Him to bring us back to Thee,
Suited to Thine own thought above,
As sons like Him, with Him to be.”
Further blessing He promises in the words, “Take My yoke upon you and learn of Me.” His yoke was entire submission to His Father’s will.
He has given us the children’s place and the children’s portion in the love and care of the Father.
And He desires our souls to enjoy it in full submission to our Father’s will; this is the path He trod. “Learn of Me; I am meek and lowly in heart.” He is our pattern and in so doing He promises that we shall find rest for our souls.
This is rest, rest, rest, not unrest.
Yes, though the tempest ‘round us
Seems safety to defy;
Though rocks and shoals surround us
And billows swell on high-
Thou dost from all protect us,
And cheer us by Thy love;
Thy counsels still direct us
Safe to the rest above.
And our blessed Lord says, “For My yoke is easy, and My burden is light.” May our hearts live in this blessed rest, walking with and following our blessed Lord and waiting for Him.

Truths for Young Christians: Anger as a Sin

Chapter 14
ANGER
Unlike the three subjects we have already considered, “selfishness, pride, and envy,” this is spoken of in two ways in Scripture; the one pointing out when it is right to be angry, the other when it is a grievous sin. Perhaps the most interesting as well as the most profitable way of looking at the subject, will be to consider first a few examples of each.
We will begin with anger as a sin, and observe from the instances selected what are its results when indulged in.
The First Instance of Anger is in the case of Cain. He was “very wroth, and his countenance fell,” the result being the MURDER of Abel (Gen. 4:5).
In Genesis 27:41, in the Case of Esau, we find another instance of how anger is akin to murder, as the Lord pointed out in Matthew 5:21-22. When anger is sinful, it is always the result of some previous sin. When it is righteous, is is the result of a righteous and holy Feeling. Bearing this in mind in going through these examples, it will be interesting to observe not only the results, but the causes of anger. In Cain’s case the cause was ENVY, in Esau’s JEALOUSY. In Numbers 20:10-11, we find the meekest man in all the earth betrayed into anger by his IMPATIENCE, the result of his anger being DISOBEDIENCE; the punishment he received being exclusion from the promised land. Many might justify Moses on this occasion, but God does not. It is true that he was provoked, but followers of Christ here see that PROVOCATION IS NO EXCUSE FOR ANGER.
It must be remembered that God, as supreme, can be angry when man cannot. Hence we frequently have the expression, “provoked Him to anger” applied to God, rightly; but man who is dependent, should not give way to anger, but leave the matter with God as supreme. Jesus when on earth took the place of man, hence He bore all with perfect patience and meekness, committing His cause to Him who judgeth righteously. The punishment to Moses’ case may seem severe, but we must remember that Moses was a great saint, “Moses, the man of God”; and that a little sin in a great saint is worse than a great sin in a sinner. God cannot lightly overlook outbreaks of natural passion in His people, even when provoked; for He has given them power to restrain it.
In 1 Samuel 20:30, we find Saul angry with Jonathan and seeking to kill him, his anger being caused by HATRED of David. In Ahab’s cruelty to Naboth (1 Kings 21) we find that ANGER LEADS TO MURDER, being caused by COVETOUSNESS. In 2 Kings 5:11, we find the anger of Naaman stirred up by his PRIDE, and leading him to despise God’s message to Him.
We might easily multiply these examples, for the seeds of them are in every human heart (of the actions of which the Old Testament is such a wonderful mirror), but we will only select one or two more.
In 2 Chronicles 16:10, we find Asa very angry with Hanani, because the latter had rebuked him for his DISOBEDIENCE. This leads Asa to put Hanani in prison, an act of gross INJUSTICE. In the case of Uzziah (2 Chron. 26:19), his wrath was caused by his being rebuked for committing SACRILEGE, for which sin he was immediately punished by God with leprosy. These last two instances show us how often anger is a result in our hearts of being rebuked or faithfully reproved for some sin that we have committed. Let us be on our guard against this. It is enough to have committed the sin, but it is far worse, when reproved of it by some servant of God, to add to it by a second, and possibly a third, as Asa did. We feel sure that if our readers will but carefully weigh these instances of anger, and compare them in cause and effect with their own history, they will find what a wonderfully accurate mirror of the human heart the Word of God is.
(Continued from Volume 2, page 326).
(To be Continued).

Christ the Center: Or, Why Christians Should Meet in His Name Alone, Part 5

But it may be objected, Has there not been failure and division amongst those who professed to own the spirit of God? Sadly true; but nothing could more clearly prove the truth of these statements respecting the Spirit’s presence. What has been the cause of all the sorrow and division? The setting aside the sovereign guidance of the Holy Ghost. But to say that failure is a reason why any should not own the guidance of the Spirit in the assembly, or refer to it as an excuse for remaining where He is disowned, is like a person saying because he, or any other Christian, has failed in walk, that therefore he should, as an individual, cease to walk in the Spirit. Should not our past sins and failures make us the more watchful and earnest to walk in the Spirit? He alone is the safeguard of the Christian and the church. Blessed Guardian!
The source of every failure the church has ever had, has been by disowning the guidance of the Spirit; no matter what comes, if she only trusts her blessed Guardian, all is well. So with the Christian: if walking in the flesh a straw may cause a fall, but if walking in the Spirit, no matter what temptation, all is well. Every past failure then in the church, or assembly, calls for unfeigned subjection to the Spirit of God. What would you think of a man saying, Such a person, who professed to be a Christian, has failed, and has been found drunk in the streets; therefore I may remain a drunkard with safety. Is it not the same in principle to say, Such of the children of God have failed to keep the unity of the Spirit; therefore I may now remain where the Spirit is not owned. I beg of you judge not this weighty question by the failures of men, but by the Word of God.
What then is the “one body”? (Eph. 4:4). The Church of Rome is not even the catholic church; much less can she be the “one body.” Catholic means universal, so that the millions of the Greek and Anglican and other churches are so many millions of living witnesses against the catholicity of the Romish Church. It cannot be either the one church, or the one body, being but a division — and the same remarks apply to every other division.
“All Mine are Thine, and Thine are Mine; and I am glorified in them.” “And the glory which Thou gavest Me I have given them; that they may be one, even as We are one” (John 17:10,22). These precious words of Jesus embrace every child of God during this dispensation. What then is the glory that the Father hath given to Jesus? He hath “raised Him from the dead, and set Him at His own right hand in the heavenly places, far above all principality, and power, and might, and dominion, and every name that is named, not only in this world, but also in that which is to come; and hath put all things under His feet, and gave Him to be the head over all things to the church, which is His body, the fullness of Him that filleth all in all” (Eph. 1:19-23). And again, “And He is the Head of the Body, the church: Who is the Beginning, the Firstborn from the dead; that in all things He might have the pre-eminence” (Col. 1:18).
The glory then given to Jesus is given to Him as The Risen Christ — and as the risen Christ, He is the beginning and head of the body. Every member then of the one body must be risen with Christ. And thus if any man be in Christ he is a new creature, or a new creation. Now does not Jesus say, “And the glory which Thou gavest Me I have given them”? And this is true of all that are His. Then every Christian is one with the risen Christ in the highest glory; as it is written, “And hath raised us up together, and made us sit together in heavenly places in Christ” (Eph. 2:6).
What a vast difference then there must be between a heavenly risen body, and an earthly society; the only earthly society that God ever had, was the nation of the Jews. Even during the lifetime of Christ, the little company or flock of disciples were of that nation. It was not until after His resurrection and ascension to glory that the Holy Ghost could be given to form “the church, which is His body.” This was the mystery kept hid from ages, that the earthly society, or nation of the Jews, should for a time be set aside, and that the Holy Ghost should gather out of all nations, Jews and Gentiles, a Heavenly Body — and that this body should be joined to the head in risen, highest glory; blest with all spiritual blessings in heavenly places in Christ. And, mark, all this is true of every child of God during this dispensation; because Christ says to the Father, “All Mine are Thine.”
(Continued from Volume 2, and to be continued).

The Coming and Reign of Our Lord Jesus Christ: The Appearing, Part 4

THE APPEARING
Thus far we have been looking chiefly at judgments connected with the Lord’s second advent, and I might cite numerous other passages, for the prophecies, as well as other scriptures, are full of the details of these events. Let us now notice two or three which show how suddenly and unexpectedly, but surely, men will be overtaken in their sins when the Lord comes.
We have already seen that the world is getting worse and worse, and that open apostasy will be the climax, drawing down God’s judgment. In Matthew 24:27-28, the Lord told His disciples that, “As the lightning cometh out of the east, and shineth even unto the west; so shall also the coming of the Son of Man be. For wheresoever the carcass is, there will the eagles be gathered together.” The Jews are represented by the “carcass,” a corrupt people; the nations, like eagles, will gather together to feed upon them, and enrich themselves with the spoil of this people, when suddenly, as a flash of lightning, the Son of man will come in judgment upon them.
“Immediately after the tribulation of those days” (that is, the last half-week), “shall the sun be darkened, and the moon shall not give her light, and the stars shall fall from heaven, and the powers of the heavens shall be shaken: and then shall appear the sign of the Son of Man, in heaven: and then shall all the tribes of the earth mourn, and they shall see the Son of Man coming in the clouds of heaven, with power and great glory” (Matt. 24:29-30).
This same truth is taught in Revelation 1:7, “Behold, He cometh with clouds; and every eye shall see Him, and they also which pierced Him: and all kindreds of the earth shall wail because of Him. Even so, Amen.”
And again in Zechariah 12:10-14: “I will pour upon the house of David, and upon the inhabitants of Jerusalem, the spirit of grace and of supplications: and they shall look upon Me whom they have pierced, and they shall mourn for Him, as one mourneth for his only son, and shall be in bitterness for Him”.
In Zechariah 14:3 we are further told of the Lord going forth to fight against the nations that have fought against His people and land, “And His feet shall stand in that day upon the Mount of Olives, which is before Jerusalem on the east; and the Mount of Olives shall cleave in the midst thereof toward the east and toward the west, and there shall be a very great valley; and half of the mountain shall remove toward the north, and half of it toward the south. And ye shall flee to the valley of the mountains; for the valley of the mountains shall reach unto Azal: yea, ye shall flee, like as ye fled from before the earthquake in the days of Uzziah king of Judah” (Zech. 14:4-5).
The remnant of Judah, witnesses for Christ in the midst of the terrible scenes of the last days, holding, as it were, their life in their hand, being hardly pressed and in great straits, are miraculously delivered by the sudden manifestation of Christ; His feet stand on the Mount of Olives, which splits asunder, and a very great valley affords them a means of escape from their enemies.
“And,” continues the prophet, “the Lord my God shall come, and all the saints with Thee. And it shall come to pass in that day, that the light shall not be clear, nor dark: but it shall be one day, which shall be known to the Lord, not day, nor night; but it shall come to pass, that at evening time it shall be light” (Zech. 14:5-7). Then, after a description of millennial blessing, in Zechariah 14:12-15, we get, “This shall be the plague wherewith the Lord will smite all the people that have fought against Jerusalem; Their flesh shall consume away while they stand upon their feet, and their eyes shall consume away in their holes, and their tongue shall consume away in their mouth. And it shall come to pass in that day, that a great tumult from the Lord shall be among them; and they shall lay hold every one on the hand of his neighbor. And Judah also shall fight at Jerusalem; and the wealth of all the heathen round about shall be gathered together, gold, and silver, and apparel, in great abundance. And so shall be the plague of the horse, of the mule, of the camel, and of the ass, and of all the beasts that shall be in these tents, as this plague.”
(To be Continued)

Teach Me Thy Way, O Lord

Dear Master, for this coming year
This one request I bring;
I do not pray for happiness,
Or any earthly thing.
I do not ask to understand
The way Thou leadest me;
But this I ask, “Teach me to do
The thing that pleaseth Thee.”
A quiet lot in life is mine,
Made up of little things;
Teach me to do as unto Thee
The duties each day brings.
Faithful in that which is the least,
Dear Master, I would be;
Thus making all my daily work
“The thing that pleaseth Thee.”

Correspondence: Mark 4:1; Heb. 2:1-4; Rom. 5:10;John 1:10, 14, 51

Question 61: Mark 4:1. What about “all they that do wickedly being burned up ‘root and branch’?” H. G.
Answer: In the Old Testament, judgment is on the earth. It is putting down all rebellion against Jehovah and purifying the kingdom in which He is to reign.
In the New Testament judgment is “from heaven,” and “against all ungodliness and unrighteousness of men, who hold the truth in unrighteousness” (Rom. 1:18), and shows it is eternal in its duration. (Rev. 21:8 is eternity).
Malachi 4:1, is speaking of the day of the Lord, when these wicked will be destroyed completely from the earth. Matthew 13:41-42, is the same time. Matthew 25:41,46, shows their eternal portion, Bear in mind that the Old Testament is judgment on the earth. The eternity of bliss and woe is unfolded in connection with the gospel.
Question 62: Hebrews 2:1-4. What is the state of those to whom this is addressed? Also, is the “great salvation” referred to, that which the sinner, whether from among the Jews or Gentiles, receives at his conversion? L. S.
Answer: The Epistle is addressed to Hebrews who have professed Christianity. They may or may not be true Christians; they are warned of the danger of going back to Judaism. All falling away in Hebrews is apostasy. The writer supposes them to be real (Heb. 6:9,10), and sees in them “things that accompany salvation”. The proof of reality is seen in their holding fast to the end their boldness and confidence of hope, and he desires that they would continue in diligence. (Heb. 6:11, 12).
The “great salvation” includes al! that Christ is for the sinner. It becomes his by faith in our Lord Jesus Christ.
Question 63: What is the force of “We shall be saved by His life”? J. E. K.
Answer: It is how the fullness of God’s grace is assured to us. It belongs to the Blessed God’s “not only so” and “much more”, that after having made us His own, heaps on more blessings. If God reconciled us to Himself by the death of His Son when we were sinners, much more shall we be saved by His life. It was by His death we were brought into blessing. Now in the mighty energy of His new risen life, He cares for us. If He died for us to put our sins away, how much more will He care for us now, living in the glory. “Because I live, ye shall live also” (John 14:19).
Question 64: What does it mean, “The world knew Him not?” (John 1:10; 14,51? Also Eph. 5:10,16.). A. P.
Answer: The world failed to recognize in Jesus its Creator; the Son revealed God, but man was darkness and did not comprehend the light. Even the Jews did not receive Him. Faith saw in the Word made flesh one in the character of an only begotten Son with the Father. See also 1 John 1:1-3. We are also the children of God, the Father, “therefore the world knoweth us not, because it knew Him not” (1 John 3:1). We know God as our Father, We are born of God. The 51st verse looks forward to the glory of Christ in the millennium, that is, when He reigns over this world; then both heaven and earth will worship Him.
Ephesians 5:10. We were once darkness, but now light in the Lord. When we walk as children of light, that is, walking in communion with the Lord, we have His approval; we find out what pleases Him.
Ephesians 5:16. “Redeeming the time, because the days are evil”, means for us to use all our opportunities, both by word and deed, to serve Him. Satan and his servants are always active, and growing more so. We need, therefore, to be in earnest for our Master.

Inspiration of the Scriptures: The New Testament, Part 5

THE NEW TESTAMENT
THE GOSPELS
Mark’s Gospel. Here the Lord is looked at more particularly as to His service. We have therefore no genealogy, no account of His birth, and His ways are traced from the baptism of John to His sitting at the right hand of God. Throughout, as the elect and righteous Servant, He is seen doing most perseveringly the will of Him that sent Him. We find the word translated “immediately,” “straightway” and “anon” much more frequently than in any other Gospel. The looks and feelings of the perfect Servant are referred to in a way we have not elsewhere. We are told that “He looked round about on them with anger”: that “He sighed”; and that “He sighed deeply in His spirit.” He went on so diligently serving with His disciples, that “they had no leisure so much as to eat”: and again, “They could not so much as eat bread,” so that His friends went out to lay hold on Him, for they said, “He is beside Himself.” The sufferings of Gethsemane and on the cross are briefly recorded, and, having accomplished the work of redemption, as risen from among the dead, He sends His servants into all the world to preach the gospel to every creature, and afterward, though received up into heaven, and sitting at the right hand of God, He confirms their ministry with signs following.
As has often been remarked, in Mark we have the events of our Lord put before us more in historical or chronological order, while in Matthew they are arranged more in regard to dispensational order, and in Luke they are more in moral order.
Luke’s line of things in his Gospel is clearly Jesus as “Son of Man.” As born of Mary He is contrasted with John who was born of Elizabeth, the son of Zacharias. Jesus was conceived by the Holy Ghost, and born Son of God, and Son of the Highest; but John was only the prophet of the Highest, and though he was honored to be the forerunner of our Lord, his testimony was, that he was unworthy to loose the latchet of His shoe. It is in Luke’s Gospel only that we have the account of our Lord at the age of twelve years, and that He “increased in wisdom and stature, and in favor With God and man.”
The genealogy of our Lord is traced in Luke to “the seed of the woman.” We, therefore, have Mary’s line through Heli brought out; through David also, for she was of the house and lineage of David; and Abraham, Noah, and Enos to Adam, because in this Gospel He is looked at as Son of Man.
In Luke’s account of Him, He is not only brought before us praying on seven different occasions, but in Gethsemane also His humanity is specially marked out in His being in an agony, and praying more earnestly, when His sweat was as it were great drops of blood falling down to the ground. Here, too, on the cross, He makes intercession for the transgressors, and comforts the penitent malefactor with the assurance of present salvation not mentioned elsewhere. In resurrection also, when some were terrified because they thought He was a spirit, He said, “Why are ye troubled? and why do thoughts arise in your hearts? Behold My hands and My feet, that it is I Myself; handle Me and see, for a spirit hath not flesh and bones as ye see Me have. And when He had thus spoken, He slowed them His hands and His feet. And while they vet believed not for joy, and wondered, He said unto them, Have ye here any meat? And they gave Him a piece of a broiled fish, and of an honeycomb. And He took it, and did eat before them” (Luke 24:38-43). Now, why are these details as to the actual resurrection of our Lord from among the dead given us in Luke’s Gospel only? Is it not because He is there by the Spirit of God brought before us as Son of Man? But further, after His resurrection, He ministered the word to them, gave commission to preach the gospel, bade them tarry in Jerusalem for the coming of the Holy Spirit to endue them with power from on high, and having led them out as far as to Bethany, and blessed them, He was parted from them, and carried up into heaven. The Man Christ Jesus, whom they had seen and known as incarnate, as dead on the cross, as buried in the sepulcher, and as risen from among the dead, they saw ascend up to heaven, till their eyes could no longer trace Him. Having now their understandings opened to understand the scriptures, they were filled with such joy, that they were continually in the temple praising and blessing God.
In John’s Gospel we have the deity of the Son, and sent by the Father into the world, in richest and abounding grace to us. In it we have the endearing relationship of children of God by faith in Christ Jesus clearly set forth, so that duties and affections might be formed and maintained suitable to such relationship. There is, therefore, no genealogy in John. He was “the Word” in the beginning, before creation, with God, and was God — a person with God, and yet eternally divine, for He was God, and the Creator of everything that was made. In due time. “the Word was made (or became) flesh and dwelt among us.” If we have the human side of Jesus as Son of man, born of Mary, in Luke, and as Son of David, “king of the Jews,” in Matthew, we have the divine side of our Lord and of His ways in John. He is the Fountain of Life, for “in Him was life” in the John 1; the Source of eternal life to every one that believeth, in the John 3; the One who gives an unending supply of living water in John 4.
He, the Son, quickens or gives life to whom He will in John 5; and is the Bread of Life (through His flesh, which He gave for the life of the world,) in John 6. In the John 7, Christ Himself is the alone source of that living, enjoyment which enables testimony for Him abundantly to flow out. With all His amazing grace to man, His words are rejected in John 8; His works in John 9; and not believed on as the Good Shepherd, because they were not His sheep, in John 10. In John 11, as has been often pointed out, He is rejected as Son of God; in John 12, first as Son of David, and then as Son of Man; and after having shown Himself to be the Resurrection and the Life, in bringing Lazarus out of the grave to life again, He willingly goes into death as the Son of Man lifted up to save sinners. The Corn of wheat must fall into the ground and die, or He would be alone. Solemn moment beyond all description.
He then keeps the Passover; Judas goes out; and when alone with His eleven true ones, He instructs us as to our course during the time of His absence, promises to send the Holy Spirit to abide with us forever; bids us to be without fear or care, but to believe on Him; and though the path be one, of tribulation, to be of good cheer; and assures us that He will come again and take us to the Father’s house, to be where He is forever. Having said these things, He commends them, and all who shall believe on Him through their word, to the Father, before He goes to Calvary’s cross to glorify the Father, and finish the work that He gave Him to do. Gethsemane is only just touched on in John, and at the sound of the Savior’s voice, those who came to take Him went backward, and fell to the ground; and on the cross, He is presented as saying, “I thirst,” in order that the Scripture might be fulfilled, in bringing Him vinegar to drink, according to Psalm 69. All then being fulfilled as it is written, He said, “It is finished,” and bowed His head and gave up His spirit, according to His Word in the tenth chapter, when speaking of laying down His life: “No man taketh it from Me, but I lay it down of Myself; I have power to lay it down, and I have power to take it again. This commandment have I received of My Father” (John 10:18). It is the Son here glorifying the Father, whose prayer, when under the shadow of the cross, had been, “Father, glorify Thy name.”
In beautiful keeping with this divine side of Christ in John’s Gospel, we find only here in John 20:17, the account of His message by Mary after His resurrection, “Go to My brethren, and say unto them, I ascend unto My Father, and your Father, and to My God and your God;” He meets His disciples with “Peace be unto you,” fills them with joy, for they “were glad when they saw the Lord,” thus giving them a taste of the blessedness of His being in the midst; and communicates risen life by breathing on them, and saying, “Receive ye the Holy Spirit.” Eight days after He meets Thomas, a type of the Jewish remnant who will not believe till they see Him; and in John 21, a striking picture is given of millennial blessing when He will formally take His place as the last Adam, and reign before His ancients gloriously.
If it be asked why we have thus glanced at each of the four gospels, our reply is, In order that it may be clearly seen, that while each honored the Lord, the lines of truth in all differed from each other, though there was nothing contradictory, and all divinely perfect. The most positive proof is afforded by it, that all was written according to the purpose of one Master mind, and could be none other than the ministry of the Holy Spirit. When we consider how much of the gospels is made up of our Lord’s own words, and works, and ways, how frequently the Old Testament scriptures are authoritatively quoted, and how much of their prophetic teaching was actually fulfilled; and when we add to all this, that the spirit of them all is so manifestly according to the operations and testimony of Him who is the Glorifier and Testifier of Jesus, and Guide into all the truth, it is impossible not to discern the clearest possible proof of their being inspired, or God-breathed.
(Continued)

Being Let Go

“And being let go, they went to their own company” (Acts 4:23). This simple statement presents a beautiful example of the instincts and tendencies of the divine nature. We always find that when a man is released from some special engagement — set free from some special demand upon him — in a word, when he is “let go,” he will, most probably, seek the company of those who are most congenial to his tastes. When parade is over, the soldiers betake themselves to their various associates and pursuits. When a school breaks up, the pupils do the same. When the warehouse or office is closed, the young men go either to a religious assembly, or reading room or, alas, to the saloon, or theater or gambling house, etc. “Being let go,” they are almost sure to go “to their own company.” It is when a man is fully at leisure that you see what his bent and tendency really are. When he gets free from present claims you will be able to judge of the pursuits and companions of his heart’s selection. Two men may be seen standing behind the same counter, from eight in the morning till six in the evening; but mark them when the clock strikes six — observe them when they are “let go” — and you will find one making his way to the saloon, and the other to his home and then to a religious meeting. Thus it is always. “Being let go.” we soon find out “our own company.”
Reader, how do you act when “let go”? What company do you seek? Do you betake yourself to those who, like the assembly in Acts 4, occupy themselves in holy worship, prayer, and praise? Or do you own as your companions, the giddy and the thoughtless, the profane and the immoral, the scoffer and the skeptic, the infidel and the atheist? O search and see. Just ask yourself, when next you take your seat in the midst of your own company, “Would I, at this moment like to hear the voice of the archangel and the trumpet of God”? Are you washed from your sins in the blood of Jesus? Are you saved? Are you at peace With God? Let me beseech you, dear friend, to make close, earnest, personal work of it this very hour. Do not trifle with your immortal soul, and with a boundless eternity. God is in earnest — Christ is in earnest — the Holy Ghost is in earnest — Satan is in earnest — and will you trifle? Will you delay? “Behold now is the accepted time; behold now is the day of salvation” (2 Cor. 6:2). May God the Holy Ghost lead you, now, to believe in the love of God, and lean fully, and without the shadow of a doubt, upon the perfect sacrifice of Christ. Then you will seek the “company” of the redeemed on earth; and, when “let go” from every weight and hindrance down here, you will join “your own company” in the mansions above.

Scripture Study: Matthew 12

Matthew 12
In the close of Matthew 11, the Lord, though rejected, promises rest to the heavy laden, laboring ones that will come to Him; this is the saved remnant. In this chapter, Matthew 12, He rejects Israel, severing His relation with them for the present; a new ground of relationship is brought in.
Matthew 12:1-8. The disciples rubbing the ears of grain in their hands and eating it, becomes the occasion for the Pharisees to find fault with them for breaking the Sabbath. The Lord refers to the time when David and his men hungered and ate the showbread that was only for the priests to eat. It was Israel’s sin then that David, the true king, was outlawed. It was Israel’s sin now that the Son of David, the Great King, was also rejected. God cannot respect ordinances, when the heart is in sin at any time. The Sabbath was the sign, of the covenant Jehovah made with Israel. (Ex. 31:13, 17. Ezek. 20:12, 20). Now it is set aside. The priests, on the Sabbath Day in the temple, profaned the Sabbath, because sacrifices for sin had to be offered, and here was one greater than the temple. If they had known the meaning of the saying, I will have mercy and not sacrifice, they would not have condemned the guiltless. For the Son of Man is Lord also of the Sabbath. It was His right to do as He pleased.
Matthew 12:9-16. Here again He acts in grace; they would not scruple to pull their own sheep out of a pit on the Sabbath day. A man is of more value than a sheep, and He will heal this man with the withered hand and answer their question. “Wherefore it is lawful to do well on the Sabbath day.” Man’s need set aside the Sabbath.
This brought out the malice of those so-called good men, the Pharisees; they “held a council against Him, how they might destroy Him” — the giver of all good. The Lord saw it and withdrew, and many followed Him, and He healed them all. And He charged them that they should not make Him known; the time for that was past.
Matthew 12:17-21. Now the Spirit witnesses to His person. (Isa. 42:1-4). “Behold My Servant, whom I have chosen; My Beloved, in whom My soul is well pleased; I will put My Spirit upon Him, and He shall show forth judgment to the Gentiles.” But though such a wonderful person, He takes a lowly place now, doing His work out of sight of man as much as possible. “Till He send forth judgment unto victory and in His name shall the Gentiles trust.” This passage gives an exact picture of His position.
Matthew 12:22-42. A man is brought to Him possessed with a devil, blind and dumb. This is a picture of the people’s condition with respect to God. The Lord heals and delivers him, so that the man both spake and saw. The people were amazed and said, “Is not this the Son of David?” But Pharisees, full of malice and jealousy, and opposed to God’s testimony, declare that the Lord is doing all this by the power of the devil. The Lord shows the foolishness of the argument. Satan would not cast out Satan; if so he could not stand. They would not say that of their sons, therefore, they should be their judges. But if He cast out demons by the Spirit of God, then the kingdom of God is come upon them; they were resisting God. The Lord hath bound the strong man, else He could not spoil His goods; and the Lord was delivering men, in grace, from the power of Satan that had held them captive. It was plainly the deep-rooted malice of their hearts. With all their religious profession, they were against God, and so against Him. He gathered — they scattered. They blasphemed the Holy Ghost. They knew better; the works of power showed it was the power of God, and not of Satan. Their case was hopeless. Words spoken against the Son of Man would be forgiven; but speaking against the Holy Ghost would not be forgiven, neither in that world (the age of law), nor in the world to come (that is, the millennial age). This is not referring to the present period of grace. It has no reference to a poor, backsliding Christian; it refers distinctly to those who, in Christ’s lifetime, said, “He hath an unclean spirit” (Mark 3:29-30).
They were a corrupt tree, a generation of vipers, evil men out of whose hearts’ abundance the evil came, for “out of the abundance of the heart the mouth speaketh.” A good man treasures in his heart good things, and bringeth them forth; an evil man has nothing but evil. A man’s words are the index of what is in his heart. And “I say unto you, that every idle word that men shall speak, they shall give an account thereof in the day of judgment. For by thy words thou shalt be justified, and by thy words thou shalt be condemned” (Matt. 12:36). May Psalm 19:14 be our earnest endeavor: “Let the words of my mouth, and the meditation of my heart, be acceptable in Thy sight, O Lord, my Strength, and my Redeemer.”
After all this they ask a sign from Him. What bold wickedness, after all His miracles that they had seen! There was only one sign for them now; it was the sign of the prophet Jonas. They were an evil and adulterous generation. Jonas was three days and three nights in the belly of the fish; so shall the Son of Man be three days and three nights in the heart of the earth. It was their Messiah cast out, rejected, and put to death. “The Ninevites shall rise in judgment with this generation, and shall condemn it because they repented at the preaching of Jonas; and behold, a greater than Jonas is here.” So also the Queen of Sheba, who came from afar to hear the wisdom of Solomon, and “a greater than Solomon is here.” Poor, despised Gentiles heard and understood the Word of God by. the prophet, or the king, better than Jehovah’s people, even when spoken by the Great King and Prophet Himself.
Matthew 12:43-45. Here then is their evil state: They are like a man who had been possessed by an unclean spirit; that is, they had been idolaters and this is the reason Israel was sent into captivity. (Acts 7:42-43). (Isaiah 40 to 48 judges their idolatry for which they were sent to Babylon captives. Isaiah 49 to 58 judges their rejection of Christ for which they were scattered after the death of Christ.) But this returned remnant had no idolatry. In the days of the Lord, they were like a house empty, swept and garnished, but they did not receive the Lord. (John 5:43). So by and by, this same spirit of idolatry will enter into Israel (which ought to be God’s house) and with seven other spirits worse than himself, will take possession again; this will be in the tribulation, thus their last state shall be worse than the first. “Even so shall it be also unto this wicked generation.” What a solemn judgment on the Jews; what sorrow they were bringing on themselves.
Matthew 12:46-50. When Jesus’ mother and His brethren present themselves to speak with Him, He uses them to illustrate that He had now broken with Israel, His natural relations, and that now those whom He would own were those who would do the will of His Father which is in heaven: the same is My brother, and sister, and mother.
The new position and work He was doing is opened up in the next chapter.

Truths for Young Christians: Anger Caused by Pride

Chapter 14
ANGER
ANGER CAUSED BY PRIDE in the person of the wicked Haman, and leading to the attempted destruction of an entire people. The same cause, PRIDE, in Nebuchadnezzar’s case, filled him with rage and fury, so that the form of his visage was changed (like Cain’s), and led to INTENSE CRUELTY on his part against his victims, which, however, God miraculously overruled. In Jonah’s case we find great anger caused by IMPATIENCE, which led him to speak against God. He appears to have so completely given way to it, that in Jonah 4:9, he actually justifies his unrighteous anger to God. In the New Testament we find the anger of Herod leading him to murder all the children of Bethlehem. We further see, in Luke 4:28, that the Jews, stung with JEALOUSY of God’s favors to the Gentiles (Luke 4:24-27), sought to MURDER Christ on the very spot, and in Acts 7:54, we find the Jews again filled with HATRED AGAINST CHRIST, actually gnashing on Stephen with rage and stoning him to death.
Causes and Results of Anger
From these illustrations we find that anger is caused by envy, jealousy, impatience, hatred, pride, covetousness, and by the just rebukes of God’s people; that, if unchecked, it tends to cruelty and murder, also to disobedience, injustice, and despising God’s Word.
Turning for a moment to what is said about it in Scripture, we find that it is expressly forbidden (Matt. 5:22; Rom. 12:19), it is a work of the flesh (Gal. 5:20), it is characteristic of fools (Prov. 12:16; 14:29; 27:3), it brings its own punishment (Job 5:2; Prov. 19:19), it is often stirred up by bad words (2 Sam 19:43), but pacified by meekness Prov. 15:1), that we should not provoke others to it (Eph. 6:4; Col. 3:21).
We will now briefly consider some instances of
RIGHTEOUS ANGER.
In Mark 3:5 we find the Lord angry, ‘‘being grieved for the hardness of their hearts.” How instinctively we feel in this case, the unselfishness of the anger. It is all for their sakes and for God’s glory. Righteous anger never has self in any shape or form for its cause. Moses was angry in Exodus 11:8; but it was for the indignities offered by Pharaoh to the Lord and His people, unlike his anger in Numbers 22, for which he was punished. We also find Moses angry in a similar way in Exodus 32:19 and Leviticus 10:16. In Nehemiah 5:6, we find Nehemiah very angry against gross injustice done by others, and to others, not against himself; hence he “did well” to be angry. In Ephesians 4:26, we get the exhortation “to be angry and sin not,” that is, not to treasure up anger and malice in our hearts.
We have now before us the two sorts of anger, the one generally the fruit of some other sin, always having “self” for its ultimate cause; the other springing from zeal or indignation for the Lord, and having Him or His people for its cause. We thus find that the first anger like other sins we have considered, is a selfish sin, and the surest way of being saved from it is to be free from oneself. It is a great moment for a Christian when he practically ceases to be the center of his interests and objects. This should be at conversion, but does not practically take place till Christ reveals Himself in sufficient power to the heart to replace the wretched idol of “self.” A Christian can only be happy in proportion as this is the case, for a selfish Christian is a most miserable object, and is indeed a contradiction in terms. The surest way, therefore, to overcome the sin of anger is not by cultivating a placid disposition, which is only dealing with externals, but by striking at the root, which is self, and replacing it with Christ. The true Christian is zealous for his Master’s interests, not his own, and may be righteously angry when His glory is concerned, but not for his own sake. May the Lord make us all more zealous for Him, and deliver us from serving and pleasing ourselves.
“Be ye kind one to another, tenderhearted, forgiving one another, even as God for Christ’s sake hath forgiven you” (Eph. 4:32).

Christ the Center: Or, Why Christians Should Meet in His Name Alone, Part 6

Wherever the child of God is as to his body on earth, in spirit he is as really one in the risen Christ as a member of the human body is joined to the person whose it is. Yea, our oneness in Christ is not union, but perfect unity. As we could not say, the union of the members of the human body, for all those members constitute one person, so also is the heavenly risen Christ. “For as the body is one, and hath many members, and all the members of that one body, being many, are one body; so also is Christ. For by one Spirit are we all baptized into one body, whether we be Jews or Gentiles, whether we be bond or free,” etc. “Now ye are the body of Christ, and members in particular” (1 Cor. 12:12-27). Certainly the Spirit uses the strongest possible words and the most striking figures to express this wondrous unity.
Compare the above passage with the following: “For we are members of His body, of His flesh, and of His bones” (Eph. 5:30). It does not say we were one with Him during His life in the flesh — that were impossible. Had He not died, He must have remained alone. (John 12:24). Earthly oneness of sinful men with a sinless Christ could not be; no, He must die, and has died for the sins of many; and having passed through death for them, as their substitute — having through the shedding of His precious blood paid their ransom — He has been raised from among the dead, and, as their surety, justified. (Isa. 50:8). And all this for us: “raised again for our justification (Rom. 4:25). And thus we are reckoned dead with Him, raised with Him, justified with Him and one with Him in that risen, justified, sinless state: So that we are, not were, one with Him.
As a man is one person, though having many members, so is the risen Christ; though having many members on earth, yet all joined to and one with and in Christ the Head in heaven. “We are members of His body.” “There is one body” (Eph. 4:4; 5:30). What a wondrous new creation, new existence, this is! Translated into the kingdom of His dear Son — we are, not we shall be when we die. “Hath delivered us from the power of darkness, and hath translated us into the kingdom of His dear Son” (Col. 1:13).
It is the forgetting of this present reality, the oneness of the whole church of God in the risen Christ in heavenly glory, that is one sad cause of the worldly systems and earthly divisions which men call churches. I often ask, “When you are in heaven, will you tolerate sects and divisions?” “O dear, no!” is the reply. Christ will then be all. But are we not now raised with Him, and made to sit with Him in heavenly places? (Eph. 2:6). And is not Christ all now? (Col. 3:11) In the new creation there is neither Jew nor Greek, Romanist or Protestant, Independent or Methodist; O no! Christ is all. “Old Things are passed away; behold, all things are become new. And all things of God” (2 Cor. 5:17-18). And this is true of every man in Christ. He is, or let him be, a new creature; The risen body of Christ, then, is one, composed of all believers out of every nation; a new creation from among the dead, raised together and joined together by God the Father (Eph. 2). Can never be separated. (Rom. 8:39). There are no divisions in that heavenly body, neither indeed can be. For the old things are passed away. Blessed Jesus, Thy prayer is answered; “That all may be one” (John 17). Yes, all who believe are one with Christ in the heavenly places.
What then is the will of God as to believers on earth? For, while one with Christ in heaven, we are now for a very short time absent from the Lord while here in the body. I do not wish to state opinions, but what is the mind of the Lord? Solemn question. May, He give grace to do His precious will.
That God condemns division, none would wish to deny who bow to His inspired Word. At the very first appearance or bud of divisions the apostle says, “Now I beseech you, brethren, by the name of our Lord Jesus Christ, that ye all speak the same thing, and that there be no divisions among you. Every one of you saith, I am of Paul; and I of Apollos; and I of Cephas; and I of Christ. Is Christ divided?” (1 Cor. 1:10-13). Surely I cannot mistake the mind of the Lord in this day, when every one saith, I am of Rome, I of the Greek, I of the Anglican, or I of Wesley. God beseeches all believers by the glory and pre-eminence of the name of the Lord Jesus that be no division. Not one name or division can God tolerate. To allow any name but His, is to lower His blessed name to the same level. I of Paul, and I of Christ. If it is thus God’s will that there should be no divisions, how can I belong to any or in any way countenance any sect without positive disobedience to God’s revealed mind? Do, my reader, answer that question in the presence of God with His Word before you.
Lest there should be any mistake, the Spirit of God again speaks on the same subject: “For ye are yet carnal: for whereas there is among you envying, and strife, and divisions, are ye not carnal, and walk as men? For while one saith, I am of Paul; and another, I of Apollos; are ye not carnal?” (1 Cor. 3:3-4). If it thus grieved the Spirit to say, I am of Paul, or Apollos, does it now please the Spirit to say, I am of Wesley, I of the Independents? Is this carnality? or is it spirituality? Does God approve or disapprove? And again, when the apostle refers to having heard there were sects among them, he says, “I praise you not, that ye come together not for the better, but for the worse” (1 Cor. 11:17).
Yea, God could not speak more plainly, not only as to what He condemns, but also what His will is as to what is right: “That there should be no schism (or division) in the body; but that the members should have the same care one for another” (1 Cor. 12:25). Man says there should be sects, and would have me join one or help to increase it. God says there should be none, for the body is one. Shall I obey God or man? Judge ye.
What a blessed unity, one with the Head above, and one with every member here below! Yes, every member, every Christian on earth! How precious the will of God: “And whether one member suffer, all the members suffer with it; or one member be honored, all the members rejoice with it. Now ye are the body of Christ, and members in particular” (1 Cor. 12:26-27).
Surely, now, we have failed to own this wondrous unity. But let us not lower the standard. Let us not call evil good. Surely division is an evil, and a bitter thing in the sight of God. He even classes it with such sins as adultery, murder and drunkenness.(Gal. 5:17-21). The word translated heresies means sects. O, let us then return unto the Lord with deep humiliation. Let us confess the common sin and shame of the divided church.
We are called to heavenly oneness with the risen Christ. It is the will of God that “ye walk worthy of the vocation wherewith ye are called, with all lowliness and meekness, with long suffering, forbearing one another in love; endeavoring to keep the unity of the Spirit in the bond of peace. There is one body, and one Spirit,” (Eph. 4:1-4). Would you, my fellow believer, do the will of God? Here, then, is the blessed path; the unity of the Spirit. This must ever be to the Head — Christ. The blessed Spirit gathers to the person of Christ, and where two or three are gathered in His name, there He is in the midst of them. Man makes a meeting in what name he likes. It is division, or scattering. The Spirit alone gathers to Christ. The two things are as different as the unity of heaven and the scattering of earth.
(Continued and to be continued).

The Coming and Reign of Our Lord Jesus Christ: The Appearing, Part 5

THE APPEARING
In the Lord’s wonderful discourse in Matthew 24 and 25, we have a remarkable description of the state of men at that day from His own lips: “As the days of Noah were, so shall also the coming of the Son of Man be. For as in the days that were before the flood they were eating and drinking, marrying and giving in marriage, until the day that Noah entered into the ark, and knew not until the flood cane, and took then all away: so shall also the coming of the Son of Man be. Then shall two be in the field; the one shall be taken, and the other left. Two women shall be grinding at the mill; the one shall be taken, and the other left” (Matt. 24:37-41).
As in the days of Noah, men will be living for themselves and their own selfish ends, little thinking of the awful judgment that shall suddenly overtake them. They “knew not until the flood came, and took them all away”; none escaped but Noah and his family, who were left in the ark. When the Son of man comes, one shall be taken away in judgment, and another left in mercy, to be brought into millennial blessing, which is exactly opposite to that which will occur when the Lord comes for His people, for at that moment the saints will he taken (caught up) to heaven, and the world left for judgment.
“Take heed to yourselves,” saith the Lord to His people “lest at any time your hearts be overcharged with surfeiting, and drunkenness, and cares of this life, and so that day come upon you unawares. For as a snare shall it come on all them that dwell on the face of the whole earth” Luke 21:34-35).
With this terrible hour before Him, rapidly approaching, God, who is not willing that any should perish, “now commandeth all men everywhere to repent: because hath appointed a day, in the which He will judge the world (the habitable earth) in righteousness, by that Man whom He hath ordained; whereof He hath given assurance unto all men, in that He hath raised Him from the dead” (Acts 17:30-31).
“The Father judgeth no man, but hath committed all judgment unto the Son: that all men should honor the Son, even as they honor the Father.... And hath given Him authority to execute judgment also, because He is the Son of Man” (John 5:22-27).
Put how blessed to know that He is swift in mercy, slow to judgment; that judgment is His strange work. (Isa. 28:21). “The Lord is not slack concerning His promise, as some men count slackness; but is long-suffering to us-ward, not willing that any should perish, but that all should come to repentance. But the day of the Lord will come as a thief in the night,” etc. (2 Peter 3:9-10).
Were it not for His long-suffering, we who believe, and are now the children of God by faith in Christ Jesus (Gal. 3:26), would never have been brought into this wondrous blessing; but haying repented, we are now delivered from the judgment, and are not in darkness that that day should overtake us as a thief, but are all the children of light and the children of the day.(1 Thess. 5:4-5). The Christian can look with joy for the coming of His Lord to receive him to Himself, and for the day when he shall be manifested with Him. “Your life is hid with Christ in God. When Christ, who is our life, shall appear, then shall ye also appear with Him in glory.” (Col. 3:3-4).
“Beloved,” says the Apostle John, “now are we the sons of God, and it doth not yet appear what we shall be: but we know that, when He shall appear, we shall be like Him; for we shall see Him as He is” (1 John 3:2). And every man that hath this hope in Him, purifieth himself even as He is pure.
Paul could anticipate that glorious moment, and say, “I have fought a good fight, I have finished my course, I have kept the faith: henceforth there is laid up for me a crown of righteousness, which the Lord, the righteous Judge, shall give me at that day: and not to me only, but unto all them also that love His appearing” (2 Tim. 4:7-8). He encourages, too, the hearts of the saints at Thessalonica, saying, “And the Lord make you to increase and abound in love one toward another, and toward all men, even as we do toward you; to the end He may establish your hearts unblameable in holiness before God, even our Father, at the coming of our Lord Jesus Christ with all His saints” (1 Thess. 3:12-13).
And again, the Spirit of God brings it before the hearts of His people in a most precious and practical way in the Epistle to Titus (Titus 2:11-14). I will quote the whole passage: “For the grace of God that bringeth salvation hath appeared to all men, teaching us that, denying ungodliness and worldly lusts, we should live soberly, righteously, and godly, in this present world: looking for that blessed hope, and the glorious appearing” (or the appearing of the glory) “of the great God and our Savior Jesus Christ; who gave Himself for us, that He might redeem us from all iniquity, and purify unto Himself a peculiar people, zealous of good works.”
(Continued and to be Continued).

Correspondence: Luke 16:9; Heb. 2:10 and 5:9; Luke 9:60

Question 65: What does Luke 16:9 mean? Are we to make friends of the world? Can they give us anything? Are their habitations everlasting? T. H.
Answer: The new translation reads the text, “And I say to you, make to yourselves friends with the mammon of unrighteousness, that when it fails ye may be received into the eternal habitations.”
We (Christians) are to consider ourselves stewards of all that the Lord has entrusted to us. If we use selfishly what we have, our possessions and natural talents will be a hindrance to us spiritually. If we remember we are stewards and use our money for the Lord, this is making friends with it. “When it fails” reminds us that we are here for a short term, then we will need to give account. (2 Cor. 5:10). We are going on to eternal habitations with the Lord. If we are faithful now, we will profit then; and profit now also, for we lay hold of what is really life.
Read prayerfully 1 Timothy 6:17-19.
See last page of Young Christian, Volume 1, for fuller explanation.
Question 66: In what character is the Lord Jesus presented for us in Hebrews 2:10 and 5:9? Does this differ from Hebrews 2:18 and 4:15? L. S.
Answer: In Hebrews 2:10, the Lord is “Leader” of the redeemed company. In Hebrews 5:9, he is the cause or means of salvation to those who obey Him. In Hebrews 2:18 and Hebrews 4:15, He is the Priest to sustain those who are in the way. In Hebrews 12:2, He is the “Leader and Completer” — the “Originator.”
Question 67: What does “Let the dead bury their dead” mean? E. M. G.
Answer: Read Matthew 8:19-22 and Luke 9:57-62. The claims of Christ over the believer come before every other claim. The man who said, “Suffer me first to go and bury my father”, was putting the claims of natural relationships first, so the Lord answered, “Follow Me, and let the dead bury their dead.” Men acting according to natural claims are here looked at as dead. If they are guided by earthly relationships, their fathers will be dead and buried before they are free to follow the Lord.
The Christian in the Epistles of Ephesians and Colossians is plainly seen as dead, buried, risen, and united to Christ in resurrection life. In this new position he receives instructions from the Lord how to carry out every relationship. Wives and husbands, children and parents, servants and masters, all are to own Christ first, and all we do is to be done in obedience to Him. His claims are supreme.

Inspiration of the Scriptures: The New Testament, Part 6

THE NEW TESTAMENT
THE ACTS AND THE EPISTLES
In the Acts of the Apostles, we have the coming of the Holy Spirit to indwell and baptize believers into “one body,” by uniting them to Christ the Head, and to one another. This work of the Holy Spirit has been going on ever since in those who have believed on our Lord Jesus Christ to the saving of the soul. At Pentecost they were all filled with the Holy Spirit, and began to speak with tongues as the Spirit gave them utterance. Peter too was filled with the Holy Spirit, and preached the gospel; and the remainder of the book gives us the various actings of the Holy Spirit who had came down in consequence of the accomplished redemption-work of our Lord Jesus Christ. Besides filling the saints for the service and praise of God, we trace the Holy Spirit’s Godhead, personal actings, and hatred to sin; His sovereignty, power, holiness, and truth. The gospel was preached not only to the Jews but also to the Gentiles, and the servants of God, principally Peter and Paul, were strengthened and filled with the Spirit to set forth a crucified, risen, ascended, glorified, and coming Savior, with abundance of blessing to souls. It was a time of the Spirit’s power, which gave no quarter to selfishness and covetousness. The record too is of the Spirit, for it magnifies God, and honors our Lord Jesus Christ, and the authority of Scripture.
We have referred already to the epistles of the apostle Paul, and have given instances in proof of the inspiration of God in his writings. It is well, however, to remember, that our Lord Jesus repeatedly spoke to him from heaven as to what he should minister to the saints; but we may notice on other occasions the way in which he so constantly points to the Lord and honors Him as such. For instance, in the second brief Epistle to the Thessalonian believers, he speaks of Him as the “Lord Jesus Christ” ten or eleven times; in Phil. 1, he speaks of Him as Lord or Christ seventeen or eighteen times, and always with reverence and honor; and he seems in all his epistles to write in such a Christ-exalting way, that he is not able to write many verses without turning to Him. Can anything more clearly demonstrate the work and inspiration of the Holy Spirit.
Though Peter’s line of instruction to the saints is very different from Paul’s, yet when Paul addresses the Ephesian saints, or Peter the converted Jews when scattered, they are both so filled with the love of God, that before they begin their communications to them, they lift their hearts in praise, and say, “Blessed be the God and Father of our Lord Jesus Christ.” Peter’s line as well as Paul’s is Christ; but Peter, especially in his first epistle, points to an unseen Savior as the fountain of “joy unspeakable and full of glory;” though such too “greatly rejoice” in Him, who has accomplished such a work in His death and resurrection, as gives us title to an incorruptible inheritance reserved in heaven for us, while we are kept for it.
John, however, who recognized the workings of infidelity even in his day, is so conscious of the apostolic ministry being the Word of God, that he declares it to be the true test of vital Christianity. He says, “We are of God; he that knoweth God heareth us; he that is not of God heareth not us” (1 John 4:6).
Though James addresses his letter to the twelve tribes, he recognizes “beloved brethren” among them, and ministers to them as such, ascribing their being begotten of God to “the word of truth;” and though not entering upon the special truths of Christianity, he instructs, quotes from the prophets, and enjoins them to practical piety in the prospect of the coming of the Lord. With James, the man who says he has faith, can only show it by his works. The more the epistle is pondered, the more clearly the Spirit’s teaching can be recognized in it.
Jude, though he treats of the apostasy, tracing it from its root to its full development and judgment, of which Enoch prophesied, exhorts the faithful to contend earnestly for the faith which was once delivered to the saints, to keep themselves in the love of God, praying in the Holy Spirit, and building up themselves on their most holy faith, while seeking the good of others, and looking for the mercy of our Lord Jesus Christ.
All these epistles, though written at different times, and by different instruments, tell us of unity of purpose and object.
(Continued)

I Could Not Do Without Thee

I could not do without Thee,
O Savior of the lost,
Whose precious blood redeemed me
At such tremendous cost;
Thy righteousness, Thy pardon,
Thy precious blood, must be
My only hope and comfort,
My glory and my plea.
I could not do without Thee,
I cannot stand alone,
I have no strength or goodness,
No wisdom of my own;
But Thou, beloved Savior,
Art all in all to me,
And weakness will be power,
If leaning hard on Thee.
I could not do without Thee,
For O, the way is long,
And I am often weary,
And sigh replaces song.
How could I do without Thee?
I do not know the way;
Thou knowest and Thou leadest,
And wilt not let me stray.
I could not do without Thee,
For years are fleeting fast,
And soon, in solemn loneliness,
The river must be past;
But Thou wilt never leave me,
And though the waves roll high,
I know Thou wilt he near me,
And whisper, “It is I.”

A Wise and Safe Thing to Do

“Thy Word have I hid in my heart that I might not sin against Thee.” (Psa. 119:11).
This, truly, is a wise and safe thing to do. Let us ponder it. Let us understand it. Let us imitate it. There are three special points suggested, namely, What have I hid? Where have I hid it? Why have I hid it? The reader will easily remember What? Where? Why?
1. What have I hid? “THY WORD.” It is not man’s word, but the Word of God, that liveth and abideth forever. This is the thing to hide. It is a treasure worth hiding. No thief can steal it, no moth corrupt it. It increases by being hidden in the way here spoken of. We cannot set too high a value upon the Word of God. So the Psalmist thought when he “hid” it. This expression sets forth how intensely he prized the Word. “I have hid it.” He placed it out of the reach of every one and everything that could deprive him of it. May we ponder it — may we understand it — may we imitate it!
2. Where have I hid it? “IN MY HEART.” It was not in his head or in his intellect: but in his heart — the seat of his affections — the center of his moral being — the source of all the influences that swayed his entire career. This is the right place to hide the Word. It is not hiding it under a bed, or under a bushel, or in the earth. It is not basely cushioning it, through a slavish dread of men, lest they should sneer at us, or oppose us. No, my reader, this will not do. We must hide the Word where the Psalmist hid it, even in the heart. May we ponder this — may we understand it — may we imitate it!
3. Why have I hid it? For a very weighty reason — a most important reason. “THAT I MIGHT NOT SIN AGAINST THEE.” It was not that he might have a rich fund of new ideas to talk about and show off upon. Nor yet was it that he might be able to confound in argument all his opposers, and silence them. The Psalmist did not care about any of these things. He had a horror of sin — a holy horror; he knew that the most effectual safeguard against sin was the Word of God, and therefore he hid it in his heart. May we ponder this — may we understand it — may we imitate it!

Scripture Study: Matthew 13, Part 1

Matthew 13:1-23
Matthew 13:1. “The same day Jesus went out of the house, and sat by the seaside.” This action signifies His leaving Judaism, to teach whosoever would hear; the last verses of Matthew 12 show that relationship by birth is no longer acknowledged, but, “he that doeth the will of My Father which is in heaven”; so that whosoever received the word, would be brought into blessing in Christ. It is now sowing the seed, a new thing; not seeking fruit from a vine or fig tree, as in Israel, but sowing seed that will produce fruit in the soul, where it is mixed with faith in those who hear it (Heb. 4:1).
Matthew 13:2-3. He speaks to the multitude in parables: The Sower is the first. “Behold, a sower went forth to sow;” it illustrates His action.
Matthew 13:4. “And when he sowed, some seeds fell by the way side, and the fowls came and devoured them up.
Matthew 13:5-6. Some fell upon stony places, where they had not much earth; and forthwith they sprung up, because they had no deepness of earth: and when the sun was up, they were scorched; and because they had no root, they withered away.
Matthew 13:7. And some fell among thorns; and the thorns sprung up and choked them.
Matthew 13:8. But other fell into good ground, and brought forth fruit, some an hundred-fold, some sixty-fold, some thirty-fold.
Matthew 13:9. Who hath ears to hear, let him hear.”
This chapter has seven parables; the last six are similitudes of the kingdom of heaven, that is, six pictures of those who are professedly under Christ’s authority on earth. The first parable is not called a likeness, but is given to show how the kingdom of heaven is formed by the preaching of the Word; those who profess His name are therefore looked at as in it. The next parable will show us that both false and real are there.
Matthew 13:10-11. The disciples ask, “Why speakest Thou unto them in parables?” He answered, “Because it is given unto you to know the mysteries of the kingdom of heaven, but to them it is not given.” This marks the difference between the believers — just a remnant — and the mass, or nation, that did not receive Him.
Matthew 13:12. The one would increase in knowledge and possession, the other would lose even what he had.
Matthew 13:13-15 tells us they are the ones spoken of in Isaiah 6:9,10, quoted in verses 14 and 15.
Matthew 13:16-17. It was a blessed time for this remnant that received the Lord, and heard and understood His Word; their eyes were opened to see Jesus as the sent one of God. Many Old Testament saints and prophets had desired to see and to hear, but had not seen nor heard them.
Matthew 13:18. He begins the explanation of the parable.
Matthew 13:19. The wayside hearer pays no attention to what he had heard; it does not enter his understanding, and Satan takes the seed away. His attention is wholly given to other things (2 Cor. 4:4).
Matthew 13:20-21. The stony ground hearer receives the Word without exercise. He has no felt need for it; he rejoices over it, but when it causes him tribulation, he does not want to have any more to do with it; there was no deepness of earth. It looked like conversion, but he had no root in himself; so he can give it up when it does not suit him. The flesh can never stand persecution because of the Word, so he is offended.
Matthew 13:22. The thorny ground hearer heard the Word, and owned the truth of the Word, but failed to put it into practice. The cares of the world and the deceitfulness of riches engaged all his attention. In him also there is no fruit. Cares and riches do not look like sin, but if permitted in the mind, they shut out God, and all is barrenness.
Matthew 13:23. In the good ground we get reality. The Word is understood, the ground is plowed up, the Word penetrates and produces fruit; but in different degrees, according to the exercises of the soul: in some an hundred-fold, in some sixty-fold, in some thirty-fold. Leaving some of those thorns in the ground (Matt. 13:22), and the lusts of other things, hinder believers from bringing forth fruit as they should. If the believer gets under the power and influence of the world and worldly things, he loses his communion with the Lord; his fruit bearing is hindered; he is sleeping (Eph. 5:14). He cannot enjoy the company of those who enjoy and talk of spiritual things, and his conscience reproves him for want of faithfulness.
“Keep thy heart with all diligence; for out of it are the issues of life” (Prov. 4:23).

Truths for Young Christians: Covetousness

Chapter 15
COVETOUSNESS
An insatiable sin, a sin that grows by that on which it feeds, a sin that leads to all sorts of other sins, the one sin of the heart directly forbidden by the ten commandments, a hidden secret sin coming from the heart. “For from within, out of the heart of men, proceed covetousness” (Mark 7:21). Applied to money it is “the root of all evil” (1 Tim. 6:10); it is never satisfied. It leads to injustice and oppression (Mic. 2:2) to departure from the faith. (1 Tim. 6:10). It is abhorred by God (Psa. 10:3). It excludes from the kingdom of God, being classed with such sins as theft, idolatry, and adultery (1 Cor. 6:10). It is one of the sins of the last days (2 Tim. 3:2; 2 Peter 2:1-3). Such is covetousness, and yet so deceitful is this sin, that but few are aware of its dangerous and awful character. In the world, indeed, it is hardly accounted a sin at all; and it is therefore difficult for a worldly Christian to understand how coveting what is another’s, is as bad before God as theft or drunkenness. The fact is, that it is only the standard of the Word of God that shows what sin is; and in a measure the world at large has profited by this. Theft and adultery, and other sins are now everywhere admitted to be wrong, but in other ages they were not. It is only within the last century that drunkenness has begun to be classed as a sin by the world, while covetousness and other sins of the heart (though equally condemned by the Word) are, as yet, totally unrecognized as such.
Covetousness Is Theft in the Heart
Writing, however, as we do, for those who take the Word and not the world’s code of morality for their standard, we would earnestly warn them against this sin, which may be called theft in the heart. But, you say, it is very hard not to covet when I am poor and struggling; to see others so well off. This is true, but, though hard, you must get the victory and, by setting your affections on things above, you will find you are as rich, and, it may be, far richer than they, so that the positions are reversed, and the rich man, discontented with his riches, covets the calm and happy mind of the humble Christian. God has made us so rich that it can be only through ignorance of our wealth or through earthly tastes that we covet at all; this we see in Psalm 73, the whole of which is written to prove this very point.
Examples of Covetousness
Before, however, saying more about it, it may be well for us to listen, as we have done before, to what the Word of God has to tell us by the way of example concerning this sin, carefully observing to what sins it especially leads. The first sin, the parent of all other sins, was partly covetousness. Eve saw the fruit was good for food; she knew it was not for her, but she coveted, and she took, and fell. Covetousness is frequently the result of looking at things we ought not. If we let our eyes drop from Christ to the world, we shall soon find our poor hearts running after it, and covetousness, and a whole host of other sins, will follow. In Joshua 7:21 we find a fearful instance of covetousness in Achan. “When I saw, then I coveted, and I took.” How like Eve, and how terrible in its results, causing not only his own death, and that of thirty-six others, but the defeat of Israel before their enemies; for God could not lead them to victory with a covetous man in their midst! Observe in both these cases covetousness leads to direct DISOBEDIENCE to God. Have any of my believers any hidden sin, like Achan’s, destroying their happiness, eating away their spiritual life, and perhaps injuring and distressing others? O, let us judge our selves, that we be not judged by the Lord.
Covetousness Leads to Many Sins
Passing on, we may notice it was the greed and covetousness of Samuel’s sons, Joel and Abiah, that lead the people to demand a king (1 Sam. 8:1-5). This king, Saul, was dispossessed of his crown and kingdom through direct disobedience to God, into which he was led by covetousness (1 Sam. 15:9-19). Passing down the stream of time we come to Ahab, who, through covetousness of Naboth’s vineyard, was led to commit judicial MURDER, led on by Jezebel. Gehazi’s covetousness led him into a course of LYING and DECEIT, and brought upon himself the fearful plague of leprosy (2 Kings 5:20-24).
That covetousness was one of the besetting sins of Israel, we may see from Jeremiah 6:13, “From the least of them even unto the greatest of them, every one is given to covetousness.” But let us remember that this covetousness in Israel was nothing like so bad in character as it is amongst us; for, after all, what they coveted was merely an undue share of that which God had given to them all, for their blessings were earthly, and none could blame them for highly esteeming money and property. The Christian’s possessions are spiritual, but it is a very rare thing for Christians to be striving to get an undue share of these as the Jews did of their temporal blessings. On the contrary, the object of the covetousness of Christians too often, is the world and the things that are in it — things on which they should not set their heart or affections at all, still less envy those who possess more than they. What a tale, therefore, it tells of spiritual deadness, when a child of God, an heir of glory, is seen to covet the poor riches of earth!
Babylon, a type of this world in its prosperity, was full of covetousness.
Turning now to the New Testament, we find in the fearful history of Judas, that it was covetousness of money that lead him to BETRAY his Master, a character of sin of which any of us may also be guilty, though of course not in the same way. The Pharisees are branded as covetous, and this led them to reject and despise the faithful, searching words, “Ye cannot serve God and Mammon.” Covetousness is also the sin of Baalam (2 Peter 2:14); those whose hearts are full of covetous practices are said to follow the way of Baalam. We have thus seen that the effects of this sin are uniformly bad, seeing that it leads to disobedience to God, rejection of His Word, lying, deceit and murder. None are exempt from this sin: those who have little would have much, those who have much would have more, It is wonderful, therefore, to possess THE SURE REMEDY FOR THIS SIN, and that is in simply having the enjoyed possession of so much, that not only can we not wish for more, but cannot even hold what we have. Such a portion is the Christian’s, and were our hearts more true to Christ, we should be but little troubled with low, covetous desires, for in Him we have more than we could wish, more than our hearts can contain. Hence, if we are really filled with all the fullness of God, what room is there for a covetous thought, however selfish we may be; if, as must be the case, occupation with Christ not only fills us, but transforms us. Covetousness is not absent so much because we are full, as because we have ceased to desire for ourselves; what we desire being for Christ’s glory, His interests having supplanted our own. Christ, then, is the cure for covetousness, by virtue of both, His satisfying and His transforming power. We are sure that the lives of many Christians are miserable, mainly, from the effects of this one sin; for, unlike other sins which may make those who commit them happy for a time, this sin makes its victim wretched, so that there is no more unhappy object than a thoroughly covetous man; while, on the Other hand, there is no happier object than a Christian who is satisfied with Christ.
(Continued)

Christ the Center: Or, Why Christians Should Meet in His Name Alone, Part 7

All believers are one in the risen Christ, and the will of Christ is that that unity should be manifested to the whole world. How deeply and touchingly this is seen in the present intercommunings of the Son with the Father: “That they all may be one; as Thou, Father, are in Me, and I in Thee, that they also may be one in Us; THAT THE WORLD MAY BELIEVE”; and again, “I in them, and Thou in Me, that they may be made perfect in one; and that THE WORLD MAY KNOW that Thou hast sent Me, and hast loved them, as Thou hast loved Me” (John 17:21,23). Thus, instead of earthly divisions and discord, the blessed Lord would have us manifest to the world our oneness with Himself in glory. We are dead with Him, risen with Him, and shall be glorified with Him. But O! to walk worthy of this oneness with the risen Christ. However we may have failed, I am not thereby excused from faithfulness to Him; and I cannot therefore be identified with, anything that grieves Him or is contrary to His mind. Sects and divisions have been shown to be utterly contrary to His will; therefore I must separate from them all if I would walk according to God’s Word. I can own no church but the one body, no principle of church government but that of the Holy Ghost, no name but that of the Lord Jesus Christ — alone Head of the risen body, the church of God.
The path may be difficult, but when was the path of faith easy? These are perilous times. Evil is called good; good, evil; indifference, neutrality. “Wherefore He saith, Awake thou that sleepest, and arise from the dead, and Christ shall give thee light.” “Wherefore be ye not unwise, but understanding what the will of the Lord is” (Eph. 5:14,17).
The Lord is at hand, and He hath said, “Surely I come quickly.” How very soon the last sound of discord shall be heard! O, haste the day when the exalted Lord shall be forever owned and adored. O, my fellow believers, with such a prospect shall we not, during this little while, seek to do His blessed will? He would have us separate ourselves and purge ourselves from every vessel of dishonor. (2 Tim. 2:19-21). He would have us gathered in His name. (Matt. 18:20). Surely we need no argument in addition to our Lord’s revealed will.
I would add a few words, in conclusion, to those who are gathered in the name of the Lord Jesus, desiring in everything to be subject to the guidance of the Holy Ghost.
Let us remember, beloved brethren, that God has gathered us together in the name of the Lord Jesus; that we have not met of our own will; that we have only to seek the glory of Christ, and to win souls to Him. Let us not be ashamed of His precious name and the blessed place in which He has set us as witnesses of Him. Yea, let us rise as one man to make known the claims of Christ. But this can only be done in unshaken faith. There may be the name and form, and not the power. When gathered in the name of the Lord Jesus, do we always expect the Spirit to testify of Him? If men go to hear an eloquent preacher, they expect to hear him. Do we thus expect the teaching of the Spirit of God by the Word? God is pleased to use gifts, but His own presence is more than all gifts.
I am hot speaking of a blind impulse, or of what some call man’s inward light. No; I ask, do we really believe in the presence of the divine person of the Holy Ghost? Then let not one rise to deliver his own thought, ready prepared as it were; and let not the weakest say, I am not fit to be used of God. Let there be a real yielding of ourselves to God, to be kept silent, or used to speak the words He shall give — it may be but the reading of a verse of Scripture. Have we not often felt more of the real power of God’s presence at such a time than we can possibly describe? How blessed to feel you are in His very presence, to hear His words, as though He were speaking in an audible voice. O, may there be such fervent prayer that the manifest guidance of the Spirit of God may be seen and felt in every gathering. Have faith, my brethren, in God.
To my brethren who are still in the sects of men, whether Roman, Greek or Protestant — of whatever name — let me earnestly entreat you to seek divine guidance in the Scriptures of truth. My confidence is in God, that He will, by this feeble paper, lead many of you to own the name of the Lord Jesus in unfeigned subjection to the Spirit of God. We may never know each other here, but when we meet around the throne, then we shall not regret having left every sect, and every name, and having been gathered only in the name of the Lord Jesus. Do not suppose that I imply that the name of Jesus is not dear to all the children of God in the various divisions of the professing church. No; for to you who believe He is precious. But you are not gathered in His name alone, as the one body of Christ. Each sect has some other name, or some other principle, which hinders all the children of God being gathered with them, in contrast to the true ground of being gathered by the Spirit of God to the name and person of Christ. In thus gathering, there is no barrier to any or all the children of God walking in subjection to the Holy Ghost.
Nor can such a gathering or assembly of God be truly called a sect, any more than the assembly of God in the days of the apostles could.
Nor would I for a moment imply that my brethren in the various divisions deny the existence of the Holy Ghost. What I say is, that when they meet for worship or teaching they do not submit to the Holy Ghost, and allow Him to preside over the meeting, using whom He will, as in 1 Corinthians 12:14. They have departed from the Spirit’s rule, and have set up human order. And thus one member is burdened, and the others become mere listeners. There is work for every member, according to the measure of grace. All cannot speak in public; but cannot God use the feeblest attempts — a word by the way? Yea, often the prayer of a poor man, filled with the Spirit, is more blest to the saints of God than the eloquence of an Apollos.
May the Lord Himself lead us all into unfeigned subjection to the Holy Ghost, according to His blessed Word.
(Continued and to be Continued).

The Coming and Reign of Our Lord Jesus Christ: The Appearing, Part 6

The Appearing
The day of the poor world’s judgment will be the day of our manifest exaltation with Christ in glory; instead of our being judged, we shall come with Him to reign and judge. “If we suffer, we shall also reign with Him” (2 Tim. 2:12); and “Do ye not know that the saints shall judge the world?” (1 Cor. 6:2).
We have already noted that angels, as well as saints, will accompany the Lord at His appearing. This is clearly shown in Matthew 25:31, a scripture to which we must now refer, as showing the dealings of the Son of Man with the nations at His advent.
“When the Son of Man shall come (or shall have come) in His glory, and all the holy angels with Him, then shall He sit upon the throne of His glory: and before Him shall be gathered all nations: and He shall separate them one from another, as a shepherd divideth his sheep from the goats”.
This is often taken to be a description of the last judgment; but if you carefully read the whole passage you will find that it is a totally different scene altogether. Many have been misled by the heading of the chapter, or the note at the top of the page in many Bibles, in which it is so termed; but these have only been put in by man, and often very erroneously, especially so here. It is the separation of nations, introductory to the millennial reign of Christ, and a thousand years before the judgment of the dead at the great white throne.
When the Son of Man shall come in His glory (or “shall have come” is the full force of the passage), He sits on His throne of glory, and all nations (mark the words, all nations) not the dead, but the living, the quick, shall be gathered before Him, who is Judge both of quick and dead. (2 Tim. 4:1). Having judged the military power of the West under the beast, and with him the false prophet at His revelation, having subdued all nations and cast Satan into the bottomless pit, He next sits upon His throne, and exercises discriminating judgment among the nations, preparatory to the establishment of His kingdom in peace and righteousness throughout the earth. They are dealt with according to the way that they have treated His brethren (not the church, who will have already gone from this scene, but the King’s brethren, Jews), who will have preached the gospel of the kingdom as a witness to all nations during the time of antichrist’s power. The righteous; separated on His right hand, are invited into the kingdom; that is, earthly blessing in the millennium. The goats hear the awful sentence, “Depart from Me, ye cursed, into everlasting fire, prepared for the devil and his angels” (Matt. 25:41).
Many features in this description most widely differ from that of the judgment of the dead. For instance, the Scripture, in speaking of “nations,” refers to those who are alive, not to those who are dead. In the scene before us we find distinctly three classes — “the brethren, the sheep, and the goats” — the first not judged at all; the second invited into blessing; the third sentenced.; whereas in Revelation 20:12 we find only one class, “the dead,” and all are sentenced to eternal judgment.
In Matthew, the Son of Man has come with angels, and sits on His throne of glory; in Revelation, there is no coming, no mention of angels, but of a great white throne. Here it is a judgment by the King on the earth; there the earth and the heaven flee away before the judgment. Here no books are mentioned; there books are opened, and the book of life. Here the goats are commanded to depart into everlasting fire, prepared for the devil and his angels; there the devil is in the lake of fire before the wicked dead are cast there (Rev. 20:10). One is distinctly the Son of Man as King dealing with the living nations prior to the establishment of His power and glory in the earth (which is the Solomon character of His reign); the other the judgment of the wicked dead after the close of the earth’s jubilee.
If we come to the Word of God with the preconceived and widely-accepted thought that there is only one resurrection and one general judgment, we must confound things which widely differ, rob Christ of that which is for His glory, and our own souls of rich blessing. God’s thoughts are not our thoughts, and our ways are not God’s ways; but as the heavens are higher than the earth, so are His thoughts higher than our thoughts, and His ways than our ways (Isa. 55:8-9).
Before closing the subject of the Lord’s appearing, I would call your attention to one more scripture out of the many that treat of it in the Old Testament. In Joel 3, we have a graphic description of events connected with the manifestation of Christ in power and judgment on the nations, the deliverance of Israel, and the blessing of the kingdom.
“Proclaim ye this among the Gentiles; Prepare war, wake up the mighty men, let all the men of war draw near; let them come up: beat your plowshares into swords, and your pruninghooks into spears: let the weak say, I am strong.” (Notice, this is exactly the reverse of Isaiah 2:4, which will be fulfilled after this). “Assemble yourselves, and come, all ye heathen, and gather yourselves together round about: thither cause Thy mighty ones to come down, O Lord. Let the heathen be wakened, and come up to the valley of Jehoshaphat” (which is close to Jerusalem on the northeast): “for there will I sit to judge all the heathen round about. Put ye in the sickle; for the harvest is ripe: come, get you down; for the press is full, the fats overflow; for their wickedness is great. Multitudes, multitudes in the valley of decision,” (marg., concision, or threshing) “for the day of the Lord is near in the valley of decision. The sun and the moon shall be darkened, and the stars shall withdraw their shining.” (Compare Matt. 24:29.) “The Lord also shall roar out of Zion, and utter His voice from Jerusalem; and the heavens and the earth shall shake;” (compare Heb. 12:26-29) “but the Lord will be the hope” (marg., place of repair, or harbor) “of His people, and the strength of the children of Israel” (Joel 3:9-16). The close of this remarkable chapter gives particulars of the blessing of the kingdom, of which we shall speak in our paper on the millennium.
Beloved reader, with this fearful judgment looming nearer and nearer for this ungodly world, I cannot refrain from once more appealing to you, and beseeching you, if still unsaved, to flee from the coming wrath. “The day of the Lord is at hand, and as a destruction from the Almighty shall it come” ( Joel 1:15), but the day of grace has not yet passed; soon, very soon, the door of mercy for you will be closed. O, enter, enter now! God has given His Son, His only Son, His well-beloved; that Blessed One has died and risen again; God is glorified in Him; and now “by Him, all that believe are justified from all things” (Acts 13:39). Do you believe?
Lo! ‘tis the heavenly army,
The Lord of hosts attending;
‘Tis He, the Lamb, the great I am,
With all His saints descending.
To you, ye kings and nations,
Ye foes of Christ, assembling;
The hosts of light, prepared for fight,
Come with the cup of trembling.
Joy to His ancient people!
Your bonds He comes to sever-
And now, ‘tis done! The Lord hath won,
And ye are free forever-
Joy to the ransomed nations!
The foe, the ravening lion,
Is bound in chains while Jesus reigns
King of the earth in Zion.
(Continued and to be continued.)

Correspondence: Luke 9:21; 1 John 2:16; 1 Cor. 15:42-44

Question 68: Please explain why the Lord asked those who knew Him, not to make Him known, in Luke 9:21 and other places, when He healed their diseases and restored the blind to sight. Also what is the meaning of Christ’s answer, or why did He give the person this kind of an answer in Luke 9:58? C. T.
Answer: In Luke 9:21 it was because the Lord was rejected as Israel’s Messiah, and was going to make atonement for sin on the cross, that He said, “Tell no man that thing”; then calls Himself the Son of Man.
We need to read the context in each case to understand the reason. In Mark’s Gospel, specially, it is as the true servant hiding Himself, and doing the Father’s will. What an example this is for us.
Luke 9:57-62 are tests of discipleship. Are we willing to lose our comforts to follow the Lord? (verse 58).
Are we willing to lose our character before men to follow the Lord? (verses 59-60).
Are we willing to lose our connections to follow the Lord? (verse 61).
These points test whether Christ or self is our object. Quite true, we get comforts from the Lord if we follow Him. We get a new character as His servants, and new connections in His people. We also see from Ephesians 5 and 6, and Colossians 3 and 4, that “wives” and “husbands”, “children” and “parents,” “servants” and “masters”, are all relationships given to us from Him to carry out (through grace) for Him.
The man in John 5, who was carried by the bed for 38 years, received strength to carry the same bed for Christ.
True discipleship puts and keeps Christ first. (John 12:26).
Question 69: What scriptures condemn the use of tobacco as a lust of the flesh? N. R. M.
Answer: Read Rom. 6:12-13,19; 13:14; Gal. 5:16-17,24; Eph. 2:3; 2 Tim. 2:22; Titus 2:12; 3:3; 1 Peter 2:11; 2 Peter 2:10,18; 1 John 2:16-17.
These and many other scriptures teach us that the flesh never walked in God’s ways, but in self-pleasing. Man, away from God, does His own will. The Lord Jesus, as a man on earth, ever walked in obedience to His Father’s will. His every thought and word and deed was obedience to His Father.”By the word of Thy lips I have kept Me from the path of the destroyer” (Psa. 17:4).
If we are believers, we are now “in Christ” (Rom. 8:1); He is our life (Col. 3:4); and the Holy Spirit dwells in us (1 Cor. 6:19). Christ is also our pattern; our lives are to be after the pattern of His. (1 John 2:6). The flesh is also in us with all its evil desires; but if we walk in the Spirit, we shall not fulfill the lusts of the flesh. (Gal. 5:16).
In the power of this new life, and by the Spirit, we are now to yield our members, the members of our bodies, to God, as instruments of righteousness. (Rom. 6:11-14,19). Once these very members were the servants of sin. Now our bodies are to be for His glory.
We have instructions in the Word for our whole life: Our food, our clothing — every mercy we receive, we receive from our Father, and we are to give thanks to Him for them (1 Tim. 4:3-5). It is sanctified unto us by the Word of God and by prayer. His Word gives all to us, and we thank God as the giver. It does not belong to us till we give thanks.
Apply this to your tobacco chewing, snuffing and smoking. Does your Father give it to you? Do you return thanks to Him every time you use it, as you do for your food? Is it not a worldly lust, an unclean habit that you have acquired? It sickened you at the first, and now it holds you in its grip, and you allow it; and, alas! in many instances the children of God seek an excuse to gratify their evil nature in this way. They do not want to admit that it is sin, but all lawlessness is sin (1 John 3:4 JND). It is pleasing to themselves, and pleasing ourselves is not pleasing the Lord. Nor are they working out their own salvation with fear and trembling (Phil. 2:12-13). They are not in this respect allowing God to work in them both to will and to do of His good pleasure. It is monstrous to say, It does not hinder the communion of those who use it. Could one in the presence of the Lord go on with it?
“Having therefore these promises, beloved, let us purify ourselves from every pollution of flesh and spirit, perfecting holiness in the fear of God” (2 Cor. 7:1).
Question 70: When we go to be with the Lord, do we receive a glorified body immediately? L. M. P.
Answer: When a believer departs to be with Christ (Phil. 1:23), his spirit is with the Lord, and his body returns to dust.
When the Lord returns for His saints (John 14:3; 1 Cor. 15:52; Phil. 3:20-21; 1 Thess. 4:16-17); then the body and spirit will be united again, and it will be a glorified body. The living believers which are then on earth, will have their bodies also changed and glorified.

Inspiration of the Scriptures: The New Testament, Part 7

THE NEW TESTAMENT
THE REVELATION
When we look into the book of Revelation, every precaution seems to be taken to guard souls from hesitating to accept it as a God-breathed or inspired writing; so much so, that it appears as if the present widespread thought of its being a book of such obscurity and difficulty that no one can understand it, and that it was never intended to be read very much, had been anticipated. We are therefore not only told it is “the Revelation of Jesus Christ,” but so thoroughly divine in its origin that it is what “God gave unto Him to show unto His servants things which must shortly come to pass” (Rev. 1:1). John also was commanded by the Lord, saying, “Write the things which thou hast seen, and the things which are, and the things which shall be hereafter (or after these)” (Rev. 1:19). Nothing then as to inspiration can be clearer; and as if this would be to some not enough, John was “in the Spirit,” in order to see the visions, and having seen them, he is told twice in the first chapter to “Write”; and write, too, not only what he had seen, but the things that are now going on, and the things that will be after these things. In the second and third chapters, which show us the things which are, we are seven times told to hearken to “what the Spirit saith unto the assemblies”: so fully are we in the region and sphere of the Holy Spirit’s activities on earth.
And further, the apostle was by the Spirit taught to distinguish between what was of Satan and what of God in the visions which passed before him. In Revelation 17, he was in the Spirit, and saw the harlot in all her worldly attire and luxuriousness, and self-complacency in the wilderness: in Revelation 21, the Spirit also showed him the bride so dear to our Savior’s heart, the Lamb’s wife, but he had to go up in the Spirit for that blessed vision. He says the angel “carried me away in the Spirit to a great and high mountain, and showed me that great city, the holy Jerusalem, descending out of heaven from God, having the glory of God” (Rev. 21:10) etc. At the end of the book of Revelation we are told that “the Spirit and the bride say, Come” in response to our Lord’s presentation of Himself as “the bright and morning star.” So that whatever we may think, the Bible does not close its communications to us without assuring us, that those led and taught of the Spirit will look up to our Lord Jesus Christ where He is, and say, “Come.” Till that moment, His grace will be with us in all its sufficiency and reality.
But if the opening of the book gives such encouragement to the reader of it, and to any who hear it read, the close is solemn beyond all thought in proof of its being divinely given truth: truth, too, which harmonizes and is interwoven by lines running all through Scripture, and this makes it so very solemn. It is said, “If any man shall add unto these things, God shall add unto him the plagues that are written in this book; and if any man shall take away from the words of the book of this prophecy, God shall take away his part out of the book of life, and out of the holy city, and from the things which are written in this book” (Rev. 22:18-19). How solemnly all this admonishes us not only to read and hear what the book says, but to receive its sayings into our hearts and minds in all simplicity as it is written; for if our Lord sends this book to us from Heaven, we may be sure that He would say of it no less than when He spake on earth saying, “Heaven and earth shall pass away: but My words shall not pass away” (Mark 13:31). We have then in reading Scripture to do with God, “not as the word of men, but, as it is in truth, the Word of God, which effectually worketh also in you that believe” (1 Thess. 2:13).
(Continued and to be Continued).

What God Is Doing

Take two illustrations of what God is now doing by the gospel. One from the higher circles, and one from the lower — or, rather, from the lowest of the low.
First: Alice was an only child, an heiress, Lovely and accomplished, she lived for this world, and this world offered her no ordinary attractions. Idolized by her parents, and beloved by an accepted suitor, she knew not the meaning of a wish ungratified. But an unexpected visitor arrived at the mansion. A pale messenger came to Alice. A hectic flush suffused her beautiful face, rendering it, if possible, more lovely still. The eagle eye of affection soon perceived that the seeds of consumption had been laid. The skilled physician pronounced the heart-rending verdict that her days were numbered, and that the career of love and self-indulgence would soon close.
Alice sank by degrees, and as she lay on her couch, surrounded with all the luxuries that wealth could procure, began to think how sad it was to leave her loving friends and all her brilliant prospects, and to go — where — where?
She could not find an answer satisfactory to her soul. So she sent for the high church clergyman.
He came. The family were assembled. He produced a missal. They all knelt round the bed. He intoned the service for the sick. Having received her confession, and pronounced absolution, he, with peculiar genuflections, administered the sacrament, and placing his hands on her, blessed her, and pronounced her a good child of the church. He departed, perfectly satisfied with his own performances, and assuring the parents that all was right.
Was Alice satisfied?
She had submitted to all. She had endeavored to join in the service, but in her inmost soul she felt a blank.
“Father,” said she, “I am going to die. Where am I going?”
The father gave no reply.
“Mother, darling, can you tell me what I am to do to get to heaven?”
No reply save tears.
“William, you who were to be the guide of my life, can you tell me anything of the future?”
No response.
“I’m lost! lost!” she exclaimed. “Am I not, father? Is there any one who can tell me what I must do to be saved?”
At length the father spoke.
“My child, you have always been a dutiful daughter, and have never grieved your parents. You have regularly attended the Abbey Church, and helped in its services, and the minister has performed the rites of the church, and expressed himself satisfied with your state.”
“Alas! father, I feel that is not enough. It is no rest to my soul. It is hollow — it is not real. O! I am going to die, and I know not where I am going. O, the blackness of the darkness! Can no one teach me what I can do to be saved?”
Blank despair was pictured on her countenance. Misery overshadowed the circle. They were overtaken by a real danger; Death was in their midst. Eternity was looming before them.
They knew not how to answer the agonizing appeal of an immortal soul, awakened to a sense of sin — to a dread of appearing before God — to the terrors of Hell.
Alice was attended by a little maid, who was in the habit of frequenting a meeting held in a barn in the village, where prayer and praise were offered in simplicity, and where they sang the old hymns
“There is a fountain filled with blood,
Drawn from Immanuel’s veins,
And sinners plunged beneath that flood,
Lose all their guilty stains.”
and
“God laid my sins on Jesus,
The spotless lamb of God;
He bears them all, and frees us
From the accursed load,”
She longed to tell her mistress that she might “wash and be clean,” but felt diffident. At last she took courage and just as the Israelitish captive said unto Naaman’s wife, “Would God my lord were with the prophet that is in Samaria, for he would recover him of his leprosy,” she told her mistress: “There is a preacher in the village who proclaims salvation through faith in the Lord Jesus Christ, and urges us to accept the forgiveness freely offered in the gospel.”
“O! that I could see Him,” exclaimed the dying girl.
Alice besought her father to invite the strange preacher to the house; and though he thought it extraordinary, her wish was law.
Again the family were assembled, and the man of God entered the room. The dying girl, raising herself, appealed to him. “Can you tell me what I must DO to obtain rest for my soul, and die at peace with God?”
“.... I can tell you what HAS BEEN DONE for you. Jesus Christ, the Savior God, has completely finished a work by which lost and helpless sinners may be righteously saved. God, who is love, saw us in our lost and ruined state. He pitied us, and in love and compassion sent Jesus to die for us. ‘God so loved the world that He gave His only begotten Son that whosoever believeth in Him should not perish, but have everlasting life.’ He shed His precious blood on the accursed tree in the stead and place of sinners, that they might be pardoned and saved. ‘Believe on the Lord Jesus Christ, and thou shalt be saved.’”
“And have I nothing to do?” “Nothing but to believe. No doing, working, praying, giving or abstaining, can give relief to the conscience burdened with a sense of guilt, or rest to the troubled heart. It is not a work done in you by yourself, but a work done for you by another, long, long ago. Jesus has completed the work of our redemption. He has said, ‘It is finished.’ Through faith in Him you have pardon. It is impossible for a sinner to do aught to save himself. It is impossible to add anything to the perfect work of Christ. Doing is not God’s way of salvation, but ceasing from doing, and believing what God in Christ has already done for you. `God hath given to us eternal life and this life is in His Son.’”
“I do believe that Jesus died on the cross for sinners; but how am I to know that God has accepted me?”
“Jesus, the God-man, has ascended into heaven. He has presented His blood before God, and has been accepted for us; and when you believe, you are accepted in Him.”
The awakened sinner listened with breathless attention. She received the Word of God, which revealed Christ to her soul. The glad tidings of salvation fell as balm upon her wounded spirit. Her face was lit up with heaven’s sunlight; looking upward, she exclaimed, “O, what love! what grace!
“‘Jesus Thy blood and righteousness,
My beauty are, my glorious dress,’”
and in a few days she departed to be with Christ.
(To be continued.)

Go out Quickly

“Go out quickly into the streets and lanes of the city, and bring in hither the poor, and the maimed, and the halt, and the blind.” Luke 14:21, 23.
The Lord is coming quickly,
And darker grows the night;
The air is hanging thickly,
And evil near its height.
Alas! how truth is sunken,
And trampled in the streets;
Some drink, too, with the drunken,
And eat their dainty meats.
O, brethren, then awaken,
Arise, ye sons of day;
We soon shall hence be taken,
Now let us work and pray.
In every town and village,
At many a cottage door,
Is soil awaiting tillage,
Is fruit among the poor.
Go forth, ye earnest preachers,
Where God shall guide your feet,
And tell His sinful creatures
How He their souls will greet.
Proclaim in lane and alley,
How Christ, the Savior, died;
And, risen from earth’s valley,
He now is glorified.
Lift up your eyes enlightened,
And look upon the fields;
Already are they whitened,
The crop abundance yields.
God, in His grace, engages,
That they shall rest who toil;
The reaper shall have wages,
The sower share the spoil.
Then, heed we not the weather,
Nor care for toil and heat;
We all shall rest together,
When Christ has housed His wheat.
Then, gone all grief and sadness,
In yonder place we’ll roam;
Rejoice in Christ’s own gladness,
And hail His harvest-home.

Scripture Study: Matthew 13, Part 2

Matthew 13:24-32
The six following parables are similitudes of the kingdom, not as set up in power, but as during the absence of the rejected King. The first three describe its outward form and are told in the hearing of the multitude (Matt. 13:2). The last three present it as what is precious to God in it, and are told to the disciples alone inside the house (Matt. 13:36); they also get the explanation of the parable of the tares and wheat. The Lord Jesus is rejected on earth as King. The Jews in rejecting Him, condemned themselves. Now it is to be a kingdom formed by sowing the Word of God. It is not power exercised in righteousness and judgment; that will be when the kingdom appears again; now it is the Word bearing testimony to the hearts of men, and using men to spread it; but God alone can give the increase. He sends the servants and watches over the harvest.
Matthew 13:24-30. The parable of the tares gives a general idea of what follows the committing of the kingdom here below to the care of men. A man sowed good seed in his field; but while men slept (men should have watched), his enemy came and sowed tares. This made the field look unlike the Lord’s work; the Lord does not sow that kind of seed, but the carelessness or weakness of the servants let the enemy do his work to bring in those into the field that did not belong to the Lord. This is the evil we see in that which bears the name of Christ. The Lord Jesus and His servants sow good seed. Satan and his servants sow bad seed ̶ teachers of law; teachers trying to uplift the people without redemption or the new birth; and grievous wolves have all found an entrance into it.
Can these tares be rooted out? The servants want to try; the Master says, “no.” The mixed condition is to go on to the time of the harvest. Men who lacked spiritual discernment, could not keep the evil from entering, and were not fit to put the evil out after it had come in. So the state of the kingdom is seen here as good and evil growing together in the field ̶ another witness that everything committed to man fails.
We must take care not to confound this with Christian fellowship in the assembly. There we are distinctly told to “put away from among yourselves that wicked person” (1 Cor. 5:12-13). It is never the duty of a Christian to have fellowship with evil. (2 Cor. 6:17-18; 2 Tim. 2:19). At the time of harvest, the tares will be bound in bundles by the reapers, but the wheat will be gathered into His granary. The servants now are to let the tares alone, and to be occupied with the good. It needs greater knowledge and wisdom than men have to discern who are the Lord’s now. It is the tares that are to be bound in bundles. The Lord will take the wheat to Himself.
Matthew 13:31-32. Here we see how that out of a small seed a tree has grown. A tree in Scripture is used to symbolize a power in the world: (See Ezek. 15; 17; 31; Dan. 4). So the profession of the name of Christ has become a great power among men, and under it others find shelter, as birds under a tree.
But, alas! in truth, every kind of evil shelters itself under the profession of Christianity in the persons of men who are Satan’s servants. (2 Cor. 11:13-15).
(To be Continued.)

Truths for Young Christians: Deceit and Lying

Chapter 16
DECEIT AND LYING
This is one of the special sins connected with the tongue, that unruly member which no man can tame. Over and over again it is emphatically forbidden and condemned by the God of truth (Col. 3:9; 1 Peter 3:10; Prov. 24:28; Prov. 12:22). When Peter speaks of the life of the Lord, NO DECEIT IN CHRIST, as an example for us to copy, he emphatically points out that no deceit was found in His mouth. Those who are deceitful are therefore evidently most unlike Christ; but not only so, but those who practice lying are children of Satan (John 8:44), and are expressly excluded from heaven (Rev. 21:27; 22:18), and condemned to hell (Rev. 21:8). As we write for those who are professedly children of God, in looking at a few examples of this fearful sin in Scripture, we will only take those where a child of God, or at least a professor, is concerned.
Lying Through Fear
We find, in Genesis 18:15, Sarah telling a direct lie through fear. How often is this the case, resulting from having done or said something we are ashamed of? It may be a right thing, and we are thus ashamed of Christ; or, it may be a wrong thing, and we are ashamed of being found cut. In either case a lie slips from our lips ere we are aware. The radical cure for this is not to do what we are ashamed of; or, if the thing is right, not to be ashamed of what we do. If, however, we have slipped into a sin, let us not add to it by another, but if the lie is about to leave our lips, let the thought “GOD HEARS ME” instantaneously arrest it. A lie of this sort to screen oneself is, perhaps, the most contemptible kind, despised alike by Christians and men of the world. Having thus looked at it, let us resolutely avoid it, even in the smallest things, and never lend our tongues to such mean deceit.
Lying for Our Own Advantage
The next instance is in Genesis 27:19, when Jacob tells a direct lie for his own advantage — another despicable variety of this hydra-headed sin. Mark, too, Jacob was a child of God, and the result is that through the next thirty years of his life he suffered from the consequences of his sin; by which, too, he gained nothing, for God would have given him all in due time. Have any of my readers fallen victims to this sin? Making haste to be rich, or improve their position, or in some way run in advance of God; have they ever, through selfish motives, told a lie? If so, I am sure they have suffered since, and there can be no real restoration until that lie is confessed not only to God but to man. Too often, alas! one lie leads to another, as in Jacob’s case, and once embarked on this fatal course who can tell what the end will be? Oh! beloved reader, I plead with you: never, never allow yourself to tell a lie for your own advantage. Think for one moment what a horrible denial such a sin is of all that Jesus ever was or did.
Lying to Cover a Sin
Passing over several, we come to David, who was guilty both of lying (1 Sam. 21:2) and deceit (2 Sam. 11) of the most fearful character by which he sought to cover up an awful sin, thereby making it twice as bad. O, how often some previous sin is the cause of a long course of deceit and lying. Beloved friends, let us, above all things, seek to be straight with God, with our fellow-men, with ourselves; and should we fall into a sin, never, never seek to cover it up by another, still worse than the first. A course of deceit positively blights the soul, destroying all simplicity, all joy, all communion. The result of these sins in David’s case was a course of sufferings almost unparalleled in their severity from the hands of his own children. Let not us, therefore, think to escape the all-searching eye of God.
Lying From Habit
We find in 1 Kings 13:18, a prophet of God lying in a most wanton manner, without any apparent reason. We find some such characters now, even amongst God’s people, some who apparently have no regard for the truth, and find it easier to tell a lie than to avoid it. The only remedy when the disease has so developed is to go straight to God, and cry to Him for strength and daily watchfulness to overcome it. One such case I remember. I noticed that a person was almost always silent, and one day asked the cause. He said that he had been so addicted to lying that he was determined now not to speak at all if he could not speak the direct truth; and, therefore, he seldom opened his lips, and always considered well before he spoke. Deep-rooted sins require some such radical measures.
Two Solemn Cases of Lying
In the New Testament the two solemn cases, one of lying and the other of deceit, in Peter and Ananias, stand out above all others. Peter, forewarned by the Lord, yet strong in his own strength, told three lies to save himself, actually going the length of denying the Savior while he was standing dumb before His accusers. Such sins are, alas! not unknown even now. Many of us are ashamed of showing our colors, and when suddenly asked an unexpected question, through fear or shame, are betrayed into a lie to the triumph of Satan and the grief of our Lord. Let us watch earnestly against this; and, if entrapped, let us follow Peter in his path of restoration. It is remarkable to see that the very one who fell himself is so perfectly restored as not only to be able to charge home the very same sin to the Jews (Acts 3:14), but was also chosen by God to be the executor of His justice on the flagrant deceit of Ananias. This, too, was a wanton sin — a course of deceit being practiced merely to give others a false impression of his generosity, and to appear other than he was.
This, alas! is another common variety of this sin. Anxious to stand well in the eyes of our fellow men, rather than in those of God, we do not hesitate sometimes to descend to deceitful practices to appear other than we are, and so get praise from men that we do not deserve. Surely such a course needs only to be named to be condemned by every upright heart. All these instances have been selected from the lives of professing children of God, and will well repay careful consideration; giving, as they do, striking illustrations of the main causes of deceit and lying among Christians. Lies may be told without using the lips; we may act so as to deceive, and seek to excuse ourselves because we have not said what is untrue. This is a worthless subterfuge, and will not stand before God for a moment. All such refuges of lies will He sweep away.
The only way to be happy before Him, and to be in any degree like Christ, is to turn our backs firmly and resolutely on deceit in every shape and form by word or deed; and determine, in God’s strength, that we will earnestly seek to say and do nothing that is not absolutely true, thus saving ourselves from reaping the bitter fruits of shame and sorrow that will some day follow. May God help each one of us that is tempted by this sin to overcome it in His strength, and to learn to abhor and hate it because it is so hateful to Christ and so dishonoring to His name.
“The lip of truth shall be established forever, but a lying tongue is but for a moment” (Prov. 12:19).

Hezekiah: Brief Lessons on Church Truth, Part 1

A BRIEF OUTLINE OF LECTURES ON HEZEKIAH
Read carefully 2 Chronicles 29.
It is important to notice, that, at this time, both Judah and Israel had utterly departed from the Lord. Sad, and low indeed, was Judah’s condition, as described in 2 Chronicles 28; all was wrong; all apostasy and idolatry. What a hopeless picture! But a picture drawn for us — written for us. Is it not a picture of all around? A man said to me the other day, as an excuse for remaining in what he knew to be wrong, “I have read, and compared the Acts, the early days of the church, with all I see now; and all is so different from what I read, that I have no hope of things being right, and so I go on as I am.” In contrast with this man, we read of Hezekiah, “And he did that which was right in the sight of the Lord” (2 Chron. 29:2). Yes, in the midst of all that was wrong, he did that which was right; and mark, not in his own sight, not in his own opinion, but in THE SIGHT OF THE LORD. This, then, is the foundation-principle of our present lesson: God can raise up a man, can enable His child, to do that which is right in the sight of the Lord, in the midst of all that is wrong.
These things were written for our instruction, and how very striking the analogy! Has not Christendom departed as far from the inspired teaching of the Holy Ghost, as Judah had departed from the inspired words of Moses? When the one is seen as a picture of the other, then every verse contains instruction to our souls.
Let us notice three things, especially illustrating the present condition of Christendom: First, “They have shut up the doors of the porch.”
Second, “And put out the lamps.”
Third, “And have not burned incense, nor offered burnt-offerings, in the holy place” (2 Chron. 29:7).
If we look at the established Church of God, as found in the beginning, we find the way into the holiest open; every believer, having boldness to enter the holiest by the blood of Jesus (Heb. 10); perfected forever by the one offering; all purged worshipers in the unclouded presence of God. “Giving thanks unto the Father, which hath made us meet to be partakers of the inheritance of the saints in light.” Delivered from the power of darkness; translated into the kingdom of His dear Son; absolute certainty as to redemption and forgiveness of sins; all trespasses forgiven; sins to be remembered no more, no more; immutable peace with God, according to all that God is; no longer afar off, but so near, in all the fullness of the Father’s love. (Col. 1:12-14; 2:13; Heb. 10; Rom. 5).
Compare all this with the state of Christendom for centuries. Read all prayer-books — Roman, Greek, Anglican, and especially the sad, despairing wail of the Ritualists. Yea, hearken to the pulpit prayers of all Christendom. Is this the worship of divine certainty — that sins have been atoned for, and, having been confessed to God, are all forgiven? Hark, is this the worship of the Christian in the holiest, in perfect peace with God? Has not Christendom practically shut up the doors? and, instead of the worship of the purged worshiper inside the veil, is it not taking again the place of the Jew afar off, crying for mercy, just as the Jew did before redemption was accomplished? Is not this saying we are Jews, when we are not, the sin of unbelief? Is it not denying that Jesus has come in the flesh, and finished the work — that work of redemption — which the Father gave Him to do? Do not millions still pray as Jewish disciples were taught by the Lord before His death and resurrection, “Forgive us our sins, as we forgive them that trespass against us”?
In contrast with the Christian’s thanksgivings now, “we have redemption through His blood, the forgiveness of sins.” The Scripture says to all Christians, “Having forgiven you all trespasses” (Col. 2:13). Christendom says, No; we must keep praying to God as miserable sinners, hoping that God will forgive us. O, how sad our unbelief! Have we not also shut up the doors? This is so solemn, that I must dwell upon it a little longer. If it were sad for Judah to shut up the doors, is it not far more so now? “Then, verily, the first covenant had also ordinances of divine service, and a worldly sanctuary”. And there was still the veil that shut man out: no saint, not a David, or an Isaiah, could pass that veil. Now the established church, as found in Scripture, was the very contrast of all this. No worldly sanctuary, and no veil to shut out the least of all saints: the veil was rent, and there was boldness to enter by the blood of Jesus. The believer’s calling and worship is heavenly. O, how sad the departure from God’s established church! to go not only back to a worldly sanctuary and ordinances, but practically to hang up again the rent veil, and shut up the doors. Not only so, they had “PUT OUT THE LAMPS.”
What did the seven lamps of the sanctuary typify? Was it not the Spirit of God in the assembly, or church? The all-sufficiency of the Holy Ghost? Those seven lamps were to be lighted, “that they may give light over against it” (the golden candlestick) — the very command of Jehovah, when He spake the first time, from between the cherubim. (Num. 8). And in that light stood the table of showbread — the twelve representative loaves, borne on that table, covered with pure gold, and the loaves covered with frankincense. What a picture! The whole redeemed children of God sustained, borne, in divine righteousness, before God, and covered with all the preciousness of Christ. Not now, however, twelve loaves — there were twelve tribes of Israel — there is only “one body,: and therefore only one loaf on the table of the Lord. But as all the particles of bread are chemically one loaf, so all Christians form spiritually the one body of Christ. All borne in divine righteousness before God — all covered with the perfections of Christ — one with Christ, the Head.
But where was the beauty of all these golden shadows of Christ when the lamps were put out?
All was darkness. Can this be a picture of Christendom? If such was the place and importance of the seven lamps of the temple, what was the place and importance of the Holy Ghost in the divinely established Church of God? Did you ever notice how much this occupied the thoughts of Christ that last night He spent with His disciples? “I will pray the Father, and He shall give you another Comforter, that He may abide with you forever; even the Spirit of truth; whom the world cannot receive,” (John 14:17).
Again, the promise is repeated in John 14:26, “He shall teach you all things.” Again, John 15:26, and much of John 16, is occupied with this all-important assurance of the coming and presence of the Spirit. It was even expedient that Jesus should go away, that the Spirit might come. And just as the typical lamps gave light over against the candlestick, so the Holy Ghost “will guide you into all truth.” And “He shall glorify Me; for He shall receive of Mine, and show it unto you.”
When redemption was accomplished, and Jesus received up to the right hand of God — Pentecost being fully come — the Holy Ghost, was sent down to take His place and form the Church of God. Thus, as when the lamps were lit, all was light in the sanctuary; so, when the Holy Ghost had His place in the Church of God, all was light.
What a reality this was! Take just one instance. The church was gathered together; the Holy Ghost said, “Separate me Barnabas and Saul, for the work whereunto I have called them;” and these beloved, honored ministers of Christ were sent to that special mission by the Holy Ghost. (Acts 13:1-4). And His divine place is fully recognized in the epistles. (See 1 Cor. 12; 14, “Dividing to every man severally as He will.”) O, think of the established church of the Scriptures! The Holy Ghost revealing the glory of Christ, using whom He will, in the assembly. The prophets thus speaking, two or three. O, sad solemn fact, for many, many centuries, man, as far as lies in him, has shut up the doors, and put out the lamps. Yes, as far as possible, the Holy Ghost has been displaced, until Christendom is so conscious of its distance from God, and its darkness, that it is constantly taking the place of the Jew before Christ died, and the Holy Ghost was sent down to abide to the end. Its prayers are those of the Jew afar off — cries for mercy; and on all sides, in the dark, may be heard prayers for the Holy Spirit to come, as though the Father had refused to send Him, and He had not come. Is not all this far more sad than the state of Judah in the days of Hezekiah? Yes, for centuries cries for mercy, distance and darkness, instead of incense and burnt-offerings, worship and adoration, in the full, blessed enjoyment of our acceptance, in all the sweet savor of Christ.
(To be Continued)

The Coming and Reign of Our Lord Jesus Christ: The Age to Come, or the Millenium, Part 1

The Age to Come or The Millennium
The millennium, or the thousand years, is the period in the future of this world’s history, when the Son of Man, the Lord Jesus Christ, shall take into His own hands the rule and government of the world. The thousand years are mentioned no less than six times in the first seven verses of the twentieth chapter of Revelation, and doubtless are to be taken in their literal import. The Scriptures teem with descriptions of the blessings and glory to be enjoyed by this now groaning creation in that glorious age, or world to come. Peter, in addressing the Jews in Acts 3:20-26, speaks of the sending of Jesus Christ, “whom the heaven must receive until the times of restitution of all things, which God hath spoken by the mouth of all His holy prophets since the world began. * * * Yea, and all the prophets from Samuel, and those that follow after, as many as have spoken, have likewise foretold of these days.” The Lord referred to the same period, when He said to His twelve disciples, “That ye which have followed Me, in the regeneration, when the Son of Man shall sit in the throne of His glory, ye also shall sit upon twelve thrones, judging the twelve tribes of Israel” (Matt. 19:28).
The despised and rejected Jesus of Nazareth has been cast out and crucified here, but “God also hath highly exalted Him, and given Him a name which is above every name; that at the name of Jesus every knee should bow, of things in heaven, and things in earth, and things under the earth; and that every tongue should confess than Jesus Christ is Lord, to the glory of God the Father” (Phil. 2:9-11). All who bow receive the priceless gift — eternal life; are called to suffer with the rejected One now, and will appear and reign with Him by-and-by; those who refuse will surely come into judgment. But there is a moments coming when “every knee shall bow” ( Isa. 45:23), when “all kings shall fall down before Him; and all nations shall serve Him” (Psa. 72:11). “And the Lord shall be King over all the earth; in that day shall there be one Lord, and His name one” (Zech. 14:9).
What joy for the heart of the Christian to know that the Blessed One in whom he has trusted, the One whom he has not seen, but yet loves (1 Peter 1:8) whose rights in this scene have been usurped by Satan, the prince of this world, will shortly have all things manifestly under His sway. Then “the government shall be upon His shoulder: and His name shall be called Wonderful, Counselor, The Mighty God, The Everlasting Father, The Prince of Peace. Of the increase of His government and peace there shall be no end, upon the throne of David, and upon His kingdom, to order it, and to establish it with judgment and with justice, from henceforth even forever. The zeal of the Lord of hosts will perform this” (Isa. 9:6-7).
(Continued and to be Continued).

Correspondence: Heathen Lost; Luke 14:26; Matt. 6:9

Ques. 71. If a man is saved by believing the gospel, are the heathen lost who have not heard it? Would it be justice? R. B.
Ans. Adam’s sin brought ruin upon all his posterity, involving all his race in the consequences of his defection from God (Rom. 5:12-21), but Christ died for all, and no one will be eternally lost because he is born a sinner, but because of his sins. He is a responsible being, and God is holy. Read carefully Romans 1 and see the fearful downgrade that man has pursued. Against this wicked course the wrath of God is revealed from heaven (Rom. 1:18-21). It is for his sins every man will be judged. “Because of these things (their sins) cometh the wrath of God on the children of disobedience” (Eph. 5:5-6). “He is alienated and an enemy in his mind by wicked works” (Col. 1:21). Babes often die in infancy; death proves they are sinners by birth, yet they are not lost, because “The Son of Man is come to save that which was lost” (Matt. 18:11). This passage is speaking of babes. In Luke 19:10 notice the difference; there it speaks of a grown-up person, who has sinned: “For the Son of Man is come to seek and to save that which was lost.” Every person is responsible for his sins. Can he put them away? No. Nothing but the work of Christ applied to the soul can do that. God in grace sent His Son to die. All who believe on Him receive remission of sins (Acts 10:43) and none can be saved without Him (Acts 4:12). God in grace has provided salvation. If He gives the sinner justice, the sinner will be eternally lost. A guilty man cannot plead for justice, but he can own his sin and cry for mercy; he can be cleansed by the blood of Christ alone (1 John 1:7).
Ques. 72. Please explain Luke 14:26. E. G.
Ans. This verse teaches us that Christ is to be our first object. His claims upon us are greater than the claims of our nearest relation. We are not to own their authority over us above His. It is in this way only that we are to hate them. We are to put Him first, as dearer than our own life.
In the epistles, the believer is looked upon as being dead and risen with Christ. (Eph. 2:6, Col. 3:1). This is the Christian position, consequently the Lord tells the believer all he is to do. It is the Lord that says, “Wives, be subject to your husbands”; “Husbands, love your wives”; “Children, obey your parents”; “Fathers, train up your children”; “Servants, obey your masters”; “Masters, give unto your servants that which is just and equal.” Thus the Lord gives us back those relationships to live in for Him. This puts Him first.
Ques. 73. Are expressions such as “Heavenly Father”, “Our Father which art in heaven”, in relation to an earthly people? And in John’s gospel, “the Father,” “the world,” “not of the world as I am not of the world,” refer to a heavenly people? R. B.
Ans. When the Lord Jesus came into this world, He began the revelation of God as Father. In John 17:26, He says, “I have declared unto them Thy name,” which refers to His lifetime on earth; “and will declare it,” goes farther, and takes in the full revelation of Christianity. In the first two gospels we have such expressions as “Heavenly Father,” and “Your Father which is in heaven.” These would apply to an earthly people. In Luke 11:2, we read, “which art in heaven”, which is an interpolation. (See J. N. D.’s and other translations.) Luke 11:13 should read, “How much rather shall the Father who is of heaven,” or “who from heaven will give the Holy Spirit to them that ask Him.”
So that Luke and John were writing more in view of the present time, when every Christian has the Holy Spirit and says, “Abba, Father” (Rom. 8:15-16; Gal. 4:6). In all the Epistles we see this nearness and heavenly character. We are children of God and enter within the veil. We are also to be with the Lord in heaven, we share His place there. (John 14:1-3; 1 John 3:1; Eph. 2:18; Heb. 10:19). This place was given to the disciples first when Jesus rose from among the dead (John 20:17), and belongs only to the present period of grace. Israel and the Gentiles in the Millennium will not have it, they are the earthly people.

Inspiration of the Scriptures: Quotations from the Old Testament

Quotations From the Old Testament
It is deeply interesting to observe with what care the Holy Spirit, by the various writers or speakers in the New Testament, quotes from the books of the Old Testament. Of course this work, like all God’s works, must be perfect; but such accuracy is observed that it is not uncommon for a quotation to end in the middle of a sentence, and for obvious reasons. We all remember the account of our Lord’s reading from the prophet Isaiah in the synagogue, and that when He had uttered the words, “to preach the acceptable year of the Lord,” He closed the book and sat down. Now why was this? Why did He not read on? Because the next words, “the day of vengeance is at hand” did not then, apply, as His mission was one of grace, not judgment. But in its literal application to the Jews, the time of their blessing and of divine vengeance will come together; for when the Lord appears as “Sun of Righteousness” with healing to the faithful remnant of Israel, He will so execute judgment on the wicked, that they shall be ashes under the feet of those who are blessed. The application, therefore, of that scripture by our Lord, and up to this time, can go no further than the acceptable year of the Lord, however rapidly the day of vengeance may be approaching.
On turning to Romans 8, we find a quotation from Psalm 44, “as it is written, For Thy sake we are killed all the day long; we are accounted as sheep for the slaughter;” and there the apostle stops, because, on referring to the psalm, the next words are such as could not be now expressed by a child of God, who is loved by the Father as He loved His Son. They could, however, be the utterance of a Jewish remnant who know not the Father, nor the blessedness of the accomplished redemption of the Son. Such might add, “Awake, why steepest Thou, O Jehovah? arise, cast us not off forever. Wherefore hidest Thou Thy face”; but it is not the Christian’s utterance who knows the liberty wherewith Christ has made him free.
Look also at Romans 10:15, which is a quotation from Isaiah 52, “As it is written, How beautiful are the feet of them that preach the gospel of peace and bring glad tidings of good things!” Now, why does the apostle stop there in the middle of the sentence, and omit what follows? Because it is clear that what follows, “that saith unto Zion, Thy God reigneth,” cannot have its fulfillment till the Lord Jesus takes His rightful place as Son of David, and reigns before His ancients gloriously as King of Israel.
In 1 Peter 3, there is a quotation from Psalm 34, “The eyes of Jehovah are over the righteous, and His ears are open unto their prayers; but the face of Jehovah is against them that do evil,” and stops in the middle of a sentence; and why?
Because the words which follow, “to cut off the remembrance of them from the earth,” while literally applying to God’s earthly people, the Jews, could never be applied to those who are saved from coming wrath, and instead of inheriting the earth, are looking for “an inheritance incorruptible, and undefiled, and that fadeth not away, reserved in heaven.” Do not such things show us the perfect accuracy with which the Holy Scriptures come from God to us?
But take one or two more examples. Psalm 68 is quoted in Ephesians 4. We read, “Wherefore He saith, When He ascended up on high, He led captivity captive, and gave gifts unto men,” and stops in the middle of the sentence, because the next words, “yea, for the rebellious also,” though they will be literally fulfilled as to rebellious Israel, could never be used toward us, who now through grace, have believed the gospel. In millennial times, when Christ will be known as Israel’s Redeemer and King, they will deeply and touchingly feel the marvelous character of the grace, above all else, which will give them “gifts” from the hand of their long-rejected Messiah.
Again, in Hebrews 2, the writer quotes freely from Psalm 8, in reference to Jesus having been made a little lower than the angels for the suffering of death, whom we see now crowned with glory and honor, and that He will have all things put in subjection under His feet, though that has not yet taken place. Now Psalm 8 is millennial, and applies strictly to Jesus, as the last Adam, who will take His rightful place of dominion over all God’s works, “all sheep and oxen, yea, and the beasts of the field; the fowl of the air, and the fish of the sea, and whatsoever passeth through the paths of the seas,” when it will be said, “O Jehovah, our Lord, how excellent is Thy name in all the earth.” It is obvious that the apostle, by the Spirit, only quotes from this Psalm enough to show that Jesus, who has tasted death for every man, is now “crowned with glory and honor,” and will, by-and-by, have all things put under His feet.
These examples are enough to show, not only the divine authority of the Old Testament writings, and regarded by the apostles as final and conclusive, but they also reveal to us the fine and delicate workmanship of the Holy Spirit in putting together for our profit the scriptures of the New Testament.
It is a point not to be overlooked, that the Old Testament scriptures are either quoted or alluded to in the New many hundreds of times.
(Continued).

What God Is Doing

Part 2
About this time the gospel tidings had spread over a large tract of country, and many were the subjects of divine blessing. In visiting the cottages, from place to place, I heard of a notorious reputed witch, whose evil power was stated to be fearful in the destruction of stock, and in turning all the dairies into utter confusion. The belief prevailed that the losses of persons who have the misfortune to offend these so-called witches, were very serious. A godly woman, near to whom I lived, had experienced some troubles of this kind, and the statements of various individuals, who bore witness to what they had seen of the disasters occurring for many days successively, led me to visit the old woman who bore this dreadful character. The people strongly dissuaded me from it, saying she had sold herself to the devil, and that it was not safe for anyone to go to her.
Turning a deaf ear to all this, I called and found her ill in bed; surely, if looks betray character, she certainly had a very bad one. After some inquiries as to her health, and what she was suffering from, I asked her if she expected to get better. She replied, “No.” “Where will you go if you die?” I asked. She stared at me fiercely, like a tiger about to spring from his lair. I gently put my hand on her shoulder, and she screamed out, “I’s going to hell! I’s going to hell! I’s wicked!” “But why do you wish to go to hell?” “I don’t want to go, but I’s forced to go.” “But who is it that forces you to go to hell?” “The devil,” she said; “I have served him all my life.” “But did you never hear of God, and of His Son Jesus Christ, who came down from heaven to save us from going to hell?” “No.” “Did you never hear of God?” I can’t read; I’s wicked.” “But do you not know what love is? Had you a child?” “Yes, I had eight.” “But don’t they love you?” “No, they robbed me.” “Did not your husband love you?” “No, he turned me out of doors.”
“And did you never love anyone?”
“No, I’s wicked, I hate all — everybody.”
Finding all was of no avail, I asked her if she would like a few nice things to eat. “I can’t have it. No one will gee it to I.” “O, yes, I will give it to you, this very night. I will send it to you.” Her amazement was equal to her horror before. “Will ee sure?” “Yes; so you see somebody loves you. Now I want to tell you that someone else loves you, and He sent me to tell you about His love.” “Who is that?” “It is the great God, the King of the world. He lives up there above the sky. This great King made all things. He made you. This great King has one Son, whom He loves very much, because He deserves to be loved; yes, this great King loved you so much that He sent His Son all the way down from heaven to die for those who, like you, have been committing sins all their lives. And He has sent me with this letter to read to you; that you may not go to hell, but to heaven.” I then read to her some of John 3, and sought to instruct her mind, and tried to make her understand who the great King of the sky was, and how He would not turn away from her. “But will He hear a poor old thing like me?” “Yes,” I said, “He will.”
“But, what shall I say to un?” “Just tell Him what you are afraid of. Tell Him what you have told me, that you are wicked.”
She at once looked straight up to the ceiling as if she saw someone there, and said, with all the vehemence of despair, “O Lord, the King of the sky, have mercy on a wicked old ooman like I — I have been a wicked old ooman all my life.”
She kept saying this till she cried bitterly. I then taught her that beautiful passage — “The blood of Jesus Christ, His Son, cleanseth us from all sin.” This she repeated after me until she had it in her memory. I then left her; and before I called again, she sent for me. Her first words were, “The blood of Jesus Christ, His Son, cleanseth us from all sin.”
I asked her who Jesus Christ was. She said, “He is the Son of the King of the sky.” “Well; what has He done for you?” “O, He has died for me!” I need not say much more, only that she found out that God loved her, and this soon made her love everyone. I saw her many times subsequently, and each time found that the Word had taken deeper root in her heart. She confessed to a life of the greatest wickedness, although she said she was not guilty of the crime for which she was forcibly driven from the parish in which I reside, about fourteen months before this time.
The last words I heard from her were, “O, I be a wicked old ooman; but I’s not afraid; ‘The blood of Jesus Christ cleanseth from all sin!’”
“The silver trumpet’s sounding
The year of jubilee;
And grace is all abounding,
To set the bondmen free.
Return, return, ye captives,
Return unto your home;
The silver trumpet’s sounding-
‘The Jubilee is come.’
Forsake your wretched service,
Your master’s claims are o’er,
Avail yourselves of freedom,
Be Satan’s slaves no more.
A better Master’s calling,
In accents true and kind;
He asks a loving service,
And claims a willing mind.
He offers you salvation,
And points to joys above;
And, longing, waits to make you
The objects of His love.
In living faith accept Him,
Give up all else beside;
While grace is loudly calling,
Look to the Crucified.”
With the Lord
How blest the hope, that we shall be
Forever, Lord, at home with Thee,
The “Father’s house” within;
Within that place of cloudless light,
Where never comes one shade of night,
Nor trace of grief or sin.
Blest Lord, to see Thee satisfied,
In having us, Thy spotless bride,
Upon the throne with Thee;
To hear Thee lead the song of praise,
Which wid’ning circles round Thee raise
The nearest circle we.
The seraphim there vail the face,
And seek to understand the grace
Which placed us on the throne;
Then, breaking forth, ascribe to Thee
All honor, glory, majesty,
Who wrought the work alone.
The “living creatures” Thee adore,
The “elders” cast the crowns before
The throne, where sits the Lamb;
The heavenly hosts take up the song,
Which all creation bears along,
To Thee, the great I AM.
Above it all, this richest strain-
“To Him who washed us from each stain
In His most precious blood”-
Shall flow from hearts supremely blest,
And brought by Thee to God’s own rest,
Who once as rebels stood.

Scripture Study: Matthew 13, Part 3

Matthew 13:33-44
Matthew 13:33. The kingdom of heaven is like unto leaven which a woman took, and hid in three measures of meal, till the whole was leavened. This is a general profession of doctrine that leavens all Christendom. Leaven is generally used for evil, but here it is its effect of permeating the whole profession that is before the mind of the Spirit, and judgment is not mentioned. The purpose of the parable is to state the fact of the general existence of the external profession of the name of Christ, leaving it to the spiritual discernment of the child of God to judge of that which so exists.
It is not difficult to see how evil is associated with these two parables. The birds of the air lodging under the branches of the tree, lead us to think of the fowls that took away the good seed sown in Matthew 13:4, and interpreted by the Lord to mean “the wicked one” (Matt. 13:19). And the general appearance of Christendom today is that of a corrupt and corrupting mass.
These three parables then, are the kingdom as viewed on earth by man in its external appearance. A harvest spoiled by the mixture of good and bad. The tree — a great human and political power on the earth.
And the leaven — a general profession of teaching of the name of Christ, without regard to individual state. It is not faith nor life, but a religion it is Christendom.
Matthew 13:34-35. Thus far this teaching is in parables, and to the multitude, that the Scriptures might be fulfilled. We shall see that it is to the disciples who follow Him, and are in private with Him, that the interpretation belongs.
Matthew 13:36. It is at this point that the Lord sent the multitude away and went into the house, and there, in privacy and intimacy, unfolds to His disciples the kingdom according to the thoughts arid mind of God. What a place of privilege the believer is seen in here: It is in the secret of His presence that we learn His mind, and to look at things as He looks at them. It is to the disciples He reveals the true character — the object — of the kingdom of heaven, and the judgment of that which is evil in it, so that all in it might be consistent with the character and heart of God.
His disciples say to Him, “Declare unto us the parable of the tares of the field.” He answered, “He that soweth the good seed is the Son of Man; the field is the world (that is, it goes outside of Israel); the good seed are the children of the kingdom (it produced children suited to the kingdom, its heirs), but the tares are the children of the wicked one; the harvest is the end of the world (or age); the reapers are the angels.”
The Son of Man sowed the good seed; the wicked one, the devil sowed the tares. The children of the kingdom were born of the Word, but the devil, by his sowing, brought in all sorts of doctrines that produced tares (imitation wheat). There is the mixture of the two, but it will not continue forever; the wheat will be gathered into the barn, safely housed; while the tares, having been already bound into bundles (Matt. 13:30), will be burned in the fire. So shall it be at the end of the age — the Son of Man shall send forth His angels, and they shall gather out of His (the Son of Man’s) kingdom, all things that offend, and them which do iniquity, and shall cast them into a furnace of fire; there shall be wailing and gnashing of teeth.
When the Lord Jesus comes for His saints, (1 Thess. 4:16-17) all the believers are taken up — both the dead in Christ and the living. Afterward the Holy Spirit works in Jews and Gentiles, and many believers are thus found waiting for Christ, as King, to set up His kingdom in power. But the wicked are gathered out from them, by the angels, and cast into the furnace of fire, where there is wailing and gnashing of teeth. We see this again in Matthew 13:49-50. The Son of Man’s kingdom is on earth when He claims it, “Then shall the righteous shine forth as the sun, in the kingdom of their Father.” These are the heavenly saints in glory with Christ, the true Sun of Righteousness. He has received from His Father the kingdom, and will have His heavenly saints in the glory with Him.
In the following parables we have what is precious to the Lord in the kingdom. Not its outward appearance, as man would see it, as in the first three.
Matthew 13:44, “The kingdom of heaven is like unto treasure hid in a field; the which when a man hath found, he hideth, and for joy thereof goeth and selleth all that he hath, and buyeth that field.”
A man has found treasure hid in a field. The man is Christ. It is not the sinner finding Christ. The sinner has nothing to sell. The treasure He finds is not Israel; it is a new treasure to be possessed when Israel was set aside. It is something of great value to Him, for which He parts with all He has, and buys the field for the treasure He knew it contained. How much it cost Him! All His glory as the Messiah was laid aside that He might do the work of redemption. He surrendered all up that He might have us. It is the work of Christ that is spoken of here. The field was not His object, but He bought it. Thus Christ has purchased the world; it is His by purchase, but it is His people in it that are the hidden treasure, that now He can claim as His own. It is really the church as the special object before Him — that which His heart sought in this world, according to the counsels and mind of God. It is they who know and enjoy this full redemption through His work on the cross. What grace to wretched sinners this is! He emptied Himself to redeem us — the people whom He loved — all else was laid aside for this.
(To be continued).

Truths for Young Christians: Worldly Amusements

Chapter 17
WORLDLY AMUSEMENTS
We have seen in a previous paper that the Christian is not to identify himself with the world, as such, religiously or socially, and have had abundant Scripture proofs that all such associations are condemned by God. If such connections then with the world, many of them for good and unselfish objects, are condemned by the Word, it is easy to see there can be no justification of joining with the world merely for one’s own pleasure.
Cricket Clubs
In the present day, when nearly all games and amusements have their special clubs and organizations, a young Christian is often very hardly pushed to join one or another. Some, indeed, may join willingly from various reasons, such as better to enjoy themselves, or perhaps thinking to do others good by a little Christian influence. Instances, however, are exceedingly rare where any good has been effected by this means; too often the result is the other way, and the Christian soon acquires the worldly tastes that characterize his associates. When this is the case, he has only himself to blame for taking the wrong step at the outset, for with the Bible (2 Cor. 6) in our hands, we must characterize as wrong, contrary to, and beneath true Christian walk, any alliances for pleasure with the world.
In this, reluctant as we are to lay down any law, or to make any path narrower than God has made it, we must include all rowing, cricket, football, tennis, bicycle and other clubs. Of course, when boys are at school together, the case is quite different, and the amusements as well as the studies are largely regulated by others. We speak of those who have left school, and are free agents, and we must repeat that for any such to join a worldly club for any purpose of pleasure or amusement is beneath their calling as Christians, and contrary to the Word of God.
Recreation and Exercise Are Profitable
Recreation and exercise are recognized by the Word as profitable for a little (1 Tim. 4:7, R. V.), meaning, we believe, for a short time (that is this life), but these can be taken and enjoyed without joining clubs. No doubt there is not the same scope or the same advantages. An eleven often cannot be made up for cricket, or a full crew for rowing. This is true, but here the child of God must take his stand, and deliberately counting the cost, be prepared to suffer all inconveniences that may arise from his being true to Christ.
An old Christian may not think it much for a young man to refuse one club after another that is pressed upon him, for he, if not too devoted a Christian, is at any rate too old to care for such things. But Christ knows, and Christ will not forget, what it costs at such a time to refuse resolutely for His sake, and His smile and approval is surely well worth the inconveniences that may follow. As the Christian grows in years, however, he begins to feel that to spend hours in mere recreation and amusement is no longer necessary, and he finds that he can combine some variety of work for the Lord with his recreation, so as not absolutely to spend all the time on himself. Long walks can often be combined with profitable visits, and change of scene and air with looking up the scattered saints of God, that seldom get a help; in many ways the believer who seeks to redeem the time, “and who feels that the Lord is at hand,” can and will seek to turn even his hours of recreation to good account.
A Christian in the World
But what shall we say if we look at the other side of the picture, no longer considering those who desire in all things to glorify Christ, but those who, though still His (at least professedly), are worldly in heart, who are seen at concerts, flower shows, cricket matches, and athletic sports, if not actively engaged at least pleased spectators, surrounded with worldly friends, and being for the moment, not only in, but of the world? What a description we get of the world (say at a concert) in Job, “They take the timbrel and harp, and rejoice at the sound of the organ. They spend their days in wealth, and in a moment go down to the grave. Therefore they say unto God, Depart from us; for we desire not the knowledge of Thy ways” (Job 21:12-14)! And can you, dear reader, feel happy in being one of such a company? O, how true it is for a believer when seeking his amusement in such a way, “Even in laughter the heart is sorrowful, and the end of that mirth is heaviness” (Prov. 14:13). The reflections next morning when the Word is opened, and we are alone with God, are not pleasant, and too often the amusements of the evening lead to the neglect of the Bible in the morning. This is repeated until the soul becomes deadened under the round of worldly gaiety and want of spiritual food, while the outward course can only be characterized by, “Lovers of pleasure rather than lovers of God” (2 Tim. 3:4). Surely all would shrink from such a brand.
Satan’s Devices
But Satan is clever, he is subtle; and if we turn from worldly amusements, he will mix them with religion to suit us; and the deluded Christian, following his own will, and not guided by God, substitutes the oratorio for the concert, the bazaar for the flower show, and thinks himself at last consistent. Alas! he has only made matters worse. What can be more blasphemous to the ears of Almighty God, what more offensive to Christ, than to hear the most sublime passages of the Word of God sung by those who openly reject Him? At first sight it certainly does seem incredible that any true Christian could be found willing to listen to the sacred sorrows of His Savior sung by some professional vocalist or “prima donna.” The mixture seems so horrible that surely it is a masterpiece of Satan’s skill to lead Christians to believe that their presence at oratorios is better than at concerts. Bazaars, too, are another specious, though not so repulsive, a form of mixing up what God has forever separated by the grave of Christ — His church, and this present evil world.
The one who is following Christ is found at none of these places. Indeed, to be there would be a pain and no pleasure to him. His happiness is in Christ and in Christ’s people. How, then, can he find it in that world that crucified his Lord. Besides, he is better employed. He does not stand about idle, waiting for Satan to send him off on some errand, for surely it is true that “idlers are the devil’s workmen,” but he is busy in work for his Master, seeking to send to Him a good account of every day that he lives, and to account to Him fully for every talent entrusted to his charge. We trust enough has been said to show that although recreative exercises are perfectly legitimate and needful for the young Christian, all distinctly worldly amusements, clubs, and other associations are not for the one who desires to be true to Christ, and to obey the Word of God.

Hezekiah: Brief Lessons on Church Truth, Part 2

Brief Outline of Lectures on Hezekiah
Read Carefully 2 Chronicles 29.
In the first year of Hezekiah’s reign, He opened the doors of the house of the Lord. No doubt he lighted the lamps, but this is not recorded. However we may have grieved and set aside the Holy Spirit, He is still here. He has not to come again. We have to own Him, in unfeigned dependence. And has not God, rich in mercy, opened again the doors of the house of the Lord? Has He not restored to our souls, in these days, the discovery, that, instead of saying we are Jews, and standing afar off, we have boldness to enter into the holiest by the blood of Jesus? Has He not swept away to faith the rubbish of all human, worldly sanctuaries? May we never forget the all-sufficiency of the Holy Ghost. Again, unhindered, may He ever glorify Christ. O, wondrous grace, thus to restore the long-lost worship in spirit and in truth!
Those words of Hezekiah are very applicable again — “My sons, be not negligent: for the Lord hath chosen you to stand before Him, to serve Him.” Yes, the Lord has chosen a feeble little band; may my reader be one of them. And what was the effect when the doors were open, the rubbish taken out, and the lamps burning brightly? “They brought seven bullocks, and seven rams, and seven lambs, and seven he-goats, for a sin-offering for the kingdom, and for the sanctuary, and for Judah.” All these were killed, and the blood sprinkled upon the altar. Hands were laid on the sin-offering, and reconciliation made for all Israel.
And what has been the case since our God has been pleased to make known the open way into the holiest, in these days, and to gather again His children in the guardian care of the Holy Ghost? Fuller and deeper discoveries of the infinite value of the blood of Jesus! The glories and perfections of His person!
In each of these offerings the number was seven — the one offering of Christ, perfect in every aspect. The blood was sprinkled on the altar, before hands were laid on the sin-offering, in identification. O, my soul, dwell on this — yea, go a little further, for it is the same principle. On the day of atonement see the golden censer, and the sweet incense, beaten small, and the fire from off the altar, and the cloud of incense covering the mercy-seat. That censer had no pattern, its manufacture is not on record. No; in this figure see the uncreated eternal Son — the Holy, Holy, Holy One! as known only to the Father. And why all this first, before the blood is taken in, and sprinkled before God? Does not God solemnly tell us in this, that no less a victim could make reconciliation for sins? Such His wrath against sin, no other propitiation for the sins of men could be made. And, more, nothing could meet the claims of God but that which is equal to God. He who in the beginning was with God, and was God; all things were made by Him, and without Him was not anything made that was made.
Just as the blood was first taken in, and sprinkled before God, and then afterward sins, all transferred to the people’s goat, and borne away, so, in our chapter, the three sevens — the bullocks, the rams, and the lambs — were first killed, and their blood sprinkled upon the altar, thus presented before God, and then hands laid on the sin-offering. What, then, is this distinction? and what its lesson to our souls? Surely that the death of Jesus has first met the infinite claims of God — His righteous, holy claims. It was taken into the holiest, figure of the heaven of heavens. Yes, the blood of Jesus must be shed, or how otherwise could God have dwelt in this sin-defiled universe? Precious words of Jesus, “I have glorified Thee.” O, the glory of the cross! He must needs suffer. “It is finished!” Dwell on this. God is glorified, so glorified by that one offering once, that Jesus, crowned with glory, is seated on the right hand of the Majesty on high. Three times seven fulfilled in this one offering.
Let us be clear about this point first. The resurrection of Jesus, by the glory of the Father, proves that God is perfectly, infinitely glorified — immutably, eternally so. For a moment sin had dishonored God; the death of Jesus has glorified Him through eternal ages. It was not that God only loved us, precious as that is; but He must be glorified, He must be righteous, in justifying us: But if the death of Jesus has met the greater, the first requirement, the infinite claims of the holiness of God: then is it not manifest that He has met the lesser, the sinner’s need? What, then, are those hands laid on the seven goats, killed to make atonement? And, mark well, this was for ALL Israel, not merely Judah, but the revolted tribes of Israel. This is important; the atonement is not merely for those who attain to a certain path of holiness, but for the whole church of God, wherever found for all believers who have passed from death unto life; though many such may be found in revolted tribes of men. Think, then, of the sins of the whole redeemed church of God, transferred to the holy, spotless Victim, who died, the Just for the unjust, to bring us to God! What a sight! What a sin-offering! Did He fail? God is glorified, we ARE sanctified, by that one offering.
I now ask the closest attention to the present place and position of Christ. “But this Man, after He had offered one sacrifice for sins, forever sat down on the right hand of God” (Heb. 10:12). What do we see here? The One who glorified God on the cross, sat clown. The word, forever, means immutable continuity. Nothing can disturb that immutable rest He has, in the unclouded presence of God. But then these very same words are applied to every child of God, in Hebrews 10:14: “For by one offering He hath perfected forever them that are sanctified.” Mark, He hath done it by one offering. HE HATH perfected, in immutable continuity, them that are sanctified. Remember, all the sins of the whole redeemed family of God transferred to Him, borne by Him. And now, as to all charge of sins, perfected in immutability; the Holy Ghost bears witness, “Their sins and iniquities will I remember no more” — no more. O, why should we doubt God? Why say, No, no, this is not true? I must stand and pray for mercy afar off. O, dwell my soul forever on the glory of the cross. Is it not remarkable that God should have restored to our souls in these days the very truths typically set forth in the history of Hezekiah?
But not only do we find the sin-offering, but also the burnt-offering; that is, not only have our sins been transferred to Christ Jesus, and borne by Him beneath the consuming judgment of God, and those sins put away, to be remembered no more, but also we are identified with Him in all the burnt-offering aspect of His death; a sweet savor to God.
And when the burnt-offering began, the song of the Lord began. Then “all the congregation worshiped.” All this continued until the burnt-offering was finished. Yes, there can be no real worship until the Holy Ghost reveals to the soul the immutability of the work of Jesus, and our immutable perfection by that one offering — complete identification with Him, in all the unchanging perfections of His person, for a sweet savor before God. Will this lift up the heart in spiritual pride? “The king, and all that were with him, bowed themselves, and worshiped.” We are thus brought to bow ourselves, and joy in God. “And they sang praises with gladness, and they bowed their heads, and worshiped.” Is it not even so? Instead of standing at a distance, uttering prayers of unbelief, have we not been brought to own again the presence of the Holy Ghost? Has He not brought before us the glory of the person of Christ, and boldness to enter the holiest by His precious blood? Yes, and instead of cries for mercy, has He not put songs of praise in our lips, and worship and gladness in our hearts?
(Continued and to be Continued).
The Coming and Reign of Our Lord Jesus Christ
There are several different terms employed in the Scriptures in reference to the kingdom; we get, the kingdom of heaven; the kingdom of God; the kingdom of the Father; the kingdom of the Son of Man; the kingdom of His dear Son; and the everlasting kingdom of our Lord and Savior Jesus Christ. It is also called the age or world to come.
The kingdom of heaven, or of the heavens, points to the rule of the heavens owned on earth; the term is only found in the Gospel of Matthew. Both John the Baptist and the Lord spoke of it as at hand, not come; that is, the particular aspect of it then presented. During the present interval it takes a secret form not perceived by nature. In Matthew 13 we get the mysteries of the kingdom of heaven, showing the character of things here during His rejection. All who profess to bow to His name are in the kingdom. But when He is manifested in power and glory, He will gather out of His kingdom, by the instrumentality of His angels, all things that offend, and them which do iniquity (Matt. 13:41), and establish it openly with rich and abundant blessing for a thousand years.
John the Baptist being imprisoned, Jesus came preaching, and saying, “The time is fulfilled, and the kingdom of God is at hand” (Mark 1:14-15). But He also said, “But if I cast out devils by the Spirit of God, then the kingdom of God is come unto you” (Matt. 12:28). And again, “when He was demanded of the Pharisees, when the kingdom of God should come, He answered them and said, The kingdom of God cometh not with observation” (margin, with outward show): “neither shall they say, Lo, here! or, lo there! for, behold, the kingdom of God is within you” (margin, among you) (Luke 17:20-21).
The new birth is necessary both to see and enter the kingdom of God in the aspect spoken of in John 3:3-5. All who received Christ by faith were born of God, and so also now (1 John 5:1), and are in the kingdom of God. In another aspect, however, it embraces all who profess the name of Christ: this is what it is outwardly, as seen by men (Luke 13:18-21); the rule or authority of God owned or bowed to. It has a moral significance, as Romans 14:17, “The kingdom of God is not meat and drink; but righteousness, and peace, and joy in the Holy Ghost.” It was connected with the presence of Christ when on earth, but with the presence of the Holy Ghost after He had left this world.
The kingdom of God is likened, in Luke 13:18-21, to similar things as the kingdom of heaven in Matthew 13, and includes the same period, though possibly commencing earlier. Although that which is said of the kingdom of heaven is also said of the kingdom of God, that which is spoken of the latter in many passages could not be said of the former.
At the close of the thousand years’ reign of Christ, He having given up the kingdom to the Father, God is all in all.
The kingdom of the Father designates the heavenly sphere of the kingdom when displayed.
The righteous shall shine forth as the sun in the kingdom of their Father (Matt. 13:43); the sun shines in the heavens.
The kingdom of the Son of Man refers to the earthly sphere of the millennial kingdom; the Son of man ruling from the river to the ends of the earth. (Psa. 72:8; Matt. 13:41).
The kingdom of His dear Son (or the Son of His love) is where the Christian is already brought. Delivered from the power of darkness, he is translated into His kingdom — no longer led captive by Satan in the darkness of nature, but under the rule of the Lord.
The everlasting kingdom of our Lord and Savior Jesus Christ points out the continuance of the kingdom of Christ till the end of all things, in contrast with the overturning of all earthly kingdoms and powers.
In Luke 9:27, the Lord said to His disciples, “I tell you of a truth, there be some standing here, which shall not taste of death, till they see the kingdom of God.” This was fulfilled on the mount of transfiguration, when Moses and Elias, types of the raised and changed saints, appeared in glory, talking with Jesus, the Jewish disciples being on the mountain beholding them. It is a sample of the kingdom in display, when the heavenly saints shall appear with Christ in glory, and Israel shall be at the head of the nations on the earth.
(Continued and to be Continued).

Correspondence: John 14:12; Young Christians

Ques. 74. Please explain John 14:12. Does that include the saints of today? L. M. P.
Ans. The Lord is teaching His disciples that His going away would be gain to them. The Comforter, the Holy Spirit, would come to abide; this would give a higher character to all they did. It would be done by the Spirit, in the power of a glorified Christ. And whatsoever they asked in His name, He would do, that the Father might be glorified in the Son. “If ye shall ask anything in My name, I will do it.” We should ask for that which is for His and the Father’s glory.
It is just as true for us now as for those He spoke to. We need to watch not to ask selfishly. And we ought to be exercised that we should not be so unbelieving.
Ques. 75. Kindly explain in The Young Christian the following passages of Scripture: Matthew 10:34; John 3:20-21; 1 Peter 2:13-14. E. B.
Ans. Matthew 10:34. This verse and its context show that the Lord Jesus is rejected and that man’s heart is enmity against God. (Rom. 8:7). Our dearest friends on earth, if not converted, may be our bitterest enemies, because we love and desire to serve the Lord Jesus. If we are true to Him, the world will hate us (John 15:18-16:2; 1 John 3:13). As long as Christ is the rejected One, His followers will need to endure opposition from the world. (Read the whole passage).
John 3:20-21. the sinful nature of man hates the light of God’s presence, for it condemns his evil ways; he therefore keeps away from the light. But he that practices the truth, comes to the light; this is a man born of God. (John 3:5). He delights to be in God’s presence, for all his sins have been judged and are put away by God; he is hiding nothing: his deeds are manifest as wrought in God. Christians are children of light (Eph. 5:8), and walk in the light (1 John 1:7; Eph. 2:8-9).
1 Peter 2:13-14. These verses speak of subjection to the powers that be — the laws of the land. (Rom. 13:1-7). These are called ordinances, and are to be respected, except where they interfere with our duty to God, then we must “obey God rather than men”. Thus Peter and the apostles answered when they were commanded not to teach in the name of Jesus. (Acts 5:28-29).
A different kind of ordinances is mentioned in Colossians 2:20-23. These we are not to be subject to — restrictions, such as, “Touch not, taste not, handle not”, what we are to eat, or drink, or use; religious ordinances for the improvement of the flesh; pledges or vows. All these we are free from, because we have “died with Christ from the elements of the world”, and, now, “risen with” Him, we are to be occupied with things above (Col. 3:1).

Inspiration of the Scriptures: The Importance of Bowing to the Authority of Scripture

The Importance of Bowing to the Authority of Scripture
Nothing can exceed the importance that our Lord and the servants of God in all ages have given to Scripture. Our Lord exposed the infidelity of the Sadducees as to resurrection, by saying to them, “Ye do err, not knowing the Scriptures, nor the power of God,” and by a quotation from Exodus 3:6. He also put the Pharisees to silence as to His person, by quoting Psalm 110:1. We are told that the disciples were perplexed, because “they knew not the scripture that he must rise again from the dead.” The Lord also sharply rebuked the two unhappy ones walking to Emmaus, by saying unto them, “O fools, and slow of heart to believe all that the prophets have spoken! ought not Christ to have suffered these things, and to enter into His glory? And beginning at Moses and all the prophets, He expounded unto them in all the scriptures the things concerning Himself.” What an exposition of the Old Testament this must have been! Is it astonishing that they said, “Did not our heart burn within us, while He talked with us by the way, and while He opened to us the Scriptures?”
The Corinthian believers were sadly wrong, because they knew not the scripture as to the resurrection of the body. The Galatian believers went wrong, because they were adding “circumcision” to the work of Christ, in order to make their salvation more secure; and the apostle, by the Spirit, uses Scripture to set them right. The Colossian saints were not quite free as to ritualism and rationalism, because they got away from Scripture, and therefore were “not holding the Head” — not in communion with the Lord Jesus Christ as Head of the body, the assembly. The Thessalonian believers were not clear on some points as to the Lord’s coming, and were therefore sorrowful, which they would not have been had they known the truth as to the resurrection of the bodies of those who are Christ’s, the change of the living, and the rapture to meet the Lord in the air.
Some of the Hebrew saints were in danger of giving up Christianity, and going back to Judaism, because they knew not the truth which the Scriptures teach as to the eternal efficacy of the one offering of Christ once for all, and the liberty to enter into the holiest with boldness by the blood of Jesus since the veil was rent, and Jesus appears in the presence of God for us as our great High Priest. Could anything more be required than the consideration of these facts, to induce us to read and meditate on the Word of God for soul-blessing, and guidance as to life and walk?
Thank God, the Holy Ghost dwells in us, is able to teach us and guide us into all the truth, so that an apostle could say of believers, “Ye have an unction from the Holy One, and ye know all things.”
(Continued)

Sought and Saved

A young man who was brought to the knowledge of Christ as the Savior of his soul, and greatly enjoyed the peace and blessing which had become his portion through believing, soon after his conversion felt great concern for a person with whom he had been acquainted. He was well aware that this person had an objection even to enter a place where the gospel of the grace of God was proclaimed. Still, having a love for his soul, he called upon him, and told him of his own conversion, and of the blessedness of the salvation of which he was a partaker. His friend listened attentively, but smiled contemptuously at what was related. The young man, however, was not deterred by this, but continued to call upon him from time to time, urging him to accompany him to a certain place where the gospel was preached; and after repeated refusals and excuses, he at length consented. The young man had prayed to the Lord to bless the preaching of His Word to the poor sinner, and waited to see what He in His grace might do. On leaving the place after the preaching, he observed that his companion was reserved and serious in his manner, so he did not intrude too closely into his thoughts. But on calling upon him a few days afterward, he found him more communicative. He also expressed a desire to go to the preaching again, and the young man was, of course, willing to bear him company. The fact was, as he afterward acknowledged, he was converted on the first night, but was so searched by the Word of God, that he was all in a turmoil, and could not converse with his friend. Great was the joy of the young man to find that God had blessed the preaching of His Word to his soul.
O! may we, who know the salvation of God in our own souls, be at all times ready to be guides of any poor, sin-stricken souls to Himself. May our prayers and actions be in the spirit of the Apostle Paul, who said, “Brethren, my heart’s desire and prayer to God for Israel is, that they might be saved” (Rom. 10:1).

Richer Than the Rich

“Thanks be unto God for His unspeakable gift” (2 Cor. 9:15). “As having nothing, and yet possessing all things” (2 Cor. 6:10).
Having Jesus, all is mine-
Gift unspeakable, divine!
Rich am I beyond all care,
Richer than a millionaire.
Poor is he whose wealth’s his all-
Poor when ‘neath the funeral pall:
Not a great can he take o’er-
O how stripped when here no more!
Dark and dismal must it be
Thus to face eternity:
Naught to calm the dread-struck heart,
Naught to “cool” a little part.
Take “true riches” while you may
Come to Jesus, don’t delay;
He is rich beyond degree,
Take salvation full and free.
Then confess that Holy One;
Ever tell what He has clone;
Live to “Him who died and rose,”
Till thy pilgrim days shall close.
Soon the blissful day will come-
He will call His ransomed home:
Thou wilt be, when welcomed there
Richer than a billionaire.
“He that overcometh shall inherit all things” (Rev. 21:7).

Scripture Study: Matthew 13, Part 4

Matthew 13:45-58
Matthew 13:45. “Again, the kingdom of heaven is like unto a merchantman, seeking goodly pearls: who, when he found one pearl of great price, went and sold all that he had, and bought it.” Here we get Christ again; this time as a merchantman seeking goodly pearls. His discernment and taste for what is beautiful, is gratified in this one pearl.
Here He has found what gratifies His heart, and He is willing (it is of such attraction to Him) to sell all to possess it. He can estimate its value, He buys IT alone.
What a wonderful thing for the Father to show Him His bride, in His purposes, away back in eternity, before anything was created. And then to see this unchanging love carrying it all out. Thus “Christ loved the church, and gave Himself for it; that He might sanctify and cleanse it with the washing of water by the Word, that He might present it to Himself a glorious church, not having spot, or wrinkle, or any such thing; but that it should be holy and without blemish, (Eph. 5:25-27). In His eyes it is a pearl of great price; a thing for His affections to center upon. Of beauty and value, He found it„ and looking upon it with delight and love, went and sold all that He had to have it. And what joy it will be to Him when He claims it to His heart, as His bride, adorned for her husband. His heavenly bride, the object of His affections, the companion of His glory. What blessedness it brings to us, to think He loves us so much. What a contrast with Israel’s blessing. He will reign over them; we shall reign with Him.
Matthew 13:47. In the next parable we see that the fishermen had cast a net into the sea. All the fish in the sea did not come into the net, but it gathered of every kind; and when it was full, they drew to shore and sat down and gathered the good into vessels, but cast the bad away. Here the spiritual mind is to see that God is gathering out of this world; it is not improving the bad, but it is gathering together the good into companies, leaving the bad on the shore. It is what the spiritual mind sees the Lord occupied with, and therefore what true, intelligent, spiritual, service will be doing. There will be judgment upon the bad, but it is not the fishermen, but the angels who carry that out.
Matthew 13:49. leads us to that. So shall it be at the end of the age: the angels shall come forth and sever the wicked from among the just. The wicked are taken away; the just are left, as in Matthew 13:41. These wicked are cast into the furnace of fire, and again the blessed Lord repeats it. “There shall be wailing and gnashing of teeth,” as if to warn precious souls to flee now to Christ the Savior. “The just” on earth at this time are those converted during the interval between the Lord’s coming for His saints, and His coming, as Son of Man, with His saints, to take the kingdom by power. They were faithful to Him during the tribulation, and endured to the end, and so are brought into the reign of Christ, but are on earth; not like us, for we will be with Him in heaven: we shall reign with Him.
Matthew 13:51-52. These parables present the kingdom expected by the Jews as taught in prophecy, but in such a new way, that every scribe instructed in these truths, is like a householder that brings forth of his treasure, things new and old.
Matthew 13:53-58. The Lord then leaves His teaching with them, and departs, and coming to His own country, He resumes His teaching, but they see Him only as the son of Joseph, and refuse Him as a prophet, as before they had rejected Him as King. Their unbelief hinders them from receiving the blessing He had for them. Such is man: such were we by nature.
Faith, too, that trusts the blood, through grace,
From that same love we gain;
Else, sweetly as it suits our case,
The gift had been in vain.

Truths for Young Christians: An Appeal to Christian Young Men

Chapter 18
An Appeal to Christian Young Men
“I find it very difficult to lead a Christian life,” said a young man to me lately.
He had been converted, and had, to a certain extent confessed that blessed fact to many; but, yet, like multitudes in similar circumstances, he found the Christian life difficult, and why? Why should a life of practical godliness be difficult? Why do we find many, especially among the young Christian men of the day, oppressed by what they call the difficulties of the way?
They believe; they accept the Word of God; they refuse to be caught by the infidelity that surrounds them; they desire to live for Christ; they thankfully own the value of His blood; and admit, in measure, His claims over them as their risen Lord; and yet with all that, they complain of difficulties, and of their inability to live up to the standard. Now, where does the fault lie? Granted that salvation is known and enjoyed; that eternal life is consciously possessed, and that the experience is that of Rom. 8, as contrasted with Rom. 7; that is, that the soul has acknowledged that “in me (that is, in my flesh), dwelleth no good thing;” and that, on the other hand, it can truly say, “There is therefore now no condemnation to them which are in Christ Jesus. * * * For the law of the Spirit of life in Christ Jesus hath made me free from the law of sin and death. For what the law could not do, in that it was weak through the flesh, God sending His own Son in the likeness of sinful flesh, and for sin, condemned sin in the flesh: That the righteousness of the law might be fulfilled in us, who walk not after the flesh, but after the Spirit,” if it can really, through grace, rejoice in this liberty, then how comes it still to find difficulty? Clearly, that difficulty is not one that arises from ignorance of doctrine.
The soul in this case does not require truth in order to deliver or set it free from sin. That deliverance is known. No, the hitch does not lie there. A clear gospel has been heard and accredited; and the soul, so far as mere knowledge is concerned, is right enough. Lack of scriptural information is not the point. Thank God for a firm grasp of a gospel that delivers the soul from the entire question of sin, and that places it before Himself in conscience acceptance! But this does not touch the difficulty in question.
Truth is moral as well as intellectual, and seeing that we are made of heart as much as of head — the heart having far more influence over us than the brain — it is necessary to own this effect of the Word on our ways. The heart is king, and the other members are its vassals. The will is liege-lord, and ever claims the throne.
Now, young men, what you want is a heart — a heart devoted to Christ.’ O, how difficulties vanish before the omnipotence of a devoted heart! It is half-heartedness that is the cause of all your trouble. You are ashamed of Christ! Your confession of His name is sadly restricted! You often play the coward! Your flag is not nailed to the mast!
O, what liberty of soul is found in making a final committal of yourself, and taking a stand of uncompromising decision for the Lord! Reservation is fatal. To hold Christ in one hand, and the world in the other, is misery to yourself, and dishonor to Him. A divided heart means chronic defeat.
You plead circumstances. Well, proper circumstances are no hindrance, but if your line of life is improper; that is, unscriptural, leave it. Better be poor with Christ, than rich without Him, or only a part of Him. God will never fail the man who follows the Lord. Christ wants followers! True service flows from true following. If you do not follow, you cannot serve, and in order to follow, there must be heart-affection.
“Difficulties” is not the word. Say they are impossibilities, and then you will know in whose strength to face them.
The path of faith is not difficult — it is impossible!
To cross the Red Sea was impossible, yet faith marched through dry shod. To traverse the desert was impossible, yet faith was fed with the food of angels, and refreshed by water from the rock. To pass the Jordan was impossible, yet again faith walked through the flood and placed her foot on the shores of promise.
If a thing be only difficult, we will face it in our own strength and perhaps be beaten. If it be impossible, we repudiate all confidence in ourselves, and go forward in the grace and power of the living God, and the mighty giant falls by a stone from the sling. “Grasshoppers?” Yes, but God can use “a worm to thresh a mountain,” and a Paul to establish a church.
Courage, dear young fellow-Christian! Throw your heart unreservedly into the interests of Christ, and speak no more of difficulties. Love knows none!
Three years after I was converted, I went on my knees and asked the Lord the meaning of, “That they which live should not henceforth live unto themselves, but unto Him which died for them and rose again” (2 Cor. 5:15).
And the result? Well, I saw that it had been self hitherto, but it was to be Christ henceforth. Ah, the sweetness of His love who died for me filled my soul, and now His claims as risen Lord asserted themselves worthily over my affections. A new vista opened, and a new life lay before me.
You will find it in full power if you read the Epistle to the Philippians:
Philippians 1, Christ is the Life.
Philippians 2, Christ is the Pattern.
Philippians 3, Christ is the Object.
Philippians 4, Christ is the Power.
Thus we find Christ the Motive, Power, and Object of the Christian life. What more can be needed?
There is a new motive — not self, but Christ: a new power — not natural energy, but Christ; and a new, blessed and worthy object to command my entire life — not my poor, ignoble interests, but a living, loving, glorified Savior.
One thing I do,” said the Apostle Paul, “I press toward the mark for the prize of the high calling of God in Christ Jesus.”
Did Paul sit at the foot of “Hill Difficulty,” and shed effeminate tears of faithless irresolution? Did he shrink from the lions that terrified “Timorous and Mistrust”? Far from it! No, man of like passions though he was, his heart was so intense, and his pursuit so keen, that none of these things moved him, neither did he count his life dear unto him, so that he might finish his course with joy. Happy man! Bright, victorious witness!
I am persuaded, dear young fellow-Christian, that these wretched difficulties are but phantoms of half-hearted indecision, and that they would vanish like vapor, if you had but the moral courage to put down your foot in bold, gracious refusal of all that is of the world.
The first injunction in the book of Proverbs is, that you say, “No” to evil. (See Prov. 1:10). O, the immense moral power of that virtuous little adverb. Moses knew its value when he “refused to be called the son of Pharaoh’s daughter”; and our life, in order to be complete and full orbed, must be negative as well as positive.
Each coin has it reverse and its obverse. Each true Christian must assert his “No” as well as his “Yes” — “No” to sin and the world; “Yes,” by grace, to Christ and His ways.
Go and bathe your heart, dear friend, in the ocean-fullness of His love. Let it be suffused by and satisfied with the store of His grace, and then seek that henceforth may be marked by one bright idea — the glory of Him who died for you, and rose again.

Hezekiah: Brief Lessons on Church Truth, Part 3

A Brief Outline of Lectures on Hezekiah
Read carefully 2 Chronicles 29.
“Then Hezekiah said, Now ye have consecrated yourselves unto the Lord, come near.” What is consecration? The margin reads, “Filled your hand.” Yes, if the Holy Ghost reveals the immutable person and work of Christ, the effect is, we are filled with Christ; and that is consecration. If this is not so, we shall be taken up with men, and things, and so-called churches of men. But if the doors are open, the lamps lighted, Christ revealed, He, He will engross every thought and desire. O, to live on account of Christ, as He lived on account of the Father! And if we are thus consecrated to Him, thus filled with Him, then the consecrated things will be in abundance. Six hundred oxen, and three thousand sheep. Nay, we are not our own. Burnt-offerings in abundance, with the fat of peace-offerings, and drink-offerings! What untold delight of heart, filled with all the fullness of Christ! “So the service of the house of the Lord was set in order.” When Christ by the Spirit has His true place, then the house is in order. All else that man calls order is simply the house in disorder.
“And Hezekiah rejoiced, and all the people, that God had prepared the people: for the thing was done suddenly.” What God did then in Judah, has He not done now in Christendom? Is it the hand of God, or another sect? Has God, during the last fifty years, opened the doors, and revealed the way into the holiest? The believer, by the one offering of Christ, perfected, as to the conscience, in immutable continuity? — and all this in direct contrast with the Judaism into which Christendom had sunk. Has not God, by the Spirit, awakened His people in all parts of the world — in Java, in Russia, throughout Europe, America, Australia? The thing is of God, and done suddenly. Souls are being brought from the dark regions of unbelief to the unclouded presence of God, with joy and gladness. Is there a doubt that this work is of God? To Him be all praise! O, what grace and mercy to us in these last days!
The Passover
2 Chronicles 30
We shall now find some very important church truth typified in this chapter. The order is very striking. We have had the doors open — the full gospel of God; the grace that brings the prodigal right into the presence of the Father. Then, the lamps giving their full, perfect light; the Holy Ghost taking of the things of Christ, and showing them unto us; the infinite value of His one sacrifice; our immutable perfection by that offering, accepted in all the sweet savor of the burnt-offering, the Lamb of God — the joy, gladness, and worship.
Now the invitation goes forth to come to the house of the Lord at Jerusalem, to keep the Passover unto the Lord God of Israel. And in wisdom and counsel of the king, the princes and all the congregation, this must be, not in the appointed time, the fourteenth day of the first month, but in the second month. For they could not keep it at that time, because the priests had not sanctified themselves sufficiently, neither had the people gathered themselves together at Jerusalem. (2 Chron. 30:3).
If we read carefully Luke 22:1-20, we see how the Lord’s supper took the place of the Passover. “With desire have I desired to eat this passover with you before I suffer.” The last Passover before He suffered: Himself the fulfillment of it. He then presented Himself to faith, no longer the body of the lamb, or the cup of the Passover. The Passover looked forward — the Lord’s supper is a remembrance. “This do in remembrance of Me.” The blessed Lord Himself, our Passover, is slain. It is no longer the wine of the Passover, but the cup, in remembrance of His blood shed for us. But they were to come to Jerusalem, to the house of the Lord. Is there any such house of the Lord now? any place of worship, or earthly sanctuary? No, there is no such place now. All this belonged to the first covenant, or to Judaism. “Then, verily, the first covenant had also ordinances of divine service, and a worldly sanctuary” (Heb. 9:1). What, then, have we now, if all the system of worldly sanctuaries, called the house of God — places of worship with divine service; all this is simply Judaism; not a vestige of which have we found in the New Testament. What have we, as the true center of gathering? Have we not Christ Himself? Jesus says, “For where two or three are gathered together in My name, there am I in the midst of them” (Matt. 18:20). I know that in millennial days to come, Jerusalem shall be the city of the Great King. But let us remember that at present He, as their King, is cut off, and has nothing; and that now the only place of gathering is to Him, the rejected One.
But why did they keep the Passover on the fourteenth day of the second month?
If we turn to Numbers 9:1-12, we shall find a very distinct reason given there. The keeping of the Passover in the first month is confirmed: but there were some men who were defiled by the dead body of a man, that they could not keep the Passover on that day. The question was brought before the Lord; and the answer of the Lord was, that if any were unclean by reason of a dead body, they should keep the Passover on the fourteenth day of the second month. Thus Hezekiah acknowledged the defiled state of Israel. Is it not so with the church of God? Is it in its first condition, or second? Has it become defiled by the dead body of the world? O, does it not become us thus to own the sad, defiled, ruined state of the church as a testimony for Christ?
Now we come to a very important point. The invitation and responsibility to keep the passover was as extensive as the atonement. The sin-offering, the reconciliation, was made for ALL Israel, ALL Israel are invited, and responsible, to come and keep the Passover. So they “make proclamation throughout all Israel, from Beersheba even unto Dan, that they should come to keep the Passover unto the Lord God of Israel, at Jerusalem; for they had not done it of a long time in such sort as it was written.” This deeply important principle as to the Lord’s table, also, is little understood. The analogy is striking; souls everywhere, who have been led to own the Holy Ghost in the assembly, like the lamps in the sanctuary, have been also led next to remember the Lord’s death, in the breaking of bread. Nay, more, they have learned from holy Scripture that the church had not done it of a long time, in such sort as it was written. I am not aware of an instance for seventeen centuries, where Christians broke bread as it was written; until within about the last fifty years. There was always something omitted, or added, to what was written. I need not dwell on the Mass. But what were we doing? Take one thing added, which we all thought right — a minister administering the sacrament. Was this written in the inspired Word? Where? Disciples came together to break bread, or “we come together” (Acts 20). In 1 Corinthians 10; 11, is there a thought of such a person at the Lord’s table? This it not a question of the gifts of Christ: the evangelist to preach the gospel, and the teacher to teach the church of God. But at the Lord’s table, “The cup of blessing which we bless, is it not the communion of the blood of Christ? The bread which we break, is it not the communion of the body of Christ? For we, being many, are one bread, and one body; for we are all partakers of that one bread.” Place a man unscripturally to administer, and the communion is lost sight of! It is the expression, essentially, of equal co-partnership in that blood, and in that one body. Blessed fact, every redeemed sinner has equal partnership in the reconciliation; every washed soul whiter than snow! Fellowship one with another, and the blood of Jesus cleanseth us from all sin.
Where it is service, then all are not alike. Just as in a firm — one servant may have a pound per week, whilst a foreman may have three. But copartners are all equally alike. In the equal co-partnership, fellowship, communion of His blood, communion of His body. There are no servants, and no foreman, in Scripture to administer the sacrament. No, for a long time the Lord’s supper had not been kept as it was written. O, the grace, the love our Father has, to have restored it in these last days, before the coming of the Lord. But have we understood the heart of Christ in this? The invitation, and the responsibility to gather to His name, to break bread as it is written? Is not the invitation to do so as wide as the atonement? The responsibility reaches every reconciled child of God on earth. Not only Judah owning allegiance to Hezekiah, but to every Israelite in the revolted tribes. Not only those gathered, and owning allegiance, to Christ, but every redeemed soul in every revolted sect on earth, from Beersheba even unto Dan. As there was the perfect substitution offered, the seven goats FOR ALL ISRAEL: we must understand here Israel as a type of the whole redeemed church of God, whatever their outward position. And this is a great truth, that every believer, passed from death unto life, stands in all the immutable perfection of Christ, accepted in the Beloved — sins and iniquities to be remembered no more, no more! Is not the Lord’s table, as it is written, the true place for every one of them? Yes, the basis of gathering to Christ to break bread, our Passover, is as wide as the atonement. Does not Jesus say to every reconciled soul, “Do this in remembrance of Me”? And would He not have each one enjoy communion with Himself? Other questions will arise, but the basis of true fellowship is seen to be equal to the extent of the atonement.
So the posts passed from city to city, but they laughed them to scorn, and mocked them. O, when did not men despise the message and the messengers of the Lord? It was so in the days of Noah, of Lot: yea, when the Son of God walked in the midst of men. And even so now, the present work of God in love is despised of men. Nevertheless, divers humbled themselves and came. Also in Judah, the hand of God was to give them one heart to do the commandment of the king, and of the princes, by the Word of the Lord. And is not the hand of the Lord seen now gathering souls to Christ, and giving them one heart? If it is not this, it is only another sect in self-will. Yes, it is the hand of the Lord. Reader, are you asleep, or awake; can you discern the present heart and hand of the Lord? If so, it is a little thing to be laughed to scorn.
The next point in order is this: when the Lord had thus gathered a company to keep the Passover, “They arose, and took away the altars that were in Jerusalem, and all the altars for incense took they away, and cast them into the brook Kidron” (2 Chron. 30:12-14). Thus, where the Lord has now gathered a company to break bread, as it is written, the first thing is to put away from themselves everything inconsistent with the holiness of His presence. Just as when the doors were opened, the cleansing away of the rubbish began in the very holiest.
The holiest we enter in perfect peace with God. There, surely, holiness becomes that holy place. This is a solemn point to all whom the hand of the Lord doth gather, to keep the Lord’s supper, as it is written.
(Continued and to be Continued)

The Coming and Reign of Our Lord Jesus Christ: The Age to Come, or the Millenium, Part 2

To turn back now to Psalm 2, we find the gathering together of the kings and rulers of the earth against the Lord, and against His anointed; “Yet,” saith the Lord, “have I set My King upon My holy hill of Zion.” Judgment, as we have seen in our last paper, falls upon the powers of this world. Christ takes the kingdom, and the remnant of Judah are delivered; for “it shall come to pass, that in all the land, saith the Lord, two parts therein shall be cut off, and die; but the third shall be left therein. And I will bring the third part through the fire, and will refine them as silver is refined, and will try them as gold is tried they shall call on My name, and I will hear them: I will say, It is my people: and they shall say, The Lord is my God” (Zech. 13:8-9).
Following upon this we get the restoration of the ten tribes of Israel, now scattered and lost among the nations. The fullness of the Gentiles being come in, the Lord begins now to bless in a special way, according to His many promises, His earthly people Israel. “The gifts and calling of God are without repentance” (Rom. 11:29).
Many deny the truth of the restoration and blessing of God’s ancient people, and spiritualize and explain away the numerous scriptures which refer to it, applying them often to the present blessing of the church of God. Such would do well to ponder Romans 11:25-36. “I would not, brethren,” writes the apostle to God’s saints at Rome, “that ye should be ignorant of this mystery, lest ye should be wise in your own conceits; that blindness in part is happened to Israel, until the fullness of the Gentiles be come in. And so all Israel shall be saved: as it is written, There shall come out of Sion the Deliverer, and shall turn away ungodliness from Jacob”. For “thus saith the Lord; If heaven above can be measured, and the foundations of the earth searched out beneath, I will also cast off all the seed of Israel, for all that they have done, saith the Lord” (Jer. 31:37).
Again; “He shall send His angels with a great sound of a trumpet; and they shall gather together His elect (Israel) from the four winds, from one end of heaven to the other” (Matt. 24:31).
Great numbers of Jews having returned to their land by the aid of a certain maritime power (Isa. 18), come into great tribulation; one-third who witness for Christ in this time of Jacob’s trouble (Zech. 13:9). are saved out of it; others are martyred; the rest judged; then the ten tribes are gathered back (the rebellious being purged out) and united with the two, as one nation, under Christ.
Ezekiel, in Ezekiel 20:34, speaks thus of this event: “And I will bring you out from the people, and will gather you out of the countries wherein ye are scattered, with a mighty hand, and with a stretched-out arm, and with fury poured out. And I will bring you into the wilderness of the people, and there will I plead with you face to face . ... And I will purge out from among you the rebels, and them that transgress against Me: I will bring them forth out of the country where they sojourn, and they shall not enter into the land of Israel; and ye shall know that I am the Lord.”
Jeremiah, in Jeremish 31:8-10, says: “Behold, I will bring them from the north country, and gather them from the coasts of the earth, and with them the blind and the lame, the woman with child and her that travaileth with child together: a great company shall return thither  ... . Hear the Word of the Lord, O ye nations, and declare it in the isles afar off, and say, He that scattered Israel will gather him, and keep him, as a shepherd does his flock.”
Isaiah 49:20-23 is a most touching description of the union of the tribes after the Redeemer has come out of Zion. “The children which thou shalt have, after thou hast lost the other, shall say again in thine ears, The place is too strait for me; give place to me that I may dwell. Then shalt thou say in thine heart, Who hath begotten me these, seeing I have lost my children, and am desolate, a captive, and removing to and fro? and who hath brought up these? Behold, I was left alone; these, where had they been? Thus saith the Lord God, Behold, I will lift up Mine hand to the Gentiles, and set up My standard to the people; and they shall bring thy sons in their arms, and thy daughters shall be carried upon their shoulders. And kings shall be thy nursing fathers, and their queens thy nursing mothers”.
Then again, in the well-known passage, Ezekiel 37, the vision of the valley of dry bones, we are distinctly told, in Ezekiel 37:11, “these bones are the whole house of Israel.” They are viewed in the vision as in their graves — the present state of the tribes scattered and lost among the nations; but the Lord God says, “I will open your graves, and cause you to come up out of your graves, and bring you into the land of Israel,” (Ezek. 37:12-14).
“Behold, I will take the children of Israel from among the heathen, whither they be gone, and will gather them on every side, and bring them into their own land: and I will make them one nation in the land upon the mountains of Israel; and one king shall be king to them all: and they shall be no more two nations, neither shall they be divided into two kingdoms any more at all  ... . And David My servant shall be king over them” (Ezek. 37:21-24).
I might cite many more scriptures; but these will suffice to show simply the fact of that which God will shortly do for His earthly people. The prophets speak so plainly in the above passages, that comment upon them is almost superfluous, as I do not aim at giving the reader much more than an outline sketch.
The Deliverer being come out of Zion, Israel saved, “they shall teach no more every man his neighbor, and every man his brother, saying, Know the Lord: for they shall all know Me, from the least of them unto the greatest of them, saith the Lord for I will forgive their iniquity, and I will remember their sin no more” (Jer. 31:34).
“Lift up your heads, eternal gates,
Transcendent dawn glows o’er ye!
At Salem’s door Messiah waits;
He is the King of glory.”
(Continued).

Correspondence: Fifty Days; Mark 4:26-29; Passion Play; John 14:12

Ques. 76. Please explain how you find fifty days between the resurrection of the Lord and the coming of the Holy Spirit? W. H. H.
Ans. The word “Pentecost” means fifty, or the fiftieth; the allusion is to Leviticus 23. In that chapter we get two offerings on the morrow after the sabbath. The wave sheaf is the type of the resurrection of Christ (Lev. 23:10-14). There is no sin offering attached to that; it is the Lord Himself. Then seven Sabbaths were counted, and on the morrow after the seventh Sabbath, that is, the first day of the week, a new meat offering — “two wave loaves baken with leaven” — is offered. A sin offering accompanies it, for it is typical of the redeemed church, which, though redeemed, has sin in each member (Lev. 23:15-21).
In the New Testament we see that the Lord rose on the first day of the week, and was seen of His disciples forty days (Acts 1:3); then came His ascension. The disciples continued in prayer and supplication the rest of the period (not many days), till Pentecost, the fiftieth day, was fully come; then the Holy Spirit came down.
Ques. 77. What does Mark 4:26-29 mean? W. H. H.
Ans. Mark 4:26-29, compares the kingdom of God “unto a man that casts seed into the ground” who rising and sleeping day and night, allows it to increase without taking any notice of it. The earth produces thus fruit of itself, first the blade, then the ear, and then the full grain in the ear. Now when the fruit is ripe, the sickle is put in at once, because the harvest is come. Thus the Lord worked personally, sowing the Word of God upon earth; and at the end, He will return, and work again in person, when the time for the judgment of this world shall have come, but now, in the meantime, He remains seated at the right hand of God, as though He did not occupy Himself with His field, although in secret He does work by His grace, and produces everything. But it is not manifest. Without being seen, He works to make the seed grow in a divine way, by His grace, whilst apparently He allows the gospel to grow, without having anything to do with it, until the harvest. Then He will appear and will Himself work openly.” (J. N. D. Collected Writings)
Ques. 78. What do you think of the Passion Play? W. H. H.
Ans. To make the Lord Jesus and His finished work the subject of an entertainment is sacrilegious. Think how horrible it must be in the eyes of God, the Father, that men should dare to personify His beloved Son in the hour of His deep suffering as an atoning sacrifice!
Ques. 79. John 14:12. To whom does the Lord refer? Is it before the cross that those who believe shall do these works, or does it take in believers in this present time? What are the works referred to? J. E. K.
Ans. The Lord is speaking of what is true of believers when He would be with the Father. The Holy Spirit would then dwell in them, and what they did would be done in the power of the Spirit. The works would include such as Peter (Acts 5:12; 15:16), and Paul (Acts 19:11-12) did; but also to see Christ — “the life of Jesus” (2 Cor. 4:10-11) — produced in redeemed men, once under the power of sin, is more wonderful than to see the moral glory in all its perfection in Him who was ever and always altogether perfect. What care we should take to walk in the power of the Spirit ungrieved. (Eph. 4:30).

Inspiration of the Scriptures: Alleged Inaccuracies of the New Testament

Alleged Inaccuracies of the New Testament
Before leaving our consideration of the New Testament, it may be well to look at some more of the alleged inaccuracies with which infidels and professing Christians have unscrupulously assailed the holy Scriptures.
Three Days’ and Three Nights
It has been said, that because our Lord died on Friday at the ninth hour, was buried that day, and rose again from the dead on Sunday, the first day of the week, that, therefore, it is not true to say He was “three days and three nights in the heart of the earth,” as our Lord declared that the Son of Man would be (Matt. 12:40). But let it be observed, that according to the Jewish mode of calculating time, there is no inaccuracy in this statement. In our day, man has changed the scriptural order of almost everything possible, and instead of speaking of an evening and morning being a day, modern Gentiles say that a day consists of a morning and evening, and so calculate accordingly. And if the Jews reckoned any part of a day, as part of a night and a day (for the evening and the morning in Gen. 1, made a day), the alleged inaccuracy wholly disappears. For Friday, up to six o’clock, would be spoken of as one night and day, Friday evening and Saturday morning another night and day, and thus the difficulty is removed.
Our Lord’s Temptations
It has been alleged, that because Matthew and Luke, who both recorded in their gospels our Lord’s temptations from the devil, do not narrate them in the same order, there must be inaccuracy. But suppose one of the Evangelists, Matthew, relates them in their chronological order, and Luke puts them in moral sequence, or according to the severity of the temptations, and therefore puts the severest, when Satan quotes Scripture, last, where then is the difficulty?
Drink No Longer Water
It has been said by learned men, Can we believe that to be inspired which tells Timothy to “drink no longer water, but use a little wine for thy stomach’s sake, and thine often infirmities”? And again, “The cloak that I left at Troas with Carpus, when thou comest bring with thee, and the books, but especially the parchments”? Why not? Does it not show that the Holy Spirit considers our bodily need, and the books and writings His servants find helpful in His blessed service?
Feeding the Multitudes
One of the boldest attacks on the truth of late years has been the statement that when our Lord fed the thousands on a few loaves and fishes, with baskets of fragments remaining as not needed, it is impossible they could have all eaten, and must have had a meal before. But such infidel statements completely leave out God, and it may be said to those who so speak, “Ye do err, not knowing the Scriptures, nor the power of God.” As to the miracle of feeding so many on so little, we are told “they were all filled,” and on referring to Psalm 132:15, we see it was only the fulfillment of what had been predicted ages before, as a true sign of Messiah, “I will abundantly bless her provision; I will satisfy her poor with bread.” Could there have been a more beautiful fulfillment of this prophecy than our Lord’s feeding the multitude? And ought not the Jews to have known by it that He was the Messiah?
(Continued)

An Anecdote of John Knox

The pure, heart searching doctrines that were preached by this Scotch Reformer, were very offensive to the carnal mind, and particularly obnoxious to the corrupt court of Queen Mary; hence he was commanded to stop preaching; but, owning no master, and obeying no mandate, which was in opposition to God and the Bible, he paid no attention to the command of the palace. The haughty Mary, hearing, from the enemies of the cross, that her orders were disobeyed, summoned Knox into her presence. When he arrived, he was ushered into a room in which were the Queen and her attendant lords. On being questioned as to his persistency, he answered plainly that he preached the truth, and dared not preach less.
“But,” exclaimed one of the lords, “our commands must be obeyed on penalty of death; silence, or the gallows is the alternative.”
The spirit of the Reformer was roused by the insinuation that human punishment could make him desert the cause of Christ, and he firmly replied, “My lords, you are mistaken if you think you can, by threats, intimidate me to do what conscience and God tell me not to do; for be it known to you that it is a matter of no importance to me, when I have finished my work, whether my bones shall bleach in the winds of heaven, or rot in the bosom of the earth.”
Knox having retired, one of the lords said to the Queen, “We may let him alone, for we cannot punish that man.”
Well, therefore, might it be said at the grave of John Knox, “Here lies one who never feared the face of man.”
“If ye be reproached for the name of Christ, happy are ye; for the Spirit of glory and of God resteth upon you” (1 Peter 4:14).

Scripture Study: Matthew 14

Matthew 14
This chapter gives another step in the rejection of the Lord, and then an outline picture of what follows the rejection of the Lord as King.
Matthew 14:1-12. Herod, hearing of Jesus’ fame, recalls what he had done to John the Baptist, and concludes that John was raised from the dead, therefore mighty works displayed themselves in him. His conscience seems to be troubling him; conscience will trouble every unsaved one. In the lake of fire it will be their worm that dieth not — they will never forget their sins. The spirit tells the heartless wickedness of those ruling over Israel at this time. Herod put John in prison because he bore a faithful testimony against his sin in having his brother’s wife. “It is not lawful for thee to have her,” and Herod desired to kill him. He was held back from doing it by his fear of the people, for all held John to be a prophet. But Herod’s birthday comes round, and with it the revelings in sin that the world calls a good time, and God is forgotten. Herodias’ daughter dances to please the king and with an oath he promises to give her whatever she asks. The wicked mother sets her on to ask the head of the faithful servant of God. The king is grieved but his false honor is at stake before his court, so John’s head is brought in and given to this wicked woman.
God has permitted it for the fulfillment of His purposes, and John exchanges an earthly prison for a heavenly paradise. The changes of dispensations, which he could not understand here, will be understood in a brighter scene. His disciples came and took the body and buried it, and went and told Jesus. We may think of what all this was to the Lord. It is not only a faithful servant removed; it is God’s testimony rejected and a precursor of His own sufferings, though personally far above John, and this place of testimony the Lord Himself took in grace as “the faithful witness” (Psa. 40:9-10). He therefore retires into a desert place. How He must have felt His position! What sufferings were His in this way also, for He never became hardened to things. He felt the growing evils and knew what it all led on to. His love for Israel; His separation from evil; His constancy in His Father’s presence made Him feel everything keenly.
Matthew 14:13-14. But He does not allow Israel’s state to hinder His goodness from flowing out to the needy. The people have followed Him on foot out of the cities and brought their sick; and as He goes forth and looks on them, His heart is moved with compassion, and He heals their sick. (Psa. 103:3).
Matthew 14:15-18. How different with the disciples! They see the multitude, and in their helplessness to meet their need, they want Him to send them away, but He answers, “They need not depart; give ye them to eat.” They express their weakness, “We have here but five loaves and two fishes.” And He said, “Bring them hither to Me.” Have we not often been guilty of such ways, when we should have taken up the case of the needy ones and reckoned on His fullness to meet the need? “Send the multitude away.” Away where? Away from Jesus! Can disciples give advice like this? Perfect servant! Patient and gracious Master! Make us more like Thee. Slow we are to count on His power and goodness, though we have proved it so often.
Matthew 14:19-21. “And He commanded the multitude to sit down on the grass, and took the five loaves, and the two fishes, and looking up to heaven, He blessed, and brake, and gave the loaves to His disciples, and the disciples to the multitude. And they did all eat, and were filled; and they took up of the fragments that remained twelve baskets full, and they that had eaten were about five thousand men, besides women and children.” Here the power and goodness of the Lord shine out in perfection. He is indeed seen as the Messiah who feeds His poor with bread. (Psa. 132:15). What plain proof of Emmanuel’s presence on earth.
Matthew 14:22-23. Following the rejection of the Lord as King, the Lord took His place on high as the great High Priest. Here we have it pictured: His disciples are constrained to get into a ship to go before Him to the other side, while he sent the multitude away. Then He goes up into a mountain to pray, and there He is alone. His disciples appear to be left alone to bear their troubles that follow, being attached to a rejected King, but in reality they benefit by His heavenly position (Heb. 7:25). He is absent from them; and so it is “the night” to the believer, while man has his day in the world. On high, He prays for His own, outwardly separated from them, but in reality spiritually nearer to them than ever. What a picture this is of the present time externally; but we know another Comforter has come, making good to us now, in a spiritual way, the presence of the Lord. (John 14:18,21,23).
Matthew 14:24-31. The remnant of the Jews will also pass through trouble, and in the midst of this, the Lord comes to them, walking on the sea. They see Him, and are troubled, but He calms their fears, saying, “Be of good cheer; it is I; be not afraid.” Peter, taking courage, says, “If it be Thou, bid me come unto Thee on the water. And He said, Come. And when Peter was come down out of the ship, he walked on the water, to go to Jesus.” This seems to bring out the path of faith; he had the Word for his path, and the Lord before him as his object; thus he walked, sustained by the Lord’s power, till he took his eyes off the Lord; then he saw the wind boisterous (yet it is as easy to walk on rough as on smooth water), and he was afraid and, beginning to sink, he cried, saying, “Lord save me.” We are well reminded of our foolishness here; the Lord is enough. If we keep the eye on Him, all is well; but nature might take credit, and Peter might have boasted of his attainment as he might have called it. But now he cannot; his failure is manifest; he walked only by the strength of Jesus. And now that he has failed, will the Lord let him sink? No, that faithful Savior is just the same, and immediately Jesus stretched forth His hand, and caught him, and said unto him, “O, thou of little faith, wherefore didst thou doubt:” We may fail, and fear in the path of faith, but the Lord will not fail at our cry of need and His strong hand will hold us up, though we are humbled rightly by our unbelief. May we learn to keep our eye fixed on the Lord.
Peter began to sink, just enough to make him get back in dependence on the Lord, and to make him feel his nothingness. How he would enjoy the care of the Lord on his way back to the ship, upheld by Jesus, but when in the ship he could not boast of his faith above the rest, yet after all he had proved the goodness of the Lord, and His faithfulness to him in a way those in the ship did not know. What comfort to Peter to lean on the Savior’s arm all the way back to the ship. Blessed journey! When we have proved what a resource He is for us.
Matthew 14:32-33. Here He is come into the ship. Israel’s troubles are over when He comes back to them, and they worship Him as Son of God.
Matthew 14:34-36, go further still. It is a foreshadowing of the Lord when He is joyfully received in the very world that once rejected Him; as one has said, “It is the blessing and healing of a distressed and groaning world, consequent on His return in acknowledged power and glory.”

Truths for Young Christians: On Our Relationship with the World

Chapter 19
On Our Relations with theWorld
It is evident that the limits of this short paper will only enable us to consider this vast subject in the briefest possible manner. The Christian is in the world, but not of it. He is surrounded with those who know not God, and it is a most important thing for him to know how to conduct himself in relation with worldly people, both publicly and privately, in social life, business life, and religious life.
Before, however, speaking briefly on this, we will just take up very shortly a few examples of relationship with the world, and its effects as seen in Scripture.
Worldly Marriages
In 1 Kings 11:1-8 we find the sad result in Solomon’s case of marrying strange wives, idolatrous women. No doubt, as many a child of God since, he trusted in his wise heart, in the splendid temple he had built, in his own long religious life, to lead him aright, but instead of that, they led him astray. And so it is in nine cases out of ten. A Christian marries a worldly girl, an idolater; that is, one who has her heart set on earthly things called idols (1 John 5). He hopes, no doubt, to set her straight, but having committed a sin by marrying her, he is soon led by her into another; for not only has he all her influence to lead him wrong, but that of his own deceitful heart as well. It is, indeed, lamentable to think how many ships, starting on their heavenward voyage, have been shipwrecked on the quicksands of this life, through sailing in company with an enemy’s vessel. For, hard as it is to believe, the young and attractive are equally Satan’s slaves with the gray haired sinner. Beware of Worldly marriages, which are condemned and forbidden by God (2 Cor. 6); perhaps few sins so surely meet with heavy chastisement, too often life long.
Most earnestly, then, would we warn young believers of this most fatal of all worldly alliances. Other false steps can be retraced at will, this NEVER. It may be these lines are read by some one whose affections are already engaged by some worldly person. We would warn you against such a marriage at your peril. Better far to have a broken heart for God’s glory, and one that He can heal, than to have a heart broken later on, as you surely will, through seeing with your eyes open, and discovering when TOO LATE, the fearful error you have committed.
World-Bordering
In Jehoshaphat we find another case of worldly alliance; this was in joining to fight a common enemy (2 Chron. 17) — no great sin apparently. The king of Syria was a foe to both of them, and the victory of one would help the other. Nevertheless, what saith the Lord? “Shouldest thou help the ungodly, and love them that hate the Lord? Therefore is wrath upon thee from before the Lord.” Has this no voice to those Christians who, like Jehoshaphat, would join the world to fight some common enemy There are common enemies: drink, vice, poverty, disease, are such to a great extent. It will, then, be seen at once that this one example strikes a fatal blow at all alliances of a social nature between believer and unbeliever. In this it is evident the Christian is not to join in improving or bettering the world. He must walk in his way and the world in theirs. Indeed, if the Christian is true, they cannot work together, for the ultimate end of the one is the advancement of the world, and the good of mankind, that of the latter the glory of Christ. In a place, therefore, where He is despised and still rejected by the world at large, it is evident that there cannot be much harmony in common pursuits — (See on this an important tract called “Jehoshaphat, a Word on World Bordering” by C. H. M.).
In Jehoram we get another instance of the evils of a worldly match (2 Chron. 21:6). Many as are the instances of the unbeliever leading the Christian astray, we do not remember a single case where in such a marriage the Christian brought the unbeliever right. In this case the evil is worse still, for not only is the husband led wrong, but the child also is led astray (2 Chron. 22:3) by the evil counsels of his mother. This, too, will often be found to be the case, especially when the mother is the unbeliever; and thus the result of one false step may descend to generations.
Again, in Ezra 9, do we get fatal instances of these unholy alliances. But surely we have had enough to show us what are almost invariably the results of thus dishonoring God.
Worshippers Must Be Christians
In Ezekiel 44:7, we come to another class of worldly fellowship, and that is in religion. One of the crying sins of Israel of old was that they brought in unbelievers to the temple worship, and the prophet is bid to “mark well the entering in of the house, with every going forth of the sanctuary”; and further, “No stranger uncircumcised in heart, nor uncircumcised in flesh, shall enter into My sanctuary, of any stranger that is among the children of Israel.” Our sanctuary, we know, can only be entered by believers (Heb. 10) in reality; but surely this plainly shows that in outward worship we cannot place believers and unbelievers together before God. It is not for us, of course, to try the hearts, and a hypocrite may creep in anywhere; but surely divine worship ought to be confined to the children of God. Indeed, none else can worship, and it is an awful mockery to see those who have no pretensions to be saved (not even the lip profession), joining with God’s people in singing His praises. This is strangely like this very sin of Israel of old. We do not now speak of preaching the gospel. At all such services unbelievers have their right place; but these are surely perfectly distinct from the worship of believers, spiritually within the veil. In religion, therefore, we cannot place believers and unbelievers on a common footing.
Neither can we seek the aid or help of the world (pecuniary or otherwise) in the Lord’s work.
Nehemiah avoided the danger of worldly help in Nehemiah 6, and one reason why he was so blessed was because he was so separate to God. The people of Israel, too, at this time entered into a curse and an oath, not to marry unbelievers (Neh. 10:29). Would that every one “having understanding” (Neh. 10:28), followed (without legality) the same course! Separation from worldly company is the result of having the Word as the enjoyed portion of the heart (Jer. 15:16-17).
The joy of the world and delight in the Word cannot go together. “Thy words were found, and I did eat them; and Thy word was unto me the joy and rejoicing of mine heart: for I am called by Thy name, O Lord God of Hosts. I sat not in the assembly of the wicked, nor rejoiced.” We have now seen enough to show us, without quoting the many precepts in the Word on the subject, that worldly marriages are in every way to be condemned and avoided, that all philanthropic and social worldly alliances are expressly condemned, however good their object. (Of course it is understood that only real alliances are here spoken of, that is, believers and unbelievers publicly banded together.) We have also seen that no religious mixture is to be tolerated either in worship or service (such as giving money). God’s people must be separate. The path is a narrow one still, and not less so because we live in Christendom. Indeed, now one needs to be closely walking with God to learn how to keep one’s feet separate from all such evil alliances, and have one’s heart wide enough for all right sympathies and feelings. May the Lord enable each of us, who desires to be true to Him, to discern the path of wisdom through this world, so as to be kept from all “unequal yokes,” learning at the same time to bear more of the “equal yoke” of Matthew 11:28 “Take My yoke upon you and learn of Me, for I am meek and lowly in heart, and ye shall find rest unto your souls.”

Hezekiah: Brief Lessons on Church Truth, Part 4

A Brief Outline of Lectures on Hezekiah
SANCTIFICATION
Read carefully 2 Chronicles 30.
The effect of killing the passover was, that the priests and the Levites were ashamed, and sanctified themselves. It is very important rightly to divide the word of life, as to the subject of sanctification. Three kinds of sanctification come before me in Scripture.
First, the absolute, immutable sanctification of every believer by the one offering of Christ. Immutably perfect, this can never change. This, as we have noticed, is abundantly seen in Heb. 10:10-19. This is what He hath done; and what He hath done must be perfect. Secondly, there is sanctification in wickedness; separating ourselves in wickedness — abomination in the sight of the Lord. “They that sanctify themselves, and purify themselves in the gardens, behind one in the midst (or one after another), eating swine’s flesh, and the abomination, and the mouse, shall be consumed together, saith the Lord.” “But to this man will I look, even to him that is poor, and of a contrite spirit, and trembleth at My Word.” In contrast with, “Yea, they have chosen their own ways, and their soul delighteth in their abominations. I also will choose their delusions,” (Isa. 66:17, 2-4). No doubt this applies to the Jews, and their cleaving to the man of sin.But it also has a solemn voice to us in these last days. Yes, the very last and worst marks of these last days. Can anything be worse than pretentious separation from others, but only to worse evil? O, beware of these mockers of the last time. “These be they who separate themselves, sensual, having not the Spirit” (Jude 19).
Thirdly, there is “Sanctification in Holiness.” This is not what we are by the work of Christ; but practical sanctification — sanctifying ourselves. Have you noticed how much there is in our chapters on Hezekiah about this, over and over again? “They could not keep the Passover at that time, because the priests had not sanctified themselves sufficiently.” This is not merely a question of gross sins, but defilement by touching the unclean. This we noticed in Numbers 9. Those who had touched a dead body were unclean. You could not say it was gross sin to do so. Neither could you say, True, that dead body is unclean but I am alive, and not unclean. This would be to deny the word of the Lord. All these types show that before the Lord, contact with evil, is evil. To touch the dead body is to be unclean. But, say you, What is to be done in the present defiled state of Christendom? If the true ground of being gathered to the name of the Lord to keep the Passover, the Lord’s Supper, as it is written, be equal to the reconciliation, as we have seen; and if the invitation is also equal, does it not follow that all ought to be together, no matter what defilement? Is there an express word of the Lord to us on this, in the midst of all the circumstances of these last days? There is; and it is very plain. “Let every one that nameth the name of Christ depart from iniquity. But in a great house, there are not only vessels of gold and of silver, but also of wood and of earth; and some to honor, and some to dishonor. If a man therefore purge himself from these, he shall be a vessel unto honor sanctified, and meet for the master’s use, and prepared unto every good work” (2 Tim. 2:19-21), and then, after describing the very corruptions of these days — yes, of that which bears the name of Christendom around us: “Having a form of godliness, but denying the power thereof.” What is the mind of the Lord? Does He say, keep in it; try to mend it: use your influence for good in it? No! but “from such turn away” (2 Tim. 3:1-5). This is to sanctify ourselves, to purge ourselves, to turn away from all known evil. But, Oh, let it not be to a worse thing, but in holiness. The Lord search our hearts, and, by His precious Word and Spirit, enable us to sanctify ourselves from all iniquity. Let us test ourselves by the precious words of Jesus. “They are not of the world, even as I am not of the world” (John 17). This brings us to a SEEMING DIFFICULTY.
There were many that had not sanctified themselves (2 Chron. 30:17), had not cleansed themselves; yet they did eat of the Passover otherwise than it is written. How was this difficulty met? Hezekiah prayed for them, saying, “The good Lord pardon every one that prepareth his heart to seek God, the Lord God of his fathers, though he be not cleansed according to the purification of the sanctuary. And the Lord hearkened to Hezekiah, and healed the people.” This is very beautiful, and important instruction to us. When we think of those balances of the sanctuary, the words of Jesus above, O, who is cleansed according to this purification? Not of the world, even as He is not of the world.
Suppose a Christian to be in such a place in the world, as a bishop of the Church of England. As a Christian, he is a priest — for all Christians are priests. But he has not sanctified himself. He touches the dead body of this world. He is linked with the state, and defiled with all the defilement of this world. And he never kept the Lord’s supper as it is written. But he earnestly desires once in his life to obey the Lord, and break bread as it is written. Does he thus prepare his heart? Does he seek to do the Lord’s will? Is this really the state of his heart? Who are they, then, whom the hand of the Lord hath gathered in mercy and grace, to hinder him from obeying the Lord this once? Surely there is blessed instruction for us in this. And then, as it was with the priests, so would it be with this bishop. Would he not, when he found the blessedness of the Lord’s supper as it is written, be ashamed of the link with the world, and cleanse himself? We ought to expect this. Let us not forget our dear precious Lord looks at the heart. Surely it would be altogether different with those who are deceiving, and being deceived, who do separate themselves, not having the Spirit. These may be easily known by their murmuring spirit, fully described in the Epistle of Jude. Many dear souls are only, it may be, deceived; and the Lord would have us so tender to these, “and of some have compassion, making a difference; and others save with fear, pulling them out of the fire, hating even the garment spotted by the flesh.” The Lord fill our hearts with love to all His own, who have been turned aside by men, and give us constantly to remember them in prayer. Surely we would fully own that none but the Holy Ghost is able to take care of the church of God in these last days.
Well, they kept the feast with great gladness, and made confession to the Lord God of their fathers. And all that came, even the strangers out of Israel, DWELT in Judah, and rejoiced. It is remarkably so now, seldom does a stranger Christian come from any revolted tribe or sect of men; but it is his joy to dwell, to abide, gathered to the name of the Lord Jesus. Rarely can one ever go back that comes with a true heart, seeking the Lord. It is altogether so different. O, how little the children of God know what deep, sweet communion of soul they miss. Truly it makes one ashamed — and well we may be — but all is pure, free grace, unclouded and unbounded. And if we confess, it is only now to feast again in His love. O, to really meet the Lord Himself! to sit at His feet! — the Holy Ghost present to glorify Him. Yes, deep, deep the joy. Great joy in Jerusalem; untold joy in His presence.
(Continued and to be Continued).

Excuses

I cannot go to meet the Lord,
As some will do today,
I do not feel so very well;
So I will stay away.
The night is wet and I can’t go
Where saints are met for prayer;
‘Tis only when the night is clear
That I am going there.
It is too cold for me to go,
This reading meeting night;
To risk my health by going there,
I think would not be right.
I know the gospel will be preached-
That message all so true,
But ‘tis too dark, the way’s too long,
I surely cannot go.
I like to hear a brother speak,
But I can’t sit so long;
‘Tis such a burden unto one
Who is not very strong.
I have so many things to do,
I really am not able
To go to evening meetings or
Be often at His Table.
Now is there not some remedy
For all this sad abuse
Of Christian rights and privilege,
And flimsy poor excuse?
Ah, yes! ‘tis when the heart is right,
And happy with the Lord,
Excuses fail, and glad response
Is given to His Word.
A heart that’s right with God, will oft
With saints be gathered round
The One who says, With such as these
My presence will be found.
Ere long, the privileges we have,
Will all have passed away,
And we will be with God’s Beloved
In God’s eternal day.
Let us no more excuses make,
But ever seek to be
Oft in the company of those
Who soon their Lord will see.
And if we’re faithful to His Word
And oft together come,
We’ll have a little foretaste of
The blessed rest of HOME.

The Coming and Reign of Our Lord Jesus Christ: The Age to Come, or the Millennium, Part 3

Following upon the establishment of Israel in the land, and their recognition of Jesus, the One whom they pierced, as the Messiah, their King, we find a terrible invasion of their country takes place by the powers of the North. Many have had the thought that immediately upon the manifestation of Christ in glory, the reign of peace and blessing will set in all its fullness; but this is not so. A terrible destruction of enemies takes place even after, though probably very shortly after Christ has appeared. A detailed account of this scene is given us in Ezekiel 38 and 39 and it is thought by many very possible to be identical with the last blow of the Assyrian, when he falls by divine judgment, which we have already glanced at.
Israel being gathered in their land with great wealth, the cupidity of a mighty power in the North is excited. The Western powers being destroyed, he thinks that his time is now come to possess the long-coveted land, and also the riches stored there. He is called in the prophecy “Gog, chief prince of Meshech and Tubal”; but the correct rendering of the original is “Prince of Rosh, Meshech, and Tubal,” and refers to the great Russian empire. He comes down on the land with a mighty army of his own, and accompanied by Persia, Ethiopia (Asiatic Ethiopia probably, territory by the Persian Gulf), Libya (or Phut by the Euphrates), Gomer, and all his bands (part of Asia Minor), and Togarmah (Armenia), and many peoples besides.
Israel will be dwelling safely without walls, bars, or gates when he comes down to take a spoil, and to take a prey (Ezek. 38:11-12). The fury of the Lord comes up in His face (Ezek. 38:18), and He says, “I will call for a sword against him throughout all My mountains: every man’s sword shall be against his brother. And I will plead against him with pestilence and with blood; and I will rain upon him, and upon his bands, and upon the many people that are with him, an overflowing rain, and great hailstones, fire and brimstone” (Ezek. 38:21, 22). The ravenous birds, and the beasts of the field, devour the vast host smitten by the judgment of God, and for seven years the children of Israel go forth and burn the implements of war, and take seven months to bury their bones. (Ezek. 39).
Thus close the awful judgments upon Israel and the nations, which God has foretold shall accompany the second advent of the Son of Man, and the establishment of His kingdom in the earth. Ezekiel 40 to the close of Ezekiel gives a description of the millennial temple, the redivision of the land amongst the tribes. This brings us to the full millennial blessing. At the present time the whole creation groaneth and travaileth in pain; but in that day it shall be delivered from the bondage of corruption into the glorious liberty (or liberty of the glory) of the children of God. (Rom. 8:20-21). The dragon, that old serpent, which is the devil and Satan, will be bound in the bottomless pit (or the abyss) for the thousand years, by an angel that shall come down from heaven (Rev. 20:3), and no longer be enabled to make this world the sphere of his wiles and delusions, leading men captive in their sins.
In this wondrous day, the earth’s jubilee, the Lord Jesus Christ, having come forth as the Sun of righteousness with healing in His wings, will reign in glory with His heavenly saints over the earth. God will head up all things in Christ in heaven and in the earth. The government will be upon His shoulder, and He will rule over all for a thousand years, as King of kings and Lord of lords.
The righteous shall shine forth as the sun with Him, in the kingdom of their Father, and the world shall know that He has loved them as He has loved Him. The throne of Christ shall be set up, and all kings shall fall down before Him, all nations shall serve Him. There shall be one King over all the earth, one Lord, and His name one (Zech. 14:9). It is then that His Name shall be called Wonderful, Counselor, the Mighty God, the Everlasting Father, the Prince of Peace. Of the increase of His government and peace there shall be no end. upon the throne of David, and upon His kingdom, to order it and to establish it with judgment and with justice, from henceforth even forever. The zeal of the Lord of Hosts will perform this. “And it shall come to pass in that day, I will hear, saith the Lord, I will hear the heavens, and they shall hear the earth; and the earth shall hear the corn, and the wine, and the oil; and they shall hear Jezreel” (Hos. 2:21-22). And the knowledge of the Lord and of His glory shall cover the earth as the waters cover the sea (Isa. 11:9; Hab. 2:14).
How blessed for the Christian, as he treads the pathway of faith, gazing upon His absent and rejected Lord, to know that He shall shortly come forth in power and great glory, to take the kingdoms of this world as His own, when He shall be universally honored, and the whole scene shall rejoice under His glorious and beneficent sway.
(Continued and to be continued).

What Doest Thou Here?

At a party given to some of the noted and wealthy of the land, where everything was accumulated that could minister to the gratification of the senses, a lady, accosting a gentleman, who had once made a high profession of following Christ, but who had turned back to the world, said to him, “I am surprised to see you here, Mr.-. What can you want here? This is no place for you.” Was not this an arrow from the Lord, which should have pierced the heart of the one who was thus addressed?
“Love not the world, neither the things that are in the world. If any man love the world, the love of the Father is not in him. For all that is in the world, the lust of the flesh, and the lust of the eyes, and the pride of life, is not of the Father, but is of the world. And the world passeth away, and the lust thereof; but he that doeth the will of God abideth forever” (1 John 2:15). “If any man draw back, My soul shall have no pleasure in him” (Heb. 10:38).

Correspondence: Grave; Brotherly Kindness and Charity

Question 80: What are the words for “grave” in Hebrew and Greek? Does the Hebrew word “Sheol” or the Greek word “Hades” ever mean the grave?
Answer: You do not need to be a Hebrew or Greek scholar to perceive the modern infidelity about a place of punishment. Any English reader may see in such a book as “Dr. Young’s Analytical Concordance”, that the Hebrew word “qeber” or “qeburah”, means “grave”, “tomb”, or “sepulcher”, and that the translators have improperly translated “Sheol” as meaning the grave in the following scriptures: Gen. 37:35; 42:38; 44:29, 31; 1 Sam. 2:6; 1 Kings 2:6, 9; Job 7:9; 14:13; 17:13; 21:13; 24:19; Psa. 6:5; 30:3; 31:17; 49:14, 15; 88:3; 89:48; 141:7; Prov. 1:12; 30:16; Eccl. 9:10; Sol. 8:6; Isa. 14:11; 38:10,18; Ezek. 31:15; Hos. 13:14.
The Greek word for “grave” is “mnema” or “mnemeion”. 1 Corinthians 15:55 is “hades,” not “grave.”
“Sheol” or “Hades” mean the unseen state in every place where either word occurs. The unseen state is the separation of the soul from the body, without determining where; but Scripture shows that the body is in the grave, and that the soul or spirit is either in bliss or woe. When the Lord Jesus died, He commended His Spirit to His Father, so that He was absent from the body, present with the Father; and the saved thief was absent from the body and present with the Lord who had saved him; and this is paradise, the garden of delights, not a prison underneath the earth, but in heaven above (2 Cor. 12:2,4).
The Lord Jesus did not go into any prison to preach, or to free captives when He was dead. He was for the time in death, but could not be holden of it (Acts 2:24).
In the parable of the rich man and Lazarus (Luke 16), the body goes into the grave; Lazarus is seen in the place of bliss (Abraham’s bosom is that to the Jews): the rich man is in torments; both are in the unseen state.
We do not speak now of either the wicked who die without Christ, or the Christian, washed in the blood of Christ, as being in Hades; for the truth enables us to speak with certainty of those who are lost as awaiting judgment, while the Christian is with Christ, which is “far better” than being here; but he is still waiting for Christ’s coming, when he will receive his body, glorified with Christ — glory belongs to the body.
“Hell” is “Hades” — the unseen — in Matthew 11:23; 16:18; Luke 10:15; 16:23; Acts 2:27,31; Revelation 1:18;6:8; 20:13-14.
“Hell” is “Gehenna” — the place of suffering — in Matthew 5:22,29-30; 10:28; 18:9; 23:15,33; Mark 9:43, 45,47; Luke 12:5; James 3:6.
“Hell” is Tartaroo” — the prison, awaiting the judgment (2 Peter 2:4).
Question 81: What is the difference between brotherly kindness and charity, rendered in the New Translation “brotherly love” and “love”? N. F. C.
Answer: Brotherly love is a precious fruit of grace. It has Christians in view, because they belong to the Lord. When genuine and pure, it flows from grace.
It is apt to avoid all that is painful to its objects, not to mar the mutual pleasantness of intercourse, and thus make them the measure of its conduct. If love of the brethren is our first object, we are apt to conform to them. We are therefore enjoined to put on love, which is the bond of perfectness (Col. 3:14). In 2 Peter 1:7, we have, “In brotherly love, love.” This love will cause genuine brotherly love, but it brings in God — love is His nature. “Hereby know we that we love the children of God, when we love God and keep His commandments” (1 John 5:2). This love is exercised as in the presence of God, and therefore rejects what is inconsistent with His presence. It is the spirit of love working, but the Spirit of God — the Spirit of truth — is the power, and therefore rejects and rebukes what is against the truth. It rejoices not in iniquity, but rejoices in the truth. (1 Cor. 13:4-7).
If our brotherly love is not springing from this fountain, it easily sinks down to natural kindness, so that man may be given the prominent place instead of God, and evil, that should be rebuked, may be covered up for peace’s sake. Love gives to God what rightly belongs to Him, for it is the activity of His nature. His love was shown at the cross in judging evil. Christ bore the judgment for our sins.
There can be no true love in indifference to evil, yet it covers a multitude of sins, for it goes after the one who is gone astray or is in danger of going astray, and by its activity the sins never come to view. We should not be careless about sin, nor should we be careless about our brother’s real good.
Divine love abhors evil, but rises over it, dealing with it in needed chastisement. Our brotherly love should flow from divine love having its true place in our hearts.

Inspiration of the Scriptures: The Passover and Lord's Supper

The Passover and the Lord’s Supper
In a theological work published, among many other charges against the Holy Scriptures, we are told that Matthew, Mark and Luke made a “mistake” in assuming that “the Lord’s supper was the Passover feast”; that “the two were regarded as identical”; and also that our Lord “had desired to eat the Passover with them, but He did not eat it.” There is nothing new in these charges, though they are very far from the truth.
1. On turning to the gospels, we find that Matthew, Mark, and Luke, each, according to the line of things given him by the Spirit, supplies us with particulars as to the eating of the Passover. John usually takes the divine side, and enters little into Jewish circumstances. His gospel is founded on their rejection of the Messiah, as stated in the beginning, “He came unto His own, and His own received Him not.”
With regard to the Passover, Matthew says that our Lord sent some of His disciples into the city to make ready the Passover. They said unto Jesus, “Where wilt Thou that we prepare for Thee to eat the Passover? And He said, Go into the city to such a man, and say unto him, the Master saith, My time is at hand; I will keep the Passover at thy house with My disciples .... and they made ready the Passover. Now when the even was come, He sat down with the twelve. And as they did eat,” (Matt. 26).
Mark’s account is the sending forth of two of His disciples into the city, much in the same way. They were to say to the goodman of the house, “Where is the guest-chamber where I shall eat the Passover with My disciples?.... and they made ready the Passover. And in the evening, He cometh with the twelve. And as they sat and did eat,” (Mark 14).
Luke gives the same account as to the furnished room. He tells us that the two disciples sent were Peter and John. They were to say, “Where is the guest-chamber where I shall eat the Passover with My disciples?... and they made ready the Passover. And when the hour was come He sat down, and the twelve apostles with Him. And He said unto them, With desire I have desired to eat this Passover with you before I suffer. For I say unto you, I will not any more eat thereof, until it be fulfilled in the kingdom of God” (Luke 22).
Thus far the account of these three Evangelists is entirely about the Passover. Where, then, is the mistake? Where is there the smallest evidence, that up to the actual eating of the Passover the disciples had an idea of its being anything but the Passover feast?
Where, then, are the writers’ mistakes? Where is there the faintest intimation that the Passover and Lord’s Supper were assumed by these inspired writers to be identical? How appalling are such rash charges, and how calculated to promote infidelity, and to bring the Holy Scriptures into contempt!
2. We have just seen that the Lord desired to eat the Passover with His disciples; that He sent Peter and John to procure a room where He might eat the Passover with His disciples; that at the appointed hour He and the twelve sat down, and as they sat and did eat, Jesus said, “I will not eat any more thereof.” Referring to Judas, He also said, “He that eateth bread with Me”. Can there be a doubt, then, that our Lord did eat the Passover? How very serious for any one without a shadow of proof to say, “He did not eat of it”!
The fact is, that the Passover having been eaten by our Lord with His disciples according to Jehovah’s mind, for the last time on earth, before the kingdom comes, in the immediate anticipation of His rejection as Messiah, and His death as a Sacrifice for sin, His resurrection and ascension, He institutes another thing which was not a Jewish feast, but a Christian ordinance, and of very different import. Though they both set forth to faith the Savior’s death and blood-shedding, the Passover was a memorial of the blood of the paschal lamb in Egypt, sheltering from judgment; but in the Lord’s Supper, the wine is to faith the memorial of the blood of Christ which speaks to the believer of remission of sins, and of his being perfected forever by that one offering (Matt. 26:28; Heb. 10:2-20). The blood of Christ assures us of redemption accomplished, sins purged, conscience purged, and the worshipper purged, thus having boldness to enter into the holiest. It would, therefore, be impossible for those who received God’s testimony to the eternal efficacy of “the offering of the body of Jesus Christ once for all,” to regard the Passover and the Lord’s Supper as identical. It is then impossible that Matthew, Mark, and Luke could have assumed that “the Lord’s Supper was the Passover feast.”
As to the Supper being instituted, Matthew says, “As they were eating;” Mark, “As they did eat.” Luke gives a fuller account of the details of the Passover feast, and adds that our Lord said, in reference to eating it, “I will not any more eat thereof, until it be fulfilled in the kingdom of God.” After this, “He took bread [a loaf], and gave thanks, and brake it, and gave unto them, saying, This is My body which is given for you: this do in remembrance of Me. Likewise also the cup after supper, saying, This cup is the new testament in My blood, which is shed for you.” Mark says, “shed for many.” and Matthew, “shed for many for the remission of sins.” We ask, then, was there anything like this in the Passover feast? Is it not clear that it was at the close of the eating of the Passover that our Lord instituted His supper? It is well, however, to remember, that since then our Lord has spoken from heaven about His Supper; and as, by His rejection by the nation, the kingdom is in abeyance, He has taught us that, “As often as ye eat this bread, and drink this cup, ye do show the Lord’s death TILL HE COME” (1 Cor. 11:23-26). We need scarcely add, that His “coming” for His saints will be before His “appearing and His kingdom.”
(Continued).

Noah's Carpenters

“You don’t look at all like a patient, Miss Emmie,” I said, as a fresh, rosy-cheeked girl of seventeen, the very picture of health — the daughter of Christian parents — came one day into my consulting-room.
“No, Doctor, I’m not come for advice, but mamma said that she thought you would help me with a little subscription;” and at the same time she produced a collecting book, entitled, “Indian Vernacular Society.”
“What is the object of this society?”
“O, its object is to teach the little boys and girls in India to read the Bible in their own language; and I am doing all I can to help it forward,” she answered most eagerly.
“A capital idea,” I replied. “I suppose, then, the real object is that the children may hear of Jesus, and be brought to believe in Him, and thus be saved, and know that they are?”
“Exactly so.”
“Well, I hope the Lord will use this effort to the blessing of many of them,” I replied; “but before going further, may I ask you, Miss Emmie, did you ever hear of Noah’s carpenters?
“Noah’s carpenters! No; who were they?” she replied, rather uneasily.
“They were people who may have helped to build the ark, by which others were saved, and yet never went in themselves.”
“I never thought of them before.”
“Very likely. But do you not think you are somewhat like them? Here you come trying to help other people to be saved, and yet, so far as I have ever heard, you are not saved yourself. Tell me, do you think you have ever yet come to Jesus yourself, and had your sins washed away? To put it plainly, Are you saved?”
This query was followed by a lengthened silence; her face flushed crimson, her eyes filled, and then, with a burst of tears, she replied: “No, I know I am not saved. I see, I have been like Noah’s carpenters.”
The bow drawn at a venture had truly entered the joints of the harness, and she was from that moment a spirit-wounded and convicted sinner. A long and interesting conversation followed, which I need not relate. We looked at the Word of God, and she found out to her utter dismay and distress, that all her own righteousnesses were but as filthy rags in the sight of God, and that she was an utterly lost soul, needing cleansing and pardon. In this awakened state, after prayer with her, she left me.
Some weeks rolled by, and I was wondering what had been going on in my young friend’s soul, when she again came at my consulting hour. Her pale anxious face betrayed what her words soon confirmed, namely, that since we parted she had passed through days and nights of deep soul-anguish.
“Mamma said she thought I might come and see you again, for I am so miserable and wretched, I don’t know what to do;” and, indeed, she looked all she said.
“I am most glad to see you, Miss Emmie. I suppose today you want something for yourself, not for others?”.
“Yes. I am most anxious to be saved, if I only knew how to come to Jesus; but I am so wicked, and my heart so hard, and I feel so dead.”
“You must come to Him as you are — in all your sins — for He has said, ‘Him that cometh to Me I will in no wise cast out.’ Just believe Him simply. Take Him at His word.”
“I do believe on Him, but I don’t get any good from it. I don’t feel any different.”
“You must not look at your feelings; you must just hear what He says, and give heed to His Word. Now, look at this verse,” and I turned to John 5:24. “Mark what Jesus says, `Verily, verily, I say unto you, He that heareth My Word, and believeth Him that sent Me, hath everlasting life, and shall not come into condemnation, but is passed from death unto life.’ Now, tell me, who is speaking here?”
“Jesus.”
“And to whom is He speaking?”
“To me.”
“Well, do you hear His Word?”
Yes.”
“And do you believe Him that sent Him? Do you believe God sent Jesus to save you, to die for you, and to wash away your sins?”
“Yes, I truly believe He did.”
“Now, then, see, you have complied with the two conditions given, you have heard and believed; listen to the three blessed consequences that the Lord says accrue to the one that hears and believes. Such an one `hath everlasting life,’ that is a present possession. Inasmuch as you hear and believe, what does Jesus say you now possess?”
“He says I have ‘everlasting life.’” “Good. Stick to that. But there is more in the verse. He says, he that heareth and believeth `shall not come into condemnation.’ That, you observe, provides for the future. There can be no condemnation for the one who believes in Jesus, because He Himself, on the cross, bore that condemnation. Now, since you have heard and believed, what does He say as to your future?”
“He says I shall not ‘come into condemnation.’”
“If He says you shall not, do you think you ever can?”
“No; of course not. He would not tell me what is not true. He cannot lie.”
“Exactly so. Thus, you see, He meets the present and the future in this verse. Nor is that all. We all lay in death; we were, each one, ‘dead in trespasses and sins,’ and out of that state we pass, the moment we hear His voice, for He quickens us by His Word; and so He adds here that the one who hears and believes, ‘is passed from death unto life.’ Nothing could be simpler or more blessed.”
“Yes, I see it now. I have heard and believed, and therefore, I have ‘passed from death unto life.’ O, how simple it all seems now!” and the pent-up feelings again got relief in a shower of tears, not now tears of conviction and distress, but those joyous, gladsome tears that will flow down the cheeks of a redeemed, pardoned, blood-washed sinner, when God’s grace is tasted and enjoyed. I prayed with her, and thanked God for His grace in saving her; and she left full of peace and joy in believing.
Many years have elapsed since my young friend found Jesus, but I rejoice to know she goes on her way, a bright, happy witness of the Lord’s grace, and is an earnest laborer for Christ, and a true soul-seeker in her own quiet sphere.
Reader, where about are you? Are you a Noah’s carpenter or a real genuine Christian? Let not this hour pass away and leave you as it found you. Did it find you unsaved? As you value your soul, let it not pass away and be forever a witness against you and your unbelief. Be persuaded to come to Jesus now. Then shall your future be bright and joyous, for you will be saved, sanctified, and satisfied.

Truths for Young Christians: On a Start in Life (for Young Men)

Chapter 20
ON A START IN LIFE.
To YOUNG MEN.
Our readers are necessarily divided, with regard to this subject, into two classes — those who have started in life, and those about to start. It is for the benefit of the latter that we especially write. There is no doubt that the most critical moment for a young believer is when he is called upon to make a start for himself, to begin the voyage across the great ocean of life, with apparently no hand on the tiller but his own; we say, apparently, for reasons that will be seen further on.
The Start
To start is a very real thing, but may take place actually in a variety of ways. To young men, for whom we now write, it occurs when the well-thumbed lesson books are finally laid aside, and the new clerk takes his seat for the first time on the high stool, or the sailor-boy steps on board his first ship, or the apprentice gets his first instruction in his future trade, or the undergraduate first dons the cap and gown. That which makes the act so serious, is not the mere fact that the steps which were only yesterday directed to the well-known school, are now turned to the office, the ship, the workshop, or the college, but that the boy has all at once sprung into the man. It is true that at times he seeks to blossom into the “genus homo,” even at school, but this is distinctly premature, and all such attempts are justly checked. But when once a boy enters a profession, a trade, or any other calling, and begins to fight the battle of life, he justly expects to be considered and regarded, at least, a young man.
The Dangers
Herein lies the chief danger for the young Christian. Up to this time he has taken all that his parents have told him for granted.
He has steadily attended the well-known church, chapel, meeting, or Sunday school, where he first learned the value of the blood of Christ; and, shielded in a comfortable home from temptation, he has caught, hitherto, but stray glimpses of the sea of wickedness without. But now comes the time when his principles are to be tested. He is sent away to a strange town, he lives in lodgings, he is thrown amongst a set of godless, careless, and often immoral young men; he is surrounded on every side with new and strange temptations. O, how many dear bright young believers have made shipwreck of their faith on these fatal rocks which are met with on first sailing out of the harbor of home! It is not too much to say that one week at a time like this, largely determines a young man’s fate for years.
How to Meet Them
If being forewarned and therefore forearmed, he leaves his home a bright, happy Christian, prepared to stand for God, and test, in a fiercer fight, the strength already gained in many a little skirmish at school; if he firmly believes in the truth that if the devil is resisted, he will flee from him, and shows his colors at the first opportunity at his work and in his leisure hours; if on the first night in his lodgings he opens his Bible, and, after reading God’s Word, prays to his Father in heaven, the victory is as good as won. In the first place, he is at once saved from a thousand temptations by showing his colors, for the really vicious at once shrink away from an openly declared Christian, and will seldom long trouble a man who at once stands up for Christ; in the second place, the stand he has taken, to a certain extent commits him for the future, and makes his life comparatively easy after the first week; and, thirdly, he having honored God, God will honor, protect and strengthen him.
We Have a Father to Guide Us
But now there is another matter, and it is this. We spoke of the young man starting on the voyage of life, his hand apparently holding the tiller, and guiding the ship. Now many a young man, and even a young believer, thinks that this is not only apparently, but is really so, and that he is the architect of his own fortunes, and that it is his will that is to direct his future life. Many accept Christ as their Savior who have but a very faint idea of what it is to accept God as their Father, and yet the one relationship is as true as the other; and if the one makes them happy for eternity, the other is certainly the secret of true happiness for time. There is a wonderful difference between the young man who goes forth rejoicing in his own strength and sagacity, and thinks that he can outwit the world; and the humble Christian, who leaves home placing the tiller of the little vessel of his life into his Father’s hand, and trusts Him to guide him aright through the dangers and difficulties of each day. “Trust in the Lord with all thine heart, and lean not to thine own understanding.” Do not think that any detail of your new life is too small for God to guide you in. The choice of your business, of your future home, of your companions, should all be entrusted to Him; and He will greatly own and honor such confidence, and lead you in the very best path. For it is folly to suppose that if we have a loving and all-wise Father, He would or could do anything else. The poet’s words are indeed true:
“All that God does, or suffers to be done,
That we ourselves should do,
Could we the future of our lives as clearly scan,
As He does now.”
The Bible Our Chart
Start, then, in life with a definite trust that “God will guide”; and though you apparently are steering the ship, get all your orders from above, so that, after all it is His hand, not yours, that is really holding the tiller. One other word and we have done. A ship requires a chart and compass as well as a rudder. Now the Christian’s chart is the Word of God, which shows him his course plainly down here, telling him that his first object should ever be, under all circumstances, the glory of God; that he is left here for this very purpose, not to please himself, but Christ.
The compass is the conscience, instructed by the Word of God, that tells me in an instant when I am out of the true course.
Make a Good Start
We would, in closing, again entreat every young man just about to sail out of the harbor, to make a good start. If he wavers at first, or yields a little for the sake of peace, he will not get it; but on the contrary, he may be drawn on, little by little, from bad to worse, until no outward sign of Christianity is left at all. A bold front at first, saves a great deal of trouble and fighting, afterward. Be sure, however, that the trust is not your own strength, but that every step is taken with prayer and dependence on God. As for the future, leave that with your heavenly Father, seeking only to live each day more truly to His glory than you did the day before. Such a course is worth a hundred sermons, for who can tell the mighty power of the unconscious influence exercised by a consistent Christian life?

Pleasure

Psalm 16:11
Once I looked for Pleasure
From this empty scene;
Counted all for Treasure,
Which was golden sheen;
Floated down the current
On light Folly’s wave;
Plunged in Passion’s torrent,
Freely, yet a slave.
Ah! the world can never
Happiness impart;
None but Christ can ever
Fill the craving heart.
Water, fresh and living,
From His bosom flows,
Purest pleasure giving,
While it life bestows.
Ye, who thirst for Pleasure,
Why from Jesus rove?
Come, and prove the measure
Of His perfect love;
‘Tis an ever-flowing,
Ever-running well,
Fullest bliss bestowing,
Joy, which none can tell.
Drinking of the river
Of this sacred joy,
Thanks to Thee, the Giver,
Must the heart employ.
Savior, Life eternal,
Holy, blessed Lord,
Sun of bliss supernal,
Be Thy Name adored.

Scripture Study: Matthew 15

Matthew 15
What a contrast is seen here between man and God. Man satisfies himself with outward show. God desires the purity and obedience that suits the light of His presence. Man used the law and ordinances to adorn himself. God had given it to prove to him the exceeding sinfulness of sin. The Pharisees’ systematic teaching set aside the Word of God, and in pretended piety went on with concealed Iniquity, lower than natural conscience. They used the law to minister to their pride, instead of seeing the corruptness of their hearts. Their own will and indulgence in lust was preferred to the duties God imposed upon them — washing their hands instead of purifying their hearts. It was outside show; God could not accept the worship of hypocrites.
This is what we find in Matthew 15:1-9. The scribes and Pharisees from Jerusalem spoke to Him of His disciples transgressing the traditions of the elders. It seemed serious to them that such ancient rules should be set aside. The Lord goes straight to the point. “Why do ye also transgress the commandment of God by your tradition? For God commanded, saying, Honor thy father and mother: and, He that curseth father or mother, let him die the death. (Ex. 21:17). But ye say, Whosoever shall say to his father or his mother, It is a gift, by whatsoever thou mightest be profited by me; and honor not his father or his mother, he shall be free.” If instead of supporting their parents, the money needed to do so was given to the priest, or temple service, then the children were free, that is, they did not need to support their parents. “Thus have ye made the commandment of God of none effect by your tradition. Ye hypocrites, well did Esaias prophesy of you, saying, This people draweth nigh unto Me with their mouth, and honoreth Me with their lips; but their heart is far from Me. But in vain they do worship Me, teaching for doctrines the commandments of men.” The Lord uncovers the heart of these hypocrites who set aside His Word to carry out their own traditions. It did not look bad to wash their hands before eating, but the Lord saw that it was only outward show, and God was not known in His true character.
And does not the Lord still see this evil in the professing church? Is not His Word set aside by the imposed rules and regulations of men? The living Savior is neglected for ordinances called means of grace.
The Holy Spirit’s presence on earth is set aside by the minister or chairman of today who rules the meetings. The living reality of His presence in the Christian is slighted by repeating prayers or reading them out of a book (Rom. 8:26; Jude 20), and what is called church membership is claimed instead of “peace with God through our Lord Jesus Christ”. And well might Esaias prophesy also of this present generation, for 2 Timothy 3:1-7, is descriptive of Christendom now.
Matthew 15:10-11. The Lord now calls the multitude and said unto them, “Hear and understand: not that which goeth into the mouth defileth a man; but that which cometh out of the mouth, this defileth a man.” God begins inside; man is a sinner, his heart is deceitful above all things and desperately wicked. (Jer. 17:9).
Matthew 15:12. But this offends those religious hypocrites, and the disciples tell Him of it. “Knowest Thou that the Pharisees were offended, after they heard this saying?” The Lord now tells their terrible condition.
Matthew 15:13-14. “Every plant, which My heavenly Father hath not planted, shall be rooted up. Let them alone: they be blind leaders of the blind. And if the blind lead the blind, both shall fall into the ditch.” Terrible indeed was their condition, and terrible is the end of all Christless profession.
Matthew 15:15-20. But Peter did not understand it, and said, “Declare unto us this parable.” The Lord answered, “Are ye also yet without understanding? Do not ye yet understand, that whatsoever entereth in at the mouth goeth into the belly, and is cast out into the draft? But those things which proceed out of the mouth come forth from the heart; and they defile the man. For out of the heart proceed evil thoughts, murders, adulteries, fornications, thefts, false witness, blasphemies: these are the things which defile a man: but to eat with unwashen hands defileth not a man.” What a sink of iniquity the heart of fallen man is, including my heart and yours! How necessary it is to know that Romans 6:6, is God’s judgment upon it, and that now we are to reckon ourselves dead indeed unto sin, but alive unto God through Jesus Christ our Lord: and this we are to do diligently.
Matthew 15:21. The Lord now departs, leaving the learned professors and infidels with their religion and their arguments, to go to the cities of Tire and Sidon — cities famous for their ignorant wickedness (Matt. 11:21,22).
Matthew 15:22-28. A woman of Canaan came out of the same coasts, and her daughter is possessed of a demon. She cries unto Him: “Have mercy on me, O Lord, Thou Son of David; my daughter is grievously vexed with a devil” (demon). But as “Son of David” she had not the slightest claim on His mercy, nevertheless, her faith recognizes Him as such. He answers her not a word, yet His heart is full of mercy, He is but wanting the proper condition in her to give the blessing. The disciples do not understand, they came and besought Him, “Send her away, for she crieth after us.” As “Son of David He could not help her, neither would he send her away. His answer to them is”, “I am not sent but unto the lost sheep of the house of Israel.” Then she came and worshipped Him. Her faith owned He was Israel’s Messiah, but greater still, He was God; He could meet her need, so she said, “Lord, help me.” Can His loving heart resist such an appeal? It was needful to do so for a moment, while He says, “It is not meet to take the children’s bread, and to cast it to dogs.” This seemed hard, but it was true; there were no promises to Gentile dogs, God had chosen Israel as His people. Yes, she owned that, too; but her faith counted on the grace of His heart, and she said, “Truth, Lord: yet the dogs eat of the crumbs which fall from their Master’s table.” How this pleased Him, as faith ever does; and how His heart gives forth the pent up blessing: “O, woman, great is thy faith: be it unto thee even as thou wilt. And her daughter was made whole from that very hour.”
Matthew 15:29-39. Next we see the Lord on a mountain in Galilee. There the afflicted and needy remnant of Israel have all their needs met, and they glorified the God of Israel. He proves again, as He had done before, that Emmanuel was with them, both in healing their diseases and in feeding the hungry (Psa. 103, and Psa. 132:15-16). Here He is acting out of His own blessed perfection as seen in the seven loaves and seven baskets. He provides perfectly for the remnant of His people Israel: and it is ours to prove that He is the same yesterday, today, and forever, to meet our every need (Heb. 13:8).

Hezekiah: Brief Lessons on Church Truth, Part 5

A Brief Outline of Lectures on Hezekiah
Now the Hammer and the Ax
Read carefully 2 Chronicles 30.
2 Chronicles 31:1-2
Yes, “Now when all this was finished, all Israel that were present went out to the cities of Judah, and brake the images in pieces, and cut down the groves.” As with the Thessalonians, they turned first to God, then from idols. We must be purged worshippers first within the veil, before there can be power for testimony without. This is God’s way. Then Hezekiah appointed the courses of the priests, and the Levites after their courses, every man according to his service. And did not the ascended Lord give gifts, every man according to his service? (Eph. 4). The Holy Ghost distributing to every man severally as He will (1 Cor. 12). Is He not the same Lord above? Is He not the same Spirit here below? “Every good gift, and every perfect gift, is from above, and cometh down from the Father of lights, with whom is no variableness, neither shadow of turning” (James 1:17). However the church may have failed — doors shut and lamps put out — have not as many as have been gathered to Christ found the Father the same; the Lord, the Holy Ghost the same? “No variableness, neither shadow of turning.” Let us, then, in faith wait on the Lord, and He will put both priests and Levites in their courses, every man according to his service — for worship or service. This brings us to the question of
Fruit
Read 2 Chronicles 31:5-11.
How beautifully the order is still brought out! Now is the time for fruit. And what abundance did the children of Israel bring; of corn, wine, and oil, and dates. And the tithe of holy things consecrated unto the Lord their God, and they laid them by heaps; or, heaps, heaps. As all fruit must be in the power of resurrection to be perfect, so in this type, “In the third month they began to lay the foundation of the heaps, and finished them in the seventh month.” What a principle this is, and so little understood: “Dead to the law by the body of Christ: that ye should be married to another, even to Him who is raised from the dead: that we should bring forth fruit unto God” (Rom. 7:4). “That I may know Him, and the power of His resurrection” (Phil. 3:10). The Holy Ghost could not be given to dwell in us until Christ was risen from the dead and glorified. And if the Spirit could not be given until then, how could we have the fruits of the Spirit? What a contrast this is to man under law! But is it not a universal fact, wherever the doors are shut, and the lamps put out? Yes, wherever men are not led by the Spirit, they are invariably placed under the law, for fruit-bearing. Just as the opposite is also true, even as it is written: “But if ye be led of the Spirit, ye are not under the law” (Gal. 5:18).
How little is this true, only resurrection — foundation principle, in the power of the Holy Ghost, for fruit-bearing, understood. We might meditate on this with great profit. Has anything occurred answering to these heaps, heaps of fruits? The doors have been opened again by a full gospel. The Holy Ghost is again owned in the assembly; the immutable perfection of the believer, by the sacrifice of Christ, again revealed. Joy and worship, the result. The Lord’s supper again as it is written. The Lordship of Christ owned; and the blessed fact again enjoyed, oneness with the risen Christ. The Holy Ghost known and owned as sent down to lead and guide. All this, not the work of men, but the hand of the Lord. Now, can it be denied that the blessed Spirit, now owned again, has poured forth such a stream of Christ — exalting ministry, by tongue and pen, as the church never knew since He was set aside at the close of the apostolic age? This, not for money, or worldly applause, but the Holy Ghost, leading the children of God thus to serve in consecration to the Lord. Spiritually, we may say with Azariah, “Since the people began to bring the offerings into the house of the Lord, we have had enough to eat, and have left plenty: for the Lord hath blessed His people; and that which is left is this great store.” Now this is a notable fact, that since the Holy Ghost has been known, and oneness with the risen Christ believed, the most astonishing numbers of tracts and pamphlets, books and periodicals, have been sent forth, without any sectarian motive, but “unto the Lord,” and for the food and edification of His people. “Tracts, periodicals,” said a brother to me the other day; “I do not know what to do with them; it is impossible to read them all.” I dare say it would have been impossible for Hezekiah to have eaten all the oxen, and rams, and heaps, heaps of fruits. But was that God’s intention? And, my brother, you may have mistaken the Lord’s intention; it may not be that His only thought is, that you should eat all the heaps, heaps of precious fruits, thus laid up in the chambers, or depots, of the treasuries of the Lord, but send them out to others.
(Continued)

The Coming and Reign of Our Lord Jesus Christ: The Age to Come, or the Millennium, Part 4

In Revelation 21:9 we get a most magnificent description, in figurative language, of the glory of the church during this period. She is spoken of as “the Bride, the Lamb’s wife,” but viewed as a beautiful city, the holy Jerusalem, descending out of heaven from God, and having the glory of God. All manner of precious stones, pearls, and gold, the most valuable objects in the eyes of men, are used to convey its glory to our minds; the wall, denoting security and salvation, being a prominent feature. It contains no temple; for the Lord God Almighty and the Lamb are the temple of it. Neither does it need the sun or moon; they are eclipsed by the glory of God, and the Lamb is the light (or lamp) thereof. The earth is radiant with its glory, the nations walking in its light, and the kings of the earth bring their glory into (to) it. And the gates are always open; there is no night there. The glory and honor of the nations shall also be brought into (to) it. All evil is forever excluded, and only those who are written in the Lamb’s book of life enter its glorious precincts.
A heavenly river of water of life, clear as crystal, shall flow out from the throne of God and of the Lamb. The tree of life, with twelve kinds of fruits, yielding monthly, and leaves for the healing of the nations, will be found there also (Rev. 22:1-2).
His servants shall serve Him, shall see His face — blessed, blessed portion! — and His name shall be in their foreheads, and they shall reign to the age of ages (Rev. 22:3-5); they need no candle, neither light of the sun; for the Lord God giveth them light.
In this coming day of glory and blessing, Israel, who are now scattered, but will then, as we have seen, have been gathered back to their own land, will have the first place among the nations, and Jerusalem become the metropolis of the whole earth.
One of the first acts of the reign, of Christ will be the erection of a magnificent temple; “Behold the man whose name is The Branch; and He shall grow up out of His place, and He shall build the temple of the LORD (or Jehovah); even He shall build the temple of the LORD; and He shall bear the glory, and shall sit and rule upon His throne; and He shall be a priest upon His throne” (Zech. 6:12-3). From the Ezekiel 40 to Ezekiel 42, we have a full description of this glorious building, and its measurements, and it shall be “called an house of prayer for all people” (Isa. 56:7).
One remarkable feature connected with this will be the restoration of the Jewish sacrifices and worship by the ordinance of God. A number of details are given in Ezekiel 45-46, and both the burnt, meat, drink, sin and peace offerings expressly mentioned. And “in those days  ... . David shall never want a man to sit upon the throne of the house of Israel; neither shall the priests the Levites want a man before Me to offer burnt-offerings, and to kindle meat-offerings, and to do sacrifice continually” (Jer. 33:16-18). “And they shall come from the cities of Judah, and from the places about Jerusalem, and from the land of Benjamin, and from the plain, and from the mountains, and from the south, bringing burnt-offerings, and sacrifices, and meat-offerings, and incense, and bringing sacrifices of praise, unto the house of the Lord” (Jer. 17:26). “Also the sons of the stranger, that join themselves to the Lord, to serve Him  ... . their burnt-offerings and their sacrifices shall be accepted upon mine-altar” (Isa. 56:6-7).
These in no way deny or detract from the one perfect offering of the Lamb of God, and the infinite value of His precious blood, which cleanseth us from all sin (Heb. 10:10-14; 1 John 1:7), but will point back to that offering as those of old pointed forwards. Men, biased by system, early teaching, and so on, are often slow to accept the truth of this; but there it is in the Word of God, and what the Lord has spoken He will surely perform.
(Continued and to be continued)

Correspondence: Hyssop Dipped in Blood

Ques. 82. What does the bunch of hyssop dipped in the blood that is in the basin, show us? (Ex. 12:22). H. G. L.
Ans. The hyssop pictures man’s littleness, as the cedar pictures his greatness. (1 Kings 4:33). From the greatest to the least, man by nature is nothing for God. All that he is, is ended in the death of Christ; this is seen in the sacrifices. (Lev. 14:4,6,49,51-52; Num. 19:6,18).
In dipping the hyssop in the blood and sprinkling the doorpost, it is as if the Israelite said: I am only a worthless sinner; Christ is everything. His blood is my shelter from the judgment of God which my sins deserve.

Inspiration of the Scriptures: The Call of Abraham

The Call of Abraham
It has been widely taught by learned men, that discrepancies abound in Stephen’s speech before his martyrdom; it, therefore, calls for a few remarks.
It is well to remember, that Acts 6 tells us that Stephen was “full of the Holy Ghost  ... full of faith and power, did great wonders and miracles  ... they were not able to resist the wisdom and the spirit by which he spake  ... and they saw his face as it had been the face of an angel.” We ask, then, was he likely to have spoken with accuracy, or not?
Add to all this, that his faithful speech was before the Sanhedrim, who were well instructed in the history of the people of Israel, and especially as to Moses and Abraham, and the Pentateuch, so that any historical discrepancy would have at once been detected by them. But of such a thing there is not a trace.
Let us now look at some of the charges of modern philosophers. One is, that in Acts 7:4, we are taught concerning Abraham, that the death of his father was after the call, and not, as according to Genesis 11:32, before it. If the accounts of the call of Abraham be carefully examined, no such phrase can be found, as “the call,” because there was evidently more than one. Stephen speaks of the God of glory having appeared to Abraham when he was in Mesopotamia, before he dwelt in Charran, and said, “Get thee out of thy country, and from thy kindred, and come into the land which I shall show thee” (Acts 7:2-3). The effect of this was, that he left his country, and it may be, many of his kindred; but his father accompanied him, and Sarah, and Lot, and dwelt in Haran. There they remained long enough for souls to be gotten in Haran (Gen. 11:31; 12:5). Then it seems God called him to get out from his country, kindred, and father’s house, and come into a land that He would show him (Gen. 12:1). The effect of this call was, that he departed out of Haran. This is confirmed by Stephen saying, “from thence (Haran) when his father was dead, He (God) removed him into this land;” showing unquestionably that there was a second interposition on the part of Jehovah. Now, where is there discrepancy between the account in Genesis, and the testimony of Stephen? Nay, rather, is not the comparison of the two accounts a further testimony to the perfect accuracy of Scripture, and of both having divine authorship?
The Years of Moses
It is alleged that there is in Acts 7:23,30,36, the distinct mention of three periods of forty years, of. which only the last is mentioned in the Pentateuch. But if we turn to Exodus 7, we are told that “Moses was fourscore years old  ... . when they spake unto Pharaoh” (Ex. 7:7). Now these fourscore years of Moses exactly agree with Stephen’s account, that he was forty years old when it came into his heart to visit his brethren, the children of Israel, and was forty years after in Madian. This also fits in perfectly with Exodus 2:11,15, and Exodus 7:7. Stephen makes the age of Moses to be one hundred and twenty years altogether; and in Deuteronomy 31:2, and Deuteronomy 34:7, he is said to have died at the age of one hundred and twenty years. Now, where is there any discrepancy?
The Terror of Moses
We believe there is no remedy if men have not the fear of God before their eyes. Take another example, to show with what levity and trifling some of the learned in our day have published their views — some more of the last words of God’s faithful martyr, when full of the Holy Ghost, and wisdom, and faith. It is said, that “the terror of Moses at the bush spoken of in Acts 7:32, is not mentioned in Exodus 3:3.” It is quite true it is not mentioned in Exodus 3:3, but in Exodus 3:6 we find the words, “And Moses hid his face; for he was afraid to look upon God.” Could any testimony more clearly confirm the agreement of the martyr’s teaching with the prophet’s statement?
Saul’s Sight of the Lord Jesus
It is widely taught that there are contradictions in the two accounts of the conversion of Saul, in Acts 9:7 and Acts 22: 9. Let us carefully compare them. Both Saul and those who traveled with him are mentioned. Saul saw the Lord Jesus Christ, was blinded by the light, heard Him speak words to him personally, addressing him in the Hebrew tongue, calling him by name, and Saul replied. The men who were with him saw the light, were alarmed, (did not speak a word, saw no one, but heard a voice or sound. In Acts 9:7 we read. “The men which journeyed with him stood speechless, hearing a voice (or sound) but seeing no man.’’ In Acts 22:9, we read, “They that were with me saw indeed the light and were afraid; but they heard not the voice of Him that spake to me.” In these Scriptures there is surely no discrepancy.
(Continued and to be continued).

Rest

“Take My yoke upon you, and learn of Me, for I am meek and lowly in heart, and ye shat find rest unto your souls” (Matt. 11:29).
‘Tis the rest of a subject heart
That beareth Christ’s burden light,
Its joy doth seek in His footsteps meek,
And walks in His own smile bright.
Yea, the rest of a broken will,
In lowly subjection now,
That hath learned at last, in days now past
To its Father’s will to bow.
‘Tis the rest of a perfect love
In a restless heart made known;
The soul that lives in the rest He gives.
Will lean on that love alone.
Such a perfect, unchanging love,
Such a peaceful, blessed rest,
Divinely led, we pillow our head
Forever on Jesus’ breast.

Truths for Young Christians: To Young Men in Business

Chapter 21
TO YOUNG MEN IN BUSINESS
One great difference between serving Christ and manifesting Him is that the former is necessarily intermittent, the latter continuous.
Especially is this true of those in business, whose opportunities of service are but few.
During the hours of work I need hardly say that direct service to the Lord cannot find much place, the truest service to Him then being to work for our earthly master faithfully and well, to remember that the time we are in business is his and not ours. The “righteous Lord loveth righteousness.” and nothing pleases Him better than to see His child humbly, faithfully, steadily, and conscientiously giving all his energies during his master’s time to carrying out his wishes and furthering his interests.
“Render, therefore, unto, Caesar the things which are Caesar’s and unto God the things that are God’s,” is a word as true of time as of money, and that perfectly maintains the balance between what we owe to our earthly and heavenly masters.
None are more quick to perceive the value of a truly faithful servant than worldly men, and although in a consistent Christian they do not find that push and dash, and what, alas, is often called for, “business” deviations from the truth, they look for, and ought to find, a thorough hearty service, and a strict integrity.
Let us remember that we cannot think of, speak of, or do two things at a time. If I am posting a ledger, or casting up an account, I cannot at the same time be pondering the last address I heard, or thinking over some favorite scripture. If I do, most likely my ledger will be wrongly posted, and my account wrongly added.
I am sure we all need to remember this, lest we bring reproach upon the name of Christ. We cannot do many things in business that others can, and we are therefore bound in common uprightness to devote our whole business time the more strictly to what we are engaged in; and if there be a spare quarter of an hour, rather give it to our master than use it for ourselves, that our service to him may be as God’s grace ever is to us, “Good measure, pressed down, shaken together, and running over.”
Another matter of much importance is always to come to our work, especially at the beginning of the week, with all our faculties in good order.
Surely it is no little disgrace to the Lord if we are found, like so many others, fit for little or nothing for the first three or four hours we come to business, and although we may be worn out with working hard and late for the Lord, while our companions are exhausted with dissipation, the difference of the cause does not excuse the sameness of the effect in our master’s eves, or hold us blameless before God.
It cannot, of course, be supposed for a moment that working for the Lord, however hard, can have any such injurious effect upon the body as is brought on by dissipation; but those who have tried it will bear witness that it is impossible to bring the energies and faculties into the full use that is required by business in the present day, unless the body and mind have had sufficient rest.
In this, as in everything else, if the heart is right with the Lord we shall not go far wrong, either on the one hand, of selfishly using for ourselves the hours of leisure instead of rendering them as a thank offering to the Lord, or on the other, of overstepping the right limit, and bringing the precious name of Christ into dishonor through failing in our engagement with our earthly masters.
I would seek to write tenderly and carefully on this subject, for I know how great the harvest is, and how few the laborers; how near the Lord’s coming, and how infinite the value of one precious soul; and yet there is one thing that is more precious than all beside, and that is the precious name of Jesus.
While thankfully owning, therefore, that many hours on the Lord’s day can be fairly devoted to His service, even by those regularly employed in business, we must remember that we have to present ourselves fresh and ready for work at a certain time the next morning, and our labors must therefore cease at such an hour the night before as will insure that our “good” on the Lord’s day will not be “evil spoken of” on the Monday.
Having already seen that for those actively engaged in business, the opportunities of direct service to the Lord are but few, it is happy to know that there is a way in which we can continually glorify Him. Manifesting Christ involves no neglect of our master’s work, interferes with no daily occupations, and finds as honorable and appropriate a sphere in the counting-house or wareroom, as in the family circle or Christian gathering.
The believer who manifests Christ in business life, makes Him known to many whom his active service could not reach: for hundreds, who never think of listening to any word spoken about the Lord, cannot but mark and own His graces when brought under their notice in daily life by the unobtrusive walk of a true disciple.
But do we fully understand what is meant by “manifesting Christ”? Having believed in Him, He has given us Eternal Life, which indeed is Himself. (“Christ, who is our life.”) This life in us has an object outside ourselves, which is also Christ. So that Christ is in us all, as life, and is our all, as object, thus explaining Colossians 3:11. Just in proportion as the divine life in us is drawn out by its object, we become practically Christ-like, and so “manifest Him.” If we study the First Epistle of John, we shall find eternal life displays itself in two ways, “light and love.” or, in other words, as expressed in Christ (John 1:14), “grace and truth.” Observe in passing that the Gospel of John is the unfolding of eternal life in Christ, the Epistle, the unfolding of the same life in the believer. The power to manifest it is by the Spirit, which dwelt in Christ and now dwells in us, so that the very fruits of the new life are spoken of in Galatians as the “Fruit of the Spirit” (Gal. 5:22).
How, then, can this light and love be made to flow out from our daily lives? By having Christ truly before us as our object, and not ourselves in any shape or form; by having our hearts filled with His love, and our conscience with His light, we can alone practically exhibit both, and obey the apostle’s exhortation, “Walk in love,” “Walk as children of light” (Eph. 5:2,8). In business life light would show itself in honesty, strict integrity, truth, and practical righteousness; while love would flow out in longsuffering, gentleness, goodness, meekness, and patience. No one can doubt that these qualities are greatly called for and valued even by worldly men. Surely it is our place to exhibit them.
Nothing so arrests a man’s attention as when he unexpectedly finds (it may be in some trifling action) a man whose code of right and wrong is not framed on the accepted basis of “commercial morality;” but who brings the light of the throne of God into the office or the shop; or, one who, instead of the natural impatience, the hasty temper, the sharp reply so often met with, displays the meekness and the gentleness that was in Christ. The very incidents of business life afford so many opportunities for the display of the treasure that is in the earthen vessel. It is the blowing of the north wind that causes the spices to flow out.
And now how far are we each thus manifesting Christ? Is every young believer so upright in the details of business, so gentle and patient in his manner as to render these few words unnecessary? Do we never hear of those, whom we know as true Christians, engaged in doubtful transactions, or giving way to a hasty temper?
Although service as generally understood, means some definite work for the Lord; and used in this active sense throughout this article, there is no doubt but that “manifesting Christ,” is one way of serving Him. Many being unable in business, actively to labor for the Lord, feel themselves cut off from doing anything for Christ, being unaware that by thus manifesting Him, they can still serve Him.
I have spoken a little on the way of doing this, but do we fully estimate the importance and blessedness of thus showing forth the life of Christ in us.
The truth is, beloved friends, that if those in business continuously displayed that practical righteousness and love in which Christ walked, it would bring a glory to His name and be a testimony to the reality of Christianity such as has never yet been given, and that no infidel could withstand.
Beloved reader, seek in the sphere in which you move thus practically to exhibit Christ; seek it prayerfully, seek it earnestly, seek it continuously, and rest not content unless the light that is in you also shines out of you. The very purpose for which you are left down here, is to grow into the likeness of Christ. The lines may be unformed, the resemblance indistinct, but let there be that at least, even in daily business life, that will show the most careless around, “that you have been with Jesus.”
A sketch, however rough and imperfect, is worth a page of description, and an illustration of the principles of true grace and truth is worth many discourses on the subject, and is of very especial value when shown in a place otherwise inaccessible to anything divine. And although those that see it, may find no beauty in the dim outline, the eye of One is gazing into the crowded wareroom, the bustling shop, or the busy counting-house, to whom the slightest resemblance of the Beloved Son is ever dear; and the meek answer, or the truthful reply, that may only raise a laugh and scoff around, ascends as a sweet savor to the throne of God. Ponder over these things, beloved reader, and seek to live them and there will be fewer wasted hours to account for in the morning of the resurrection.

Not Heaven But Christ

Mr.- was a man of much intellectual vigor and many engaging qualities. He had a loving wife and several bright, beautiful children, but with all these joys he had one dreadful trial — he was blind from his birth.
An eminent French surgeon while in this country called upon him, and examining the blind man with much interest and care, said to him: “Your blindness is wholly artificial; your eves are naturally good, if I had operated upon them twenty years ago I think I could have given you sight. I may possibly be able to do so now, though it will cause you much pain.”
“I can bear that,” was the reply. “if you can but enable me to see.”
The surgeon operated upon him, and was gradually successful; first there were faint glimmerings of light, then more distinct vision. The blind father was handed a rose; he had smelt one before, but had never seen one; then he looked upon the face of his wife, who had been so true and faithful to him; and then his children were brought, whom he had so often fondled, and whose charming prattle had so frequently fallen upon his ears.
He then exclaimed: “O, why I have seen all of these before inquiring for the man by whose skill I have been enabled to behold them! Show me the doctor.” And when he was pointed out to hint, he embraced him, with tears of gratitude and joy.
So, when we reach heaven, and with unclouded eves look upon its glories, we shall not be content with a view of these. No, we shall say, “WHERE IS CHRIST? He to whom I am indebted for bringing me to this glorious place and enjoying its beauties; show Him to me, that with all my soul I may adore and praise Him through endless ages.”

Scripture Study: Matthew 16, Part 1

Matthew 16
Matthew 16:1. “The Pharisees also with the Sadducees came, and tempting, desired Him that He would show them a sign from heaven,” plainly as John 1:5 says, “The light shineth in darkness; and the darkness comprehended it not.” Though directly opposed to each other, the Pharisees and Sadducees unite in opposing the Lord, and the truth. At another time Herod and Pilate, at enmity with each other, join hands over Christ (Luke 23:12). Here self-righteousness and infidelity join hypocritically and ask a sign from heaven. How perverse the will of man is, and that after all the display of His goodness and grace that has been done before their eyes, in healing the sick and feeding the hungry. They could read the face of the sky, and see when fine or foul weather was coming. Why did they not read, in the Lord’s presence and miracles, the promise of blessing to the needy? or why did they not read, in the condition of Israel at that moment, that an awful storm of judgment must soon fall upon them? Surely it was because their sin had blinded them, and the Lord tells them that they are a wicked and adulterous generation, for with all their profession of defending the law, they were hypocrites; serving themselves, mammon worshippers, covering up their awful sins. No sign could be given to them but the sign of the prophet Jonas. “And He left them and departed.” The prophet Jonas was typical of death and resurrection. And at this point they lose the Christ.
Matthew 16:5-12 manifest the slowness of the disciples to enter into the thoughts of God. Living in things seen and temporal, they need explanations to get their minds to see the spiritual meaning of what was in the Lord’s words, “Beware of the leaven of the Pharisees and Sadducees.” The leaven of the Pharisees is self-righteous hypocrisy. They and we are in danger of it. If we know God, we know that in ourselves — in our flesh — good does not dwell; we have a heart that is, by nature, deceitful above all things and desperately wicked. The leaven of the Sadducees was infidelity — free-thinking — really unbelief of God and His Word. We cannot trust our own hearts. He that trusts his own heart is a fool (Prov. 28:26).We must believe God, His Word is true, His power is almighty. “Casting down imaginations, and every high thing that exalteth itself against the knowledge of God, and bringing into captivity every thought to the obedience of Christ” (2 Cor. 10:5). “The shield of faith” — I believe God — quenches all the fiery darts of the wicked one (Eph. 6:16).
What a rebuke He gave them in reminding them of His feeding five thousand with the five loaves, and the four thousand with the seven loaves, and how many baskets of bread were left. Why should they think Him anxious about bread? He was seeking the good of their souls, to protect them from the evil doctrines that they failed to notice.
Matthew 16:13. A third form of unbelief appears when He asked His disciples, “Whom do men say that I, the Son of Man, am?” Some say one thing, some another. Why are such differences seen? Simply because these people are indifferent. They have not felt the need of an object for their hearts; they have not had their consciences stirred; it is with them just a matter of opinion. The mind may work, but the conscience is not exercised. They do not feel the need of a Savior, and other things interest them as much, or perhaps more. Satan blinds the eyes with the things of time and sense (2 Cor. 4:4).
Matthew 16:15. “But whom do ye say that I am?” And Simon Peter answered and said, ‘‘Thou art the Christ, the Son of the living God.”
This home question found an answer in Simon Peter’s soul. He had felt his need and had found in the person of the Lord Jesus that which met it; as he says at another time (John 6:68), “Lord, to whom shall we go? Thou hast the words of eternal life. And we believe and are sure that Thou art that Christ, the Son of the living God.” He alone can satisfy the awakened conscience and heart. But how did Simon Peter know this one? It was a distinct revelation the Father gave him. Jesus answered, “Blessed art thou, Simon Bar-jona: for flesh and blood hath not revealed it unto thee, but My Father which is in heaven.” It was not education, nor imagination, but revelation. And now the Lord speaks according to this revealed character, and gives him his new name. And I say also unto thee, That thou art Peter (a stone) and upon this rock I will build My church (assembly) and the gates of hell (hades) shall not prevail against it.” Here the Lord tells us of what was to be recognized of Him on earth when Israel was set aside. “I will build,” expresses intention. It began at Pentecost by the Holy Spirit coming down and uniting the disciples into one body. It was Christ’s assembly, and built of living stones (1 Peter 2:5) on the Rock-Christ, the Son of the living God; the power of death — the gates of hades — could not prevail against it. Many individuals have been martyred for Christ, but the church can never die. The Lord will come and take it home to share His heavenly glory: this is its destiny.
There were men of faith — saints all down through the ages of man’s history — but not the church of God. This is entirely new since Pentecost, after Christ was glorified at Pentecost, the Holy Spirit came upon the disciples (Acts 2) baptizing them into one body, uniting them together, and uniting them to Christ, the glorified Head in heaven (1 Cor. 12:13). Peter speaks of it as “living stones, built up a spiritual house” (1 Peter 2:5), but Paul unfolds the truth of the mystery of Christ and the church, His body and His bride, for to Paul alone was it given to do so (Eph. 3:4-10). This verse in Matthew 16:18 is the first time the church is mentioned in Scripture, and it is mentioned as a then future thing.
(To be continued.)

Hezekiah: Brief Lessons on Church Truth, Part 6

A Brief Outline of Lectures on Hezekiah
Read carefully 2 Chronicles 31.
This just brings us to the question, what is
THE PORTER’S SITUATION?
Observe, in this business of the fruits, each name is known to the Lord, and registered; enough for the servant of the Lord. “And Kore, the son of Imnah the Levite, THE PORTER toward the east, was over the free-will offerings of God, to distribute the Oblations of the Lord, and the most holy things” (2 Chron. 31:14). Let us examine the Scriptures on this deeply interesting subject — the trusteeship and responsibilities of the porter. The first thought generally as to a porter in any establishment is, that it is the lowest situation. And it is true here also; if any man would serve the Lord, he must take the lowest place.
Will you turn to, and read 1 Chrononicles 9:17-32? They were keepers of the gates of the tabernacle, keepers of the entry, porters of the door of the tabernacle, porters in the gates. And mark, wherever you find the word “office,” the margin reads, “trust.” We shall find this is not truth for officials, but for every child of God, as a trustee of Christ.
What is the door now but Christ? Their first service was to keep the door; and have we not now to guard the door; and, not only seek to keep Satan and men from closing the doors again — but maintain the gospel of Jesus, the way into the holiest? Mark, they were placed by wards. In four quarters were the porters — toward the east, west, north, and south. O, that each beloved servant of the Lord may know his appointment of the Lord! Now we find four chief porters in their trust, “over the chambers and treasuries of the house of God,” and their place is to lodge near the house of the Lord. And their charge, or trust, was to open these chambers of treasures of the Lord every morning. And some have charge of the ministering, vessels — the fine flour, and the wine, and the oil, and the frankincense, and the spices. And some in their trust over the things made in the pans. And some over the shewbread. O, my soul, dwell on each of these types of Christ, and then say, Hast thou ever understood the porter’s trust — all the treasures of the house of the Lord? The opening of those treasures every morning. The fine flour — the perfection of Christ in His blessed humanity. The wine — the joy of heart in God. The oil-anointed with the Holy Ghost. The frankincense and spices — the infinite preciousness of Christ. The meat-offering in the pans the Person and walk of Christ, as a sweet savor to God. The shewbread-Israel covered with Christ. The frankincense, and borne before God in divine righteousness. Now also fulfilled in the church, as presented in Christ, in all the sweet savor of His person. One Spirit with the Lord. All these glories and perfections of Christ committed in trust to the porter.
Still more, turn to 1 Chronicles 15:18-24. Will you notice Obed-edom and Jeiel Now it is highest praise with harps, in the Sheminith, or eighth, to excel  ... . The worship of the eighth, or resurrection  ... . I heard harpers harping with their harps. What a trust is this, thus to worship in Spirit and in truth. Lowest, as to self, is the porter’s situation — highest in Christ, with songs of praise. And these same porters are keepers of the ark. And is not that ark Christ? entrusted to make known the very heart of Christ? As He lived on account of the Father, surely the porter is called to live on account of Christ.
Now read 1 Chronicles 26:1-19. Strong men do these porters need to be. Such were the sons of Obed-edom — “able men for strength for the service, three score and two” (1 Chron. 26:8). These porters were placed in their wards by lot, the then way of appointment for every gate. Now the Holy Ghost appoints to the porters every man his work (1 Cor. 12). Some have service in the gatherings, as it says, “To Obed-edom, southward, and to his sons the house of Asuppim” (margin, “gatherings”). Others have service in the causeway going up, ward against ward. Eastward, six Levites; northward, four a day; southward, four a day; and toward Asuppim, two and two. At Parbar, westward, four at the causeway, and two at Parbar. So now each porter has his appointed place and service. Some in the gatherings in fellowship, two and two; others in the open air, the causeway going up well for each one to know his own appointed Parbar. That service which is of the Spirit will be going up, waiting for the coming of the Lord. That which is of the flesh will be going down to the world.
In 2 Chronicles 23, we find the porters are the royal guards of the hidden king. Here they are divided into thirds. “A third part of you entering on the sabbath of the priests and Levites, shall be porters of the doors. And a third part at the king’s house; and a third part at the gate of the foundation.” What a trust here again during these the days of the hidden King, whom the heavens must receive until the times of restitution! To guard the true rest of the saints in Christ; to guard the person of the hidden King of glory; to guard foundation-truth. “And he set the porters at the gates of the house of the Lord: that none which was unclean in anything should enter in” (2 Chron. 23:19). This is a great trust, and questioned by many. Nay, many do practically deny this, by maintaining that contact with those that hold false doctrine does not defile.
Let us now return to 2 Chronicles 31:14. This verse is the key to the whole subject of the porter’s trusteeship and responsibilities. Whatever may be the precious treasures committed in trust to the porters, it is that they may distribute the oblations of the Lord, and the most holy things. Yes, my brother, if you do not know what to do with the heaps upon heaps of tracts, and publications, and precious truths, given to us by the Holy Ghost, distribute them. Does the Lord thus commit to your trust some fine flour, wine, or oil, or frankincense — some unfoldings of the preciousness of Christ Ah, it is not merely for your own eating, important as this is — no, but distribute it to others. But I think I hear a porter saying, I have no gift; I might, perhaps, tell of Jesus to an infant class in a Sunday-school. So it was with these porters in their trust, to give to their brethren by courses, as well to the great as to the small: beside their genealogy of males, from three year old and upward. Thus were they to give to everyone their daily portions. Yes; if you can only speak to one of the Lord’s little ones — three years old and upward. Let Christians thus love to distribute. Did not Jesus say, “Feed My sheep,” “Feed My lambs”? Do not say this belongs to an official class. All believers are priests to worship, and Levites, porters, to serve. I saw the arrival of the prince at St. Pancras station; not a porter was there but would have felt it a high honor to carry a parcel for him. Will you not rejoice to be a parcel-carrier for Christ? To carry, to distribute the precious truth He has committed? Surely the precious treasures of Christ, committed to His saints in these last days, are for the whole redeemed church of God.
Is not every Christian responsible to distribute according to the grace given to him? Is not this the principle of Romans 12:4 according as God hath dealt to every man the measure of faith? “For as we have many members in one body, and all members have not the same office: so we, being many, are one body in Christ, and every one members one of another.” Then follows a list of different gifts, but each one responsible to use that he hath. So, in Peter, “As every man hath received the gift, even so minister the same one to another, as good stewards of the manifold grace of God” (1 Peter 4:10). Such, then, was the porter’s trust, “For in their set office they sanctified themselves in holiness” (2 Chron. 31:18). “And thus did Hezekiah throughout all Judah, and wrought that which was good, and right, and truth before the Lord his God” (2 Chron. 31:20). And whatever he began in the service of the Lord, he did it with all his heart. Is He not the same God now — can He not lead His children now thus to do that which is good, and right, and truth before the Lord? O, that we may have faith in God, to seek only thus to do His will, and to do it with all our hearts!
(Continued)

The Coming and Reign of Our Lord Jesus Christ: The Age to Come, or the Millennium, Part 5

There will not be any ark, as in the temple of old; for the prophet Jeremiah tells us, in Jeremiah 3:16-17, that “in those days, saith the Lord, they shall say no more, The ark of the covenant of the Lord; neither shall it come to mind, neither shall they remember it, neither shall they visit it, neither shall that be done any more.” (margin, be magnified). “At that time they shall call Jerusalem the throne of the Lord; and all the nations shall be gathered unto it, to the name of the Lord, to Jerusalem.”
“In those days it shall come to pass, that ten men shall take hold, out of all languages of the rations, even shall take hold of the skirt of him that is a Jew, saying, We will go with you for we have heard that God is with you” (Zech. 8:23). Idolatry, which will have been forced upon the nation (and readily accepted by the mass), and also throughout the domain of the beast, will be utterly abolished by the Lord when He comes in judgment. For “in that day a man shall cast his idols of silver, and his idols of gold. which they made each one for himself to worship. to the moles, an to the bats” (Isa. 2:12-22).
The shekinah glory, the glory of the God of Israel, will come from the way of the east, where it was last seen when retreating from the earth, after the captivity of Judah. (Ezek. 11:22-23). It will then fill the house or temple (Ezek. 43:1-5), which is called “the place of My throne, and the place of the soles of My feet, where I will dwell in the midst of the children of Israel forever” (Ezek. 43:7). “The Lord is great in Zion: and He is high above all the people. Let them praise Thy great and terrible name; for it is holy. The king’s strength also loveth judgment; Thou dost establish equity, Thou executed judgment and righteousness in Jacob. Exalt ye the Lord our God, and worship at His footstool; for He is holy” (Psa. 99:2-5).
There will also be a prince at the head of the house of Israel, mentioned many times from Ezekiel 44-46. He must in no way be confounded with the Lord, as he is the prince of Israel, ruling as His vicegerent in the land, and offering sacrifices in connection with the temple worship; whereas Christ will reign with His saints over the earth.
The land, Israel’s ancient inheritance, now almost a wilderness, and under the dominion of the Gentiles, will then be redivided after a new order, each tribe having its measured portion, and a holy oblation foursquare in the center, divided among the priests and the Levites, and part for the city and suburbs. On either side of this, the residue will be for the prince. (Ezek. 48; 45:7).
Then as to the city of Jerusalem itself in that day, its glory and magnificence, as described in the prophets and the Psalms, will far surpass anything that the world has ever seen, much as men boast of the wonders they have wrought. Neither the ancient cities of Nineveh and Babylon, with all their glories, nor the modern cities of London and Paris, bear any comparison.
From Zechariah. 14:10 we find that “all the land shall be turned as a plain, from Geba (north) to Rimmon, south of Jerusalem: and it (that is, the city) shall be lifted up, and inhabited in her place.” The Mount of Olives is described as cleaving asunder when touched by the feet of the Lord at His appearing. Important physical changes will take place, thus affording a most magnificent plateau for the enlargement of the city during His reign.
Well may the Psalmist sing, “Beautiful for situation, the joy of the whole earth, is mount Zion, on the sides of the north, the city of the great king” (Psa. 48:2). And again, “Walk about Zion, and go round about her: tell the towers thereof. Mark ye well her bulwarks, consider her palaces; that ye may tell it to the generation following” (Psa. 48:12-13). And again, “Out of Zion, the perfection of beauty, God hath shined” (Psa. 50:2). “Glorious things are spoken of thee, O city of God. Selah” (Psa. 87:3). “Put on thy beautiful garments, O Jerusalem, the holy city” (Isa. 52:1).” Thou shalt call thy walls Salvation, and thy gates Praise (Isa. 60:18). “For Zion’s sake will I not hold my peace, and for Jerusalem’s sake I will not rest, until the righteousness thereof go forth as brightness, and the salvation thereof as a lamp that burneth. And the Gentiles shall see thy righteousness, and all kings thy glory; and thou shalt be called by a new name, which the mouth of the Lord shall name. Thou shalt also be a crown of glory in the hand of the Lord, and a royal diadem in the hand of thy God. Thou shalt no more be termed Forsaken: neither shall thy land any more be termed Desolate; but thou shalt be called Hephzibah (that is, my delight is in her), and thy land Beulah (that is, married): for the Lord delighteth in thee, and thy land shall be married” (Isa. 62:1-4).
“They shall call thee, The city of the Lord, The Zion of the Holy One of Israel” (Isa. 60:14).
These are a few of the many passages indicating the unprecedented glory and splendor of the city of the Lord of hosts, in the day of Israel’s blessing, and the earth’s jubilee. But this is not all; for not only will it be a praise in the earth for its beauty, but the center of government and blessing for the earth under the Lord’s righteous rule. “And it shall come to pass in the last days, that the mountain of the Lord’s house shall be established in the top of the mountains, and shall be exalted above the hills; and all nations shall flow unto it. And many people shall go and say, Come ye, and let us go up to the mountain of the Lord, to the house of the God of Jacob: and He will teach us of His ways, and we will walk in His paths: for out of Zion shall go forth the law, and the word of the Lord from Jerusalem” (Isa. 2:2-3). “Because of Thy temple at Jerusalem shall kings bring presents unto Thee” (Psa. 68:29). “Yea, all kings shall fall down before Him; all nations shall serve Him” (Psa. 72:11). It is then that the government shall be upon His shoulder, and He shall reign as the true Solomon, the Prince of Peace, from the river to the ends of the earth (Isa. 9:6-7).
“He shall judge among the nations, and shall rebuke many people; and they shall beat their swords into plowshares, and their spears into pruninghooks: nation shall not lift up sword against nation, neither shall they learn war any more” (Isa. 2:4; Psa. 46:9; 68:30; Mic. 4:3). The Mighty God shall judge the earth, and take war from it. Instead of nation arming and warring against nation as now, peace shall flow as a river; a king shall reign in righteousness, “the Prince of Peace,” and princes with Him shall rule in judgment (Isa. 33:1).
(Continued and to be continued).

Correspondence: Earthly Blessing and Earthly Inhabitation

Ques. 83 What is the meaning of Genesis 22:17; Rev. 21:24? Do we not get the promise of earthly blessing to Israel and the nations in the eternal state? Who are to inhabit the new earth? N. S. C.
Ans. Genesis 22:17 is a promise to Abraham and to his seed, which is Christ (Gal. 3:16), that in Him all families would be blessed (Eph. 3:15).
Revelation 21:9-22:5 is a description of the church in Millenial glory, reigning with Christ over the world, Revelation 21:24-26 is the nations paying tribute to Christ through the church. Revelation 22:2, they receive healing through her.
Israel and the nations posses the earth then (Psa. 72).
In the eternal state, righteousness dwells (2 Peter 3:13).
In the description of Eternity (1 Cor. 15:28; Rev. 21:1-8) New Heavens, New Earth and the Lake of Fire are mentioned. The heavenly families will still possess the heavens. The holy city New Jerusalem, the Bride adorned for her Husband — is seen, distinct from all. She is also the Tabernacle of God. He tabernacles with men, He is their God. Israel and the nations are seen no more. They are now men and God dwells with them in eternal bliss.

Inspiration of the Scriptures: Holding Jesus by the Feet

Holding Jesus by the Feet
It is often said that the statement that “the women held Jesus by the feet and worshiped him,” is a palpable contradiction of His command to Mary not to touch Him.
In Matthew 28:9, we read that as “they went (according to the angel’s word) to tell His disciples, behold Jesus met them, saying, All hail. And they came and held Him by the feet and worshiped Him.”
In John 20:17, when Mary seems ready to approach the Lord; in her accustomed manner, “Jesus said unto her, Touch Me not; for I am not yet ascended to My Father: but go to My brethren, and say unto them, I ascend unto My Father, and your Father; and to My God and your God.”
It seems to us impossible that any one can see the beauty of these two scriptures, and the entire absence of contradiction, unless the different lines of truth and the relationships of which each evangelist treats are discerned. This, too, resolves many other difficulties. The moral perfection of Scripture is then most striking. In Matthew we have, from first to last, facts which are for the most part narrated as having a Jewish bearing. Hence we find much of that kind of instruction which is not found in the other gospels. When our Lord spake of His death and resurrection, He added, “But after I am risen again, I will go before you into Galilee” (Matt. 26:32). The Messiah having been rejected by the Jews and Jerusalem, He now intimates that when raised from among the dead, He will still recognize the “poor of the flock” in Galilee, who will thus be a sample of the future remnant spoken of by the prophets that will be brought into their promised blessing at His appearing and kingdom. This is doubtless why we have no ascension in Matthew. This is also why the angel sent the message to the disciples to meet Jesus in Galilee, which was afterward confirmed by our Lord, and acted on by the disciples. (Matt. 28:5, 10, 16). It is here that Jesus as Messiah, having died for that “nation,” is now able, on the ground of His work, to recognize relationship with the faithful residue.
Hence, in keeping with this, the women are allowed to hold Him by the feet and worship Him; for the Jewish thought is to have Messiah bodily with them on earth. This will be manifested in due time.
But John’s is a different line of things. All through his gospel we have the Father and the Son. Jesus is here looked at as having taught believers that they were loved of the Father, given to Him by the Father, and that their destiny was the Father’s house. When He went back to the Father, He spake of sending the Holy Spirit to be with us and in us during all the time of His absence; and that He would come again and receive us unto Himself, that where He is there we might be also. Therefore, in John 20:17, we have the new relationships of being God’s children, and Christ’s brethren, announced, not in connection with a Messiah seen on earth, but with the Lord of glory, not here but in heaven. It is in principle Christianity and not Judaism. Mary therefore was forbidden to “touch” Him, but was sent to tell others of His sphere henceforth being in ascension and not on earth. They were to know Him, serve Him, and honor Him as gone back to the Father. The Christian’s relationships therefore are heavenly, and forever. Precious message! “Go to My brethren, and say unto them, I ascend unto My Father, and your Father; and to My God, and your God.” O, the untold blessedness of these new relationships, founded on Christ’s work of eternal redemption!
Sin and Transgression
Though the books of Moses and the gospels have been the portions of the sacred writings against which the shafts of skepticism have been more generally thrust, yet the epistles have not wholly escaped their censure. We only select one instance, out of many, from a published pamphlet now before us.
It is alleged as a proof of contradiction, that in Romans 2:12, it is said, “For as many as have sinned without law, shall also perish without law;” and in Romans 4:15, “Where no law is, there is no transgression.” In the accuser’s mind sinning and transgression are here the same thought, hence to perish without law, and yet for there to be no transgression without law, to him is a contradiction; but it is not so. We are told that “all have sinned,” from Adam downwards; but the people of Israel were also transgressors, because they went across God’s positive commands. Adam also transgressed in doing what God prohibited (Rom. 5:12,14).
The accuracy of Romans 2:12 is seen in that not having been under the law, they will not be judged by it: but having a conscience and the knowledge of good and evil since “sin entered into the world,” they will be judged on other grounds. Such may be referred to in Romans 1:18 and following verses. The accuracy of Romans 4:15, is seen in the holiness of the principle, that if there had been no law given, there could be nothing to be disobeyed, no transgression. Speaking generally, then, all Gentiles and Jews are sinners by nature, and practice — all are “under sin,” “have sinned, and come short of the glory of God”; but the children of Israel, who willingly put themselves under law have been proved to be transgressors, and when Jews and Gentiles are classed together another word is used, the word “offenses,” for all are offenders, though not all transgressors, inasmuch as Gentiles have not the law — “Who was delivered for our offenses, and was raised again for our justification” (Rom. 4:25).
It may be well to add, that the word “sin” is used about thirty times in only two chapters in Romans, Romans 6-7: It is called “the old man,” “the flesh,” which the believer is not in as to his standing, though it is still in him but to faith he is righteously delivered from, because “our old man is crucified” and “dead with Christ.” “Sins” are forgiven, because they have been borne, suffered for, and atoned for by Christ, whose blood “was shed for many, for the remission of sins.” We read, therefore, of remission of sins, and of our old man (the nature that did the sins) being crucified with Christ; and these are important distinctions. We find “sin,” “sins,” “offenses,” and “transgression,” used in this epistle and elsewhere with the greatest precision.

Thank You for Telling Me

“Thank you for telling me.” Such was the grateful remark of a lady, lying very ill, to a friend beside her. “Thank you for telling me,” she said, with a smile; “it was just what I wanted to know.”
What was the good news, do you think, that had just been communicated to her, and that called forth such a speech? Was it that some dreaded medicine need no longer be taken? Had the doctor promised relief to her suffering? or had he foretold a quick recovery to health and strength? Was it some intimation of this kind that drew forth those words, amidst her suffering, “Thank you for telling me”?
No, nothing of the sort: The lady we write of was dangerously ill, and her friend had just announced to her the fact that she could last only a short while longer, and that very soon she would pass out of time into eternity.
Reader, can you picture yourself for one moment in such a position? Supposing you were told that in a few hours at the most you would have to face death. Would you be prepared for the news? Would you feel perfectly sure that having had to do with Jesus in this world as Savior, there would be no possibility of meeting Him as judge hereafter? For unless you were certain in your own soul that the question of your sins had been eternally settled by the Lord Jesus on the cross; and that they had been eternally forgiven and forgotten by God, I know you could not face death with other than terror and fear.
Death is a solemn reality, though to the believer the sting is gone. The Lord Jesus came to deliver those who, through fear of death, were all their lifetime subject to bondage. Hence to this lady the prospect of death was a very welcome one. She knew that, for her, it meant passing from this scene of sin and sorrow — ay! and of dishonor to Christ too — to be forever in the presence of the Lord Jesus, which is indeed “far better”. Not a fear, not a doubt, once crossed her mind or harassed her soul; but so glad was she to hear that the time of her departure was at hand, that she could reply, “Thank you for telling me; it is just what I wanted to know.”
Perhaps you fancy that she must have been same saintly being, whose life of religious devotedness made her hope to merit the favor of heaven? But could her lips — now silent in death — speak again, they would tell a very different tale. She would say what a marvel it was that the grace of God should ever have picked her up. And nothing but the “exceeding riches of His grace” would have done so. But God is rich in mercy, and delights in plucking brands from the burning. She said herself one day, “He knew I was worthless before He took me.” Yes, indeed, Jesus knew it all.
“He saw us ruined by the fall,
He loved us notwithstanding all.”
Such was “the grace of our Lord Jesus Christ, that though He was rich, yet for our sakes He became poor, that we through His poverty might be rich.” Although worthless in ourselves, fallen in our nature, sinful in our practice, yet He has set His love upon us; and having died for us, to bring us to God, He now owns us as part of that “pearl of great price” which He sold all that He had to possess.
Can you wonder that she longed to see the One who had done so much for her? She had found that “Thy lovingkindness is BETTER than life” (Psa. 63:3), and undoubted peace tilled her soul at the prospect of soon being in His presence.
And how is it with you, dear reader? Would such tidings come to you as “the very thing you wanted to know”? Or is the idea of death intolerable to you? Do you shudder at the sight of a funeral? Do you tremble to walk through a cemetery? If so, let me entreat you to delay no longer in seeking that Savior, who has Himself been into death, and broken all its power; and knowing Him, you will be able to triumph in His blessed name, “which quells the power of death.”
“Lord, while our souls in faith repose
Upon Thy precious blood,
Peace like an even river flows,.
And mercy like a flood.”

Truths for Young Christians: On a Start in Life (for Young Women)

Chapter 22
ON A START IN LIFE
FOR YOUNG WOMEN
A start in life must come sooner or later to young women, as well as to young men. Like the latter, they must say farewell to their quiet girlhood in entering upon some occupation — it may be as servant, it may be as seamstress, or as a clerk in some of the higher branches of women’s work, in the postal, telegraph, and other services; or it may be as a governess, or music teacher. In entering on any of these new positions, a great and important step is taken, entirely analogous to that on which we wrote a few words in our last chapters. To such who have thus, in some way or other, to leave their homes to earn their daily bread, what we said there is perfectly applicable.
Show Your Colors
A young girl, going out from under her mother’s care to service, or becoming an apprentice, or a clerk, is surrounded with temptations. She will, in most cases, find that those around her are worldly, giddy, and have a general dislike to “religion.” If she would live to God’s glory, she must begin by showing her colors, and the sooner she lets it be plainly known amongst her fellow-servants, or fellow workers, that she belongs to Christ, and means to stand up for Him, the sooner she will be understood. It is in vain to try and go on with the world, and to follow Christ as well. Once she has taken her stand, the world — represented by her godless companions — is against her, and she must understand this. Of course, in her ways and manners she must be additionally obliging and kind, doing others a good turn whenever it lies in her power. But she must be prepared for every little inconsistency to be shown up in the most glaring light.
The very fact of her being in such company will help to keep her straight, for if she tries to copy any worldly ways, her companions will be the first to point out the inconsistency with some such remark as, “She a Christian, indeed!” These taunts are felt, because in a measure they are deserved, and the young Christian sees that the world is sharp enough to discover when she does not adorn her profession.
By Practical Christianity
It is evident, therefore, that both for her own sake and for her Master’s the more quiet and consistent her walk, the happier her life. Respecting her work, it must be well and thoroughly done. No worldly master or mistress can understand or respect a Christianity, which consists in reading religious books, instead of doing one’s work, or in always wishing to go out to some meeting, instead of keeping the house straight. Our work must be done cheerfully, and be “Good measure, pressed down, shaken together, and running over.” A Christianity that makes one help a slower worker, when one has done one’s own task, that does not draw the line at the exact amount of work paid for, that seeks to help and assist generally, that takes an interest in what is being done, is readily understood and appreciated. O! seek, then, my dear fellow-believers, thus to adorn the gospel of God your Savior in all things. Not only in being in your accustomed seat on the Sunday, but in little acts of kindness all through the week, done for Christ’s sake — ever seeking opportunities of doing good. If you have helped some poor girl to get through her heavy task, and taken half the load on your own shoulders, you have won the gratitude of your sister, and may have opened a door in her heart to some loving words about the One who took your burden on His shoulders. In your dress, too, surely there is great scope for adorning the gospel of Christ, in a way those of your own sex especially, are ready to observe. I do not mean in dressing like a nun, or sister of mercy, but like a Christian, having a desire to please Christ even in your outward appearance.
A Good Example Encourages Others
One reward, of standing up for Christ, is that it frequently is the means of encouraging some more timid sister to show her colors, and you will often discover some hidden member of Christ in this manner. It is a wonderful help, when there are two in a business or situation, that love the Lord. It doubles their strength and halves their trials. If, therefore, you find you are really alone, resolve that by God’s grace you will not remain alone, and earnestly seek to win some soul for Him from amongst your companions.
Marriage
Besides all these ways, however, in which you make a start in life, there is another and a more important way still, and that is by marriage. To young men marriage seldom comes as a beginning in life, for, generally, they have been out in the world some years before. But to girls, and especially to those who are not compelled to labor for a livelihood, marriage is often the portal through which they are introduced, from the quiet seclusion of home, into the vast world without. Many children of God have, from time to time, borne witness as to the vast importance, for good or evil, of this momentous step. It has been shown by instances drawn from real life, and by the direct Word of God, how this union to be blest, must be in the Lord (that is, both husband and wife children of God), and of the Lord (that is, both naturally and spiritually suited to each other, and His guidance sought in the matter). We do not now allude further to this, save again to point out that more young Christians are wrecked, and the fair promise of their young lives blighted, by hasty and ill-assorted marriages, than by anything else. It is probable that most, who read these lines, are yet unmarried; to such we would earnestly say, above all things honor God in this step. Let no inclination, no apparent worldly advantage, lead you to overlook the fact that as surely as you are God’s child, and as surely as He is your Father, so surely as you sow you shall reap; and if you, with your eyes open, disobey Him to please yourself, you must inevitably suffer deeply for it whereas if you seek in this truly to glorify Him, He will uphold you.
How to Act in Married Life
We will suppose, however, that you have taken the step, and that no objection is to be made to your marriage, there still remains the question, “How are you to act in your new relationship?” In the first place, never let the new scenes and occupations interfere with the old duties — daily private reading, and prayer. This is the sheet-anchor of your soul, and if you have already experienced the blessing of it in your girlhood, it is worse than folly to neglect it now. Next, as in business, so here — it is the first step which is all-important. Let is be plainly understood at the outset by your new connections and friends that you are a believer, by your servants that they have to do with a Christian mistress. Lastly, make your start in life with a definite object before you for attainment, and that is, to glorify God in the new sphere in which He has placed you, as a wife — a mistress — a friend — and a mother. Let nothing obscure this object, but let it quietly underlie all your actions, and you will be blessed in all your relations. Not that such a steady course is easy. You will have to strive through many cross-currents, especially when your interests, or those of your children, seem to point one way, and God’s glory another. But if it is the constant habit of your life to know and feel that this is your object, you will be greatly helped at such times, and, by God’s grace, ever gain the victory.

Scripture Study: Matthew 16, Part 2

Matthew 16
Matthew 16:19. “And I will give unto thee the keys of the kingdom of heaven: and whatsoever thou shalt bind on earth shall be bound in heaven: and whatsoever thou shalt loose on earth shall be loosed in heaven.” Now this is not the church; there are no keys for it, nor do men build with keys, nor are there any keys for heaven. The present aspect of the kingdom is the place of profession of Christ’s name on earth, it is therefore a kingdom where the King is absent in heaven, so it is called the kingdom of heaven; and the keys given to Peter, was Christ’s authority for Peter to act for Him in opening the door for the Jewish converts in Acts 2, and for the Gentile converts in Acts 10: after that the door is open to both Jew and Gentile. It was Peter that gave the last offer to the nation of the Jews in Acts 3. This was in answer to Christ’s prayer on the cross, “Father, forgive them; for they know not what they do,” but this offer closed with the murder of Stephen, fulfilling Luke 19:14. It was Peter also that bound the first sin in the assembly on Ananias and Sapphira so that they were disciplined by the Lord and carried out dead. (Discipline is not eternal judgment, 1 Cor. 11:32). It was Peter, with John, who prayed that the Samaritan converts might receive the Holy Spirit, and thus the feud was removed between Jews and Samaritans, and the Lord preserved unity. It was Peter who pronounced Simon the sorcerer, not a true child of God at all. All this was clearly apostolic authority, which the Lord conferred on Peter. There were other apostles also to whom the Lord gave authority, but this was specially Peter’s. There is no apostolic succession in the Word of God. They gave us the Word of God, but those who came after them were grievous wolves. (Acts 20:29). The Word of God has been ever since our only authority.
Matthew 16:20. “Then charged He His disciples that they should tell no man that He was the Christ.” That testimony is over. He shows to them that He must go into Jerusalem, and suffer many things of the elders and chief priests and scribes, and be killed, and be raised again the third day. Ah, how that touches them; and, favored Peter, how will he take it?
That means loss to them, and a path of suffering, instead of a great and glorious place, and Peter lets nature guide him instead of the thoughts of God. No doubt, it was kindness, but it opposed the cross, and without the cross we could not share the glory. And Peter said, “Be it far from Thee, Lord: this shall not be unto Thee.” The Lord turned and said unto Peter, “Get thee behind Me, Satan: thou art an offense unto Me: for thou savorest not the things that be of God, but those that be of men.” Peter was not an enemy of Christ. He was a true disciple, but led astray by his own thoughts, he acts as an enemy of the cross of Christ. Alas! through unwatchfulness, how often this happens with us now! Peter, privileged to receive revelations from the Father in heaven, in the same chapter is called Satan, by the Lord. The old nature, even in an apostle, is no better than in any of us. If Paul was caught up to the third heavens, and he was, he must wear a thorn in the flesh ever after to keep him humble. (2 Cor. 12). How closely do we need to cleave to Christ, and not trust our own hearts for a moment, or we make some mistake. But it does not alter the love and grace of Christ. He just goes on to make the lesson good in their hearts, saying, “If any man will come after Me, let him deny himself, and take up his cross and follow Me.” It is association with one rejected by this world, so the rejected path must be the lot of the one who will come after Me.” It must be the setting aside of self. If I put self first, it is a wasted life, I will lose it. If I put Christ first, it is gain now, in the joy of His approval, and gain for all eternity.
What profit would it be to possess the whole world, and lose the soul? The world cannot give happiness now, and the soul without Christ will be eternally miserable. Christ can give happiness now, and an eternity of bliss in His own company. “What shall a man give in exchange for his soul?” Which is better? — a good time now — the pleasures of sin for a brief moment, or rejection with Christ, but assurance of His love and grace, and pleasures which are forever more and fullness of joy with Him for all eternity?
Matthew 16:27. “For the Son of Man shall come in the glory of His Father with His angels; and then He shall reward every man according to his works.” What new thoughts these must have been to them; thoughts of the Son of Man’s kingdom, but to have it He must now be the rejected, suffering, crucified, Son of Man. He is the Savior now. He will be the Judge then.
Matthew 16:28. This verse connects with the transfiguration which some of them would see; and notice that it is a true sample of the kingdom, witnessing His power and coming and majesty. (2 Peter 1:16)
(Continued).

Hezekiah: Brief Lessons on Church Truth, Part 7

A Brief Outline of Lectures on Hezekiah
Read carefully 2 Chronicles 32.
Well, and after these things, and the establishment thereof (2 Chron. 32) —
Then the Tug of War
What after the doors were open, the Holy Ghost owned in His true place: the infinite value of the blood of Jesus: the worship and the gladness: the Lord’s supper in its true place: testimony against all evil: abounding fruits of the Spirit: the porters’ vast range of trust and responsibilities: hearts fully awake to distribute the precious things of the Lord: sanctified, separated to the Lord in holiness: serving the Lord with purpose of heart. Is not this the end? No, indeed, now Satan comes with all his power; “Sennacherib, king of Assyria, came, and entered into Judah, and encamped against the fenced cities, and thought to win them for himself,” or, as in the margin, to break them up, Such is the fact, and it is well to know it; wherever God has gathered a little company to Christ, it is Satan’s object to break it up. In Isaiah 36 we learn that he took the cities of Judah — sad havoc, and at such a time! But was it more terrible than the havoc Satan has made in the professing church of God? So early as the third stage of its history, we find, even in Pergamos, Satan’s seat was there; yea, where Satan dwelleth (Rev. 2:13). In Thyatira, or Rome, we find the depths of Satan spoken of.
And at this moment, the Lord says, “Behold, I will make them of the synagogue of Satan, which say they are Jews, and are not, but do lie: behold, I will make them to come and worship before thy feet, and to know that I have loved thee” (Rev. 3:9). “That I have loved thee” — not that thou hast loved Me. But this is very solemn. My reader may be of the synagogue of Satan, or loved of Jesus: which is it? What a question! Now, as Hezekiah was delivered, we shall do well to study carefully how he meets the power of the adversary. The first thing he did, when he saw the purpose of the enemy, was this: “He took counsel with his princes to stop the waters of the fountains which were without the city; and they helped him.” Then, also, they stopped the brook that ran through the midst of the land. It is very easy to see the wisdom of this on the part of Hezekiah; but not so easy always to see our path in this matter — our safety is in dependence on the Holy Ghost — lowly dependence. The moment we act in independence we are exposed to Satan — nay, we give him a handle. He succeeded with the first Adam, he did not with the Second.
But it may be said, Surely it is right for the water of life to run through the land. Surely it is right for the fountains to gush out anywhere, even without the city. In the unity of the Spirit, and in dependence, perfectly right. But would not that be wrong, however right in itself, if done at the bidding of Satan, or in independence of will, without the guidance of the Spirit? Let us look at the holy dependent One, He who was of quick understanding in the fear of Jehovah. “And when the tempter came to Him, he said, If thou be the Son of God, command that these stones be made bread” (Matt. 4:3). Now, if the Father had commanded Him to turn all the stones on the rocky shores of Gennesaret into bread, would that have been wrong? Did the Holy One listen to the tempter to do that which was right in itself? Did He act independently of His Father’s will? Impossible! He answered him, “It is written, Man shall not live by bread alone, but by every word that proceedeth out of the mouth of God” (Matt. 4:3). We do not find this quick understanding in the fear of the Lord, even in the beloved Paul — well of living water as he was. “After they were come to Mysia, they essayed to go into Bithynia; but the Spirit suffered them not.” Jesus never essayed to do anything that the Spirit suffered Him not.
On another occasion, when Paul and other servants of the Lord, were together in the assembly, the Holy Ghost said, “Separate me Barnabas and Saul for the work whereunto I have called them.” And so they were sent forth on a special mission of service by the Holy Ghost. Says Satan, Are you not sons of God, porters of Christ? Then open a fountain of blessing, or form a gathering here or there, outside the unity of the Spirit. Send forth a running brook through the land — turn the stones into bread. You are wonderful folks, cannot you do as you like? My brethren, is there not special danger here? That which would be most blessed, if done in the unity and fellowship of the Spirit, in lowly dependence, may it not become pride and self-will? The Lord lead us in His own lowly steps, in unfeigned dependence, lest the very truth be used for Satan, like the water for the kings of Assyria. I see great danger in this independence of action. We cannot too much seek the guidance of His eye.
(Continued)

The Coming and Reign of Our Lord Jesus Christ: The Age to Come, or the Millennium, Part 6

Another very interesting, feature of that day is worthy of our notice, and that is, a miraculous river, mentioned several times in the Scriptures. Zechariah 14:8 speaks of it thus: “And it shall be in that day, that living waters shall go out from Jerusalem; half of them toward the former sea, and half of them toward the hinder sea; in summer and in winter shall it be.” The former sea refers to the Dead Sea, the hinder to the Mediterranean; the flowing of the river, you will observe, is in no way affected by the change of seasons. Further details are given in Ezekiel, and we find there that these waters issue out from under the threshold of the house, or temple, eastward, go down into the desert, and “go into the sea (Dead), which being brought forth into the sea, the waters shall be healed.”
At the present time, as is well known, this sea is of such a saline character, that fish cannot live in it; it is often called the Salt Sea for this reason. But the effect of the flowing of the miraculous living waters into it will be, that its present condition will be changed, its waters being healed. “And,” says the prophet Ezekiel, “it shall come to pass, that everything that liveth, which moveth, wither soever the rivers shall come, shall live: and there shall be a very great multitude of fish, because these waters shall come thither: for they shall be healed; and everything shall live whither the river corn eth. And it shall come to pass, that the fishers shall stand upon it from En-gedi even unto En-eglaim; they shall be a place to spread forth nets; their fish shall be according to their kinds, as the fish of the great sea, exceeding many. But the miry places thereof and the marshes thereof shall not be healed; they shall be given to salt. And by the river upon the bank thereof, on this side and on that side, shall grow all trees for meat, whose leaf shall not fade, neither shall the fruit thereof be consumed: it shall bring forth new fruit according to his months, because their waters they issued out of the sanctuary: and the fruit thereof shall be for meat, and the leaf thereof for medicine” (Ezek. 47:1-12).
Joel 3:18 also speaks of this remarkable river, “.... a fountain shall come forth of the house of the Lord, and shall water the valley of Shittim.” Psalm 46:4: “There is a river, the streams whereof shall make glad the city of God.” Psalm 65:9: “Thou visitest the earth, and waterest it: thou greatly enrichest it with the river of God, which is full of water.” Zechariah 9:10 and Psalm 72:8 probably both refer to the same: “He shall have dominion also from sea to sea, and from the river unto the ends of the earth.”
On the other hand, God has decreed the utter destruction of well-known waters, which are now flowing, and also the temporary drying up of certain rivers. “The Lord shall utterly destroy the tongue of the Egyptian sea,” that is, of the Red Sea, which skirts the eastern coast of Egypt, and through which He has already once made a miraculous passage, when He delivered Israel out of Egypt of old; “and with His mighty wind shall He shake His hand over the river, and shall smite it in the seven streams, and make men go over dryshod” (marg., in shoes). “And there shall be an highway for the remnant of His people, which shall be left, from Assyria; like as it was to Israel in the day that he came up out of the land of Egypt” (Isa. 11:15-16).
The river would be the Nile, which at the commencement of the reign of Christ will be smitten in its seven streams, that a way into the land may be opened for a remnant of His beloved people.
These facts are repeated in other passages. In the burden of Egypt (Isa. 19) we are told that the waters shall fail from the sea, and the river shall be wasted and dried up. Again, in Zechariah 10:11, “He shall pass through the sea with affliction, and shall smite the waves in the sea, and all the deeps of the river shall dry up; and the pride of Assyria shall be brought down, and the scepter of Egypt shall depart away.” Also Isaiah 50:2, “Behold, at My rebuke I dry up the sea, I make the rivers a wilderness: their fish stinketh, because there is no water, and dieth for thirst.”
Besides this, the great river Euphrates (once turned from its course at the destruction of Babylon) will also have its waters dried up, that the way of the kings of (or “from”) the east might be prepared. This event takes place under the sixth vial (or bowl) just previous to the end of the future crisis, introductory to the millennium.
But although God’s judgments will fall both on Egypt and Assyria, old enemies of His beloved people, yet will He remember mercy. “And the Lord shall smite Egypt; He shall smite and heal it: and they shall return even to the Lord, and He shall be entreated of them, and shall heal them. In that day there shall be a highway out of Egypt to Assyria, and the Assyrian shall come into Egypt, and the Egyptian into Assyria; and the Egyptians shall serve with the Assyrians. In that day shall Israel be the third with Egypt and with Assyria, even a blessing in the midst of the land; whom the Lord of hosts shall bless, saying, Blessed be Egypt My people, and Assyria the work of My hands, and Israel Mine inheritance” (Isa. 19:22-25).
What a glorious day will this be for this earth! And both man and beast will participate in the deliverance. “The eyes of the blind shall be opened, and the ears of the deaf shall be unstopped. Then shall the lame man leap as an hart, and the tongue of the dumb sing” (Isa. 35:5-6). “The ransomed of the Lord shall return, and come to Zion with songs, and everlasting joy upon their heads: they shall obtain joy and gladness, and sorrow and sighing shall flee away” (Isa. 35:10).
Instead of the present short span of life spoken of by the psalmist, threescore years and ten, and fourscore with labor and sorrow(Psa. 90:10), in that day men’s lives will be greatly prolonged. Of old, men lived to a far greater age than now; but even Methuselah, the longest-lived of all, did not reach to a thousand years; but, in the coming day of millennial blessing, the Lord says, “I will rejoice in Jerusalem, and joy in My people: and the voice of weeping shall be no more heard in her, nor the voice of crying. There shall be no more thence (thenceforth) an infant of days, nor an old man that hath not filled his days: for the child shall die an hundred years old; but the sinner, being an hundred years old, shall be accursed. And they shall build houses, and inhabit them; and they shall plant vineyards, and eat the fruit of them. They shall not build, and another inhabit; they shall not plant, and another eat: for as the days of a tree are the days of My people, and Mine elect shall long enjoy the work of their hands,” (Isa. 65:19-22). An infant of days will not be heard of then; but if a person dies at the age of one hundred years, he is only reckoned as a child, and then his death is on account of sin, when Christ is reigning in righteousness, and he is cut off and accursed. The old men will fill their days: that is, live out a thousand years. The days of the Lord’s people are compared to the days of a tree, and we know that some trees live to an immense age.
There is another striking allusion to the longevity of men in Zechariah 8:4: “Thus saith the Lord of hosts, There shall yet old men and old women dwell in the streets of Jerusalem, and every man with his staff in his hand for very age” (marg., multitude of days). “And the streets of the city shall be full of boys and girls playing in the streets thereof.”
So great and wondrous will be the blessing of God’s people, city and land, that the nations and peoples from all parts of the globe will be attracted to God’s earthly center. “Thus saith the Lord of hosts, It shall yet come to pass, that there shall come people, and the inhabitants of many cities: and the inhabitants of one city shall go to another, saying, Let us go speedily to pray before the Lord, and to seek the Lord of hosts; I will go also. Yea, many people and strong nations shall come to seek the Lord of hosts in Jerusalem, and to pray before the Lord. Thus saith the Lord of hosts, In those days it shall come to pass, that ten men shall take hold out of all languages of the nations, even shall take hold of the skirt of him that is a Jew, saying, We will go with you: for we have heard that God is with you” (Zech. 8:20-23).
Many have had the thought that Satan being bound at this period, the curse taken off the earth, and such wondrous blessing flowing out, that death, the wages of sin, will be done away with; but this is not the case. It is an error often arising from two causes; one from linking all sin with Satan, and forgetting that there is sin in men as well; and the other, referring the scriptures which speak of all Israel knowing the Lord, from the least to the greatest, at the commencement of the millennium, to man generally throughout the whole period.
It is clear from Scripture that men will commit sin during the reign of Christ, and also suffer the penalty — death. For instance, “The child shall die an hundred years old; but the sinner, being an hundred years old, shall be accursed” (Isa. 65:20). And this, you will remark, is a prophecy relating to Jewish blessing.
How terrible is the fall of man! However favorable the circumstances in which he is placed, he utterly breaks down. In innocence he disobeyed God; left to his will unrestrained, the earth was filled with violence; later on he broke the law of God, slew His prophets, murdered His beloved Son. Grace now reigns through righteousness, and man sins openly with a high hand. Christ will shortly reign in righteousness, but sin will manifest itself notwithstanding, though swiftly and surely to be judged. The millennium must run its course, the whole scene be dissolved, before sin will be taken from the earth altogether. Then death and hades being cast into the lake of fire, in the eternal state righteousness will dwell.
(Continued and to be continued).

Correspondence: Psalm 91:5 Explained

Ques. 84 Please explain: Psalm 91:5. E. G.
Ans. Psalm 91 is divided thus:
Psalm 91:1. “He that dwelleth in the secret place of the Most High (the Supreme) shall abide under the shadow of the Almighty.”
Psalm 91:2. The dependent Man, the Messiah, says, “I will say of Jehovah, He is My refuge and My fortress: My God; in Him will I trust.”
Psalm 91:3-8. The Holy Spirit testifies to the blessing of such a man.
Psalm 91:9-13. Israel’s voice also testifies to His protection
Psalm 91:14-16. Jehovah answers His faith with promises.
This Psalm is true in principle for every godly man, but is specially applied to the Lord Jesus as Messiah. But notice that the Lord Jesus suffered in obedience to the Father’s will, even to being forsaken of God. We may suffer also, but will never be forsaken. All these blessings were made good to the Lord in resurrection. But in His life time, He took all from the Father’s hand. So may we. “Thou wilt keep him in perfect peace, whose mind is stayed on Thee; because he trusteth in Thee,” (Isa. 26:4).
Now we can look at Psalm 91:5 and see how safe he is who is dwelling in the secret place of the Most High God, abiding under the shadow of the Almighty God. No enemy can shoot him by day, and no unseen enemy by night can get past the Lord to reach him, but only as God wills.
(Continued)

Inspiration of the Scriptures: The Sixth Hour in John 19:14

The Sixth Hour in John 19:14
The question is, If Mark tells us that Jesus was crucified at ‘‘the third hour,” and Matthew in accordance with Mark’s testimony, says, “from the sixth hour there was darkness over all the land unto the ninth hour,” how can John possibly be correct in saying that the trial of our adorable Lord before Pilate was not ended till “about the sixth hour”?
If, however, it be true, as is generally accepted, that John’s gospel was the last book of Scripture which was written, say about thirty years after the destruction of Jerusalem, and by that time the Jewish mode of reckoning a day, from about six of one evening to about six in the next evening, had pretty generally given away to the Roman mode of reckoning a day, as we now do from midnight to midnight, then all thought of discrepancy in these passages of Scripture vanishes. This would make the ending of the trial to be at six in the morning, and the time of crucifixion to be nine.
Accepting, then, the above-named thought that John, unlike the other evangelists, used Roman time, and that about three hours intervened between the end of the trial and the actual crucifixion of our precious Lord and Savior, let us from the brief Scripture record we have, what transpired during these three hours. What hours of sorrow and suffering they must have been to Him!
First of all, we read, after the wicked trial by infuriated men, led on by Satan, that the holy sufferer was “scourged,” which must have taken up some time; then He was brought into the Pretorium, and the whole band of soldiers were gathered unto Him. There they stripped Him of His own clothes, and put on Him a scarlet robe. There too they “platted a crown of thorns,’’ “put it upon His head, and a reed in His right hand: and they bowed the knee before Him and mocked Him, saying, Hail, King of the Jews!” The enmity of the heart of man to the Holy Son of God was further manifested by their spitting upon Him, and taking the reed and smiting, Him upon the head. All this outrage and other indignities must have extended over some time, for, in addition to all that we have noticed it is said. “After that they had mocked Him,” intimating that the Savior was publicly held in derision for some time. Lastly, we find that they took the robe from off Him, and put His own raiment on Him, and led Him away to crucify Him.
All through these three hours the two malefactors were most likely being tried by Pilate for, as our Lord was the first to be crucified, it is more than probable that He had the precedence in the trial. The whole account seems to imply this. And as the scriptures of the prophets could not but have their fulfillment, the thieves must be crucified with Him, for He must not only bear the sins of many, but “be numbered with the transgressors.” Considering all these things, the three hours’ interval between the end of the Savior’s trial and His crucifixion may easily be accounted for.
The Lamb of God is now led on to Calvary, hearing His own cross till they compelled a man of Cyrene to bear it after Him. On the road, He addressed the weeping women, and enjoined them not to weep for Him, but for themselves and their children, because of what was coming upon them. Then reaching Calvary, He was most cruelly nailed to the cross, fulfilling truly the words of the prophet, ‘‘He was oppressed, and He was afflicted; yet He opened not His mouth: He is brought as a lamb to the slaughter, and as a sheep before her shearers is dumb, so He openeth not His mouth.’’ It was also blessedly true that “He made intercession for the transgressors,” and that “He was wounded for our transgressions, He was bruised for our iniquities,” for “it pleased the Lord to bruise Him” (Isa. 53:5-12). After hanging three hours on the cross in agony and shame, there was darkness over the whole land from the sixth to the ninth hour, at the close of which, He so bitterly felt the anguish of being forsaken of God as our Sin-bearer, that He cried out, “Eloi, Eloi, lama Sabachthani?” and having fulfilled what had been written of Him, He said, “It is finished,” committed Himself to the Father, and gave up the ghost. The rending of the veil in the temple was, no doubt, God’s own testimony to the eternal efficacy of the death of the cross, by which He can now come out to sinners with the gospel, and the believer draw nigh to God by the blood of Jesus.
Looking briefly at the other parts of John’s Gospel in which hours are named, we noticed that our Lord said, in John 11, “Are there not twelve hours in a day?” which may be regarded as true, whether we look at time according to the Jewish or Roman mode of calculation.
In John 1, we read that the two disciples came to Jesus “about the tenth hour,” which looks like ten in the morning, for they abode with Him that day, and we are not told of anything having been done by them on that day before that hour (John 1:35-39).
In John 4, we find our Lord wearied with His journey, sitting on the well “about the sixth hour.” The time seems to have been six in the evening, for the disciples were gone away into the city to buy food, which might have been for the evening meal. After this the Lord went into the city, and many of the Samaritans believed, and besought Him to tarry with them; a kindness often shown in the East when the day was far spent (John 4:6,8,31,40).
At the close of John 4, we are told, that the nobleman’s son was healed “at the seventh hour,” which most probably was seven in the evening. Our Lord said unto him, “Go thy way, thy son liveth  ... . and he went his way.” Now as Capernaum was a distance of several miles from Cana of Galilee, if he took his journey that night so as to reach his home in the morning, then it can easily be understood that when his servants met him, they should have said, “Yesterday at the seventh, hour the fever left him.” “So the father knew that it was at the same hour, in the which Jesus had said unto him, Thy son liveth; and himself believed, and his whole house” (John 4:46-54).
(Continued)

The Shut Door

“And the Lord shut him in” (Gen. 7:16).
Noah did not close the door. These are words that God keeps for Himself. The burden is too heavy for the back of man. To shut that door on a world about to perish would have been too great a responsibility for a son of Adam. Another moment, and another, and another might have been granted by Noah, and the door might never have been shut, and the ship that carried the life of the world might have been swamped. And so it is in the ark of salvation. It is not the church nor the minister that shuts or opens the door. These do God’s bidding; they preach righteousness; they offer salvation, and it is God that shuts and opens the door. O, what a sigh and shudder will pass through the listening universe when God will shut the door of the heavenly ark upon the lost!”

Truths for Young Christians: What the Word of God Forbids for Young Women

Chapter 23
FOR YOUNG WOMEN
“Favor is deceitful, beauty is vain, but woman that feareth the Lord she shall be praised” (Prov. 31:30).
WHAT THE WORD OF GOD FORBIDS.
Dress and vanity. “I will therefore * * * that women adorn themselves * * * not with broidered hair, or gold or pearls, or costly array.” “Whose adorning let it not be that outward adorning of plaiting the hair, or of wearing of gold, or of putting on of apparel” (1 Tim. 2:9; 1 Peter 3:3).
Gossip. “Not only idlers, but tattlers also and busy-bodies, speaking things which they ought not” (1 Tim. 5:13).
Worldliness. “Know ye not that the friendship of the world is enmity with God? Whosoever therefore will be a friend of the world, is the enemy of God” (James 4:4).
What the Word of God Enjoins
As to dress and trite adornment. “I will therefore  ... that women adorn themselves with modest apparel, with shamefacedness and sobriety  ... with good works” (1 Tim. 2:9-10). “Whose adorning let it be the hidden man of the heart, in that which is not corruptible even the ornament of a meek and quiet spirit which is in the sight of God of great price. For after this manner, in the old time holy women also, who, trusted in God, adorned them selves” (1 Peter 4:4-5).
As to walk. ‘‘And now I beseech thee, lady  ... . that we love one another. And this is love, that we walk after His commandments. This is the commandment. That as ye have heard from the beginning, ye should walk in it” (2 John, 5-6).
As to place. (At the feet of Jesus). “Mary hath chosen that good part, Which shall not be taken away from her” (Luke 10:42).
Let us consider together a little the meaning of the scriptures quoted above.
The first danger they point out and also dwell upon at greatest length, is that of dress. So much has already been spoken and written upon this subject that were it not evident that the Spirit speaking both through Peter and Paul regards it of the first importance, nothing more need be said. But we trust our readers will bear with another word on this subject.
The warning in each case specifies that natural adornments (as of hair), artificial adornments (such as gold or pearls), and gay or costly attire are to be particularly avoided. Is such a word unneeded now? Alas! no. It is also high time that every woman who loves the Lord, makes a decided stand on this point. It is one of the silent ways in which a woman can testify for Christ. The public ministry of the Word is peculiarly for men; to godly women God gives a special way of witnessing even by their very dress. A woman’s attire ought to be a subject of serious thought with her (since God has not considered it beneath His notice) and in it surely she should study to please the Lord. He must be the object, or all is valueless, for a nun may be prouder of her gown than a lady of her train; still although the Word of God prescribes nothing more than “modest apparel,” it distinctly and emphatically condemns costly, worldly dress and ornaments. If therefore believers will dress as the Word forbids, they are committing a very serious sin with their eyes open. How is it with you, dear reader? Have you so learned to obey God’s Word, as to heed its solemn voice on this matter, in spite of every inclination to the contrary? O beware of disobeying God by gratifying a love of vanity and dress.
May the Lord reach your conscience with these few words, and show you, perhaps, the secret of much deadness and coldness. If we please ourselves instead of Christ, we shall reap nothing but corruption. From this day let one word describe your appearance on every occasion — “modest apparel.”
The next danger to be avoided is that of foolish talking and evil speaking. We are all prone to this in spite of the many solemn warnings God has given that of every idle word that we speak we shall yet give account. Surely a great deal of talking and letter writing must be included under the head of “idle words.” There is no doubt but that we all require continually to read James 3, a chapter that is but little studied generally. We never shall know till that day the terrible results of idle words and gossip.
Those whom we now address should especially beware of this sin. They have frequent opportunities for social converse, and at such times a strict watch should be set over the door of the lips, that the speech may be that which is good, to the use of edifying! It is distressing to think how many golden hours of the few remaining days before our Lord’s return are not only wasted, but are thus positively used for Satan. Beware, then, of listening to, or circulating, that which is not good; and whatsoever things are true, honest, just, pure, lovely, of good report, “think on these things.” Having spoken of wasting time, it is well to remember there are also other ways in which Satan steals our hours. Many a child of God (it may be the one who is reading these lines) spends more time in the day over some light and foolish book than over her Bible. Novel reading in some shape or form is very common way, even amongst young Christians of wasting many a precious hour.
(To be Continued).

She Obeyed His Voice

She sat and looked at the dress of lace,
A thoughtful look on her dear young face
She gave her heart to Christ last night,
And today her life has been full of light.
Her joy has been full when she thought of His love,
And she wants to obey her Master above.
The dress is lovely. Shall I wear it and go?”
But a voice seemed to whisper, “No! No.”
“Can I not be a Christian and go to a ball.?
If I have Christ must I give up all?”
Like the tinkling of a silver bell.
These words upon her ear now fell;
“I gave My life, My all for thee.
What could’st thou then withhold from Me?”
With her hands on the Word of life she knelt,
And sweetest peace her soul then felt.
She cried from the depth of her pure young heart;
“O! Lord, Thou canst occupy every part,
Into Thy hands I commit my soul,
And ask Thee to take complete control.”
And when she had offered up the prayer,
A song flowed out on the evening air;
A song she had never sung before,
And as she sang it o’er and o’er,
She folded her dress and laid it away;
But she sighed as she thought what her mother would say.
In a moment her mother came into the room,
Arrayed for the ball in a wondrous costume.
“Why, Mary! not dressed? Are you ill my child?”
“I’m not going, mother,” she calmly replied.
“I now belong to my Savior dear.
And I’d rather stay with Jesus here,
For I could not take Him there you know;
To a place like that, He could not go.”
The mother pleaded and talked in vain.
So at last with a look of sorrow and pain,
“Well, Mary, I’ll leave you this time,” she said,
“But such notions as this I’ll soon take from your head.”
“O! God, save my mother,” she cried in despair.
And this she prayed all the evening there.
And God in heaven heard her cry.
He knows if a child just breathes a sigh.
He sent Mary’s word to her mother’s ear,
“I now belong to my Savior dear.”
She tried to drown it in dance and song,
But something told her she was wrong.
She heard the voice so sweet and low,
“I could not take Him there you know.”
With burdened heart she hurries home.
And finds her, daughter in the room.
“O! God save my mother,” she murmurs yet
With accent sweet and low and soft.
-The mother knelt beside her there,
And a cry for mercy was her prayer.
She trusted Jesus with her soul,
And her burden of sin on Him did roll.
He gave the weary soul its rest,
By leaning on His loving breast.
They learn to love Him more and more,
And when the work of day is o’er,
They sit in the evening and talk of His love
And read of the blessings which come from above.
The attraction is gone from the concert and ball,
And Christ become their all in all.

Scripture Study: Matthew 17, Part 1

Matthew 17
The transfiguration scene was a sample of the glory in which the Lord Jesus will yet appear and this sample is the fulfilling of the promise that some standing there should not see death before seeing the kingdom of the Son of Man. Peter’s First Epistle refers Matthew 16; his Second Epistle to this one, Matthew 17. He says (1 Pet. 1:16): “We made known unto you the power and coming of our Lord Jesus Christ.” “Were eye witnesses of His Majesty. For He received from God the Father honor and glory, when there came such a voice to Him from the excellent glory, This is MY beloved Son, in whom I am well pleased. And this voice we heard, when we were with Him in the holy mount.” This confirmed the word of prophecy it was given in view of His death to confirm the faith of His disciples.
Matthew 17:1-2. After six days the Lord brings Peter, James and John with Him into a high mountain apart; and there was transfigured before them. “His face did shine as the sun and His raiment was white as the light”. The glory of His person shone out before their eyes.
Matthew 17:3-9. “And, behold, there appeared unto them Moses and Elias talking with Him.” Luke tells us the subject of their conversation was the Lord’s decease which He should accomplish at Jerusalem; this was the great theme of heaven. Moses was the great law giver, who died and was buried by Jehovah, no man knows where, so no one could worship his burying place. Elias was the prophet, the great law restorer, and was taken to heaven without dying. These might figure the heavenly saints with Christ, the dead raised, and the living changed. Peter, James and John on earth, represent the saints in the kingdom of the Son of Man.
Peter, in his human thought, forgetful of the glory of the revelation the Father gave him of the Lord’s person, desires to make three tabernacles, to put the three persons on the one level. How slow we are to apprehend God’s thoughts of His own Son. Grace put Moses and Elias in His company, in the same glory, but how foolish to think of them as having equal authority over the heart of the believer, a place none can occupy but Christ. And if we should try to do so, the Father must vindicate the rights of His Son. Peter may and did feel it good to be there, but he is not done speaking when the bright cloud overshadowed them; it was the glory cloud of God’s presence. Peter called it “the excellent glory.” And the Father’s voice is heard proclaiming the glory of the person of His beloved Son, the object of His affection, and in whom is His delight. He is the one the disciples are to hear — “Hear Him.” Moses and Elias disappear, and the Lord is there alone, worthy of all honor, the One to teach those who hear the Father’s voice. The Father’s voice proclaims Him, the Father’s delight, and worthy of our affections. What grace! to put such a One before our hearts. What wondrous grace! Here the law, and all attempts to restore man to it, have passed away. The Lord Jesus, the glorified Son of Man, Son of the living God, abides, unfolding to us the knowledge and the mind of God. The disciples, as they heard it, fell on their faces, sore afraid. Jesus came and touched them and said, “Arise, and be not afraid.” “And when they lifted up their eves, they saw no man save Jesus only.” How He encourages them, and sets them at ease with Himself as a man. He who was manifesting the glory of God, was their Lord and Friend, the same as before, better known now than ever. And what blessing is in seeing no man “save Jesus only.”
Matthew 17:9-13. “And as they came down from the mountain, Jesus charged them, saying, Tell the vision to no man, until the Son of Man be risen up from (among) the dead.” He was to suffer, and God would have witness with power in His resurrection, and He would ascend up as Son of Man where He was before as Son of God.
His disciples asked Him, saying, “Why then say the scribes that Elias must first come” Jesus answered, “Elias truly shall first come, and restore all things. But I say unto you, that Elias is come already and they knew him not, but have done unto him whatsoever they listed. Likewise shall also the Son of Man suffer of them.’’ Then the disciples understood that He spake of John the Baptist. John came in the spirit and power of Elias, and acknowledged that he is the forerunner of the Messiah, the rejected One (Isa. 40:3; Mal. 3:1), but not the Elias of Malachi 4:5-6 who is still to come before the Lord appears in power and glory to restore Israel. But to His disciples He could say, for they had received Him, I say unto you he has come already (Matt. 11:14). They belonged to the rejected Messiah, whom men would cause to suffer as John did.
The Scriptures had foretold of the sufferings of Christ, and the glories that were to follow but the natural man looked for the glories, and forgot that the suffering came first. Jesus, the Son of Man, the Father’s well-beloved Son, is now rejected by man, and is going on to suffer death on the cross; this was needful for the glory of God and for the cleansing of the kingdom. Atonement must be accomplished, if redemption, eternal glory and God revealed in grace to man in righteousness, was to be known. The work of the cross secured it all, “Now is the Son of man glorified, and God is glorified in Him” (John 13:31). The kingdom on earth would be part of the fruit of it, in resurrection glory of Christ. God would restore all things. When He brings again His Firstborn Son into the world ( Acts 3:19; Heb. 1:6), then all His promises for Israel would be fulfilled.

Hezekiah: Brief Lessons on Church Truth, Part 8

A Brief Outline of Lectures on Hezekiah
Read carefully 2 Chronicles 32.
Separation to the Lord is a sacred and special thing. Human reason cannot understand it and it must be maintained. The next thing was the building of the wall that was broken down, “and raised it up to the towers.” Ah, those towers! “I will stand upon my watch, and set me upon the tower, and will watch to see what He will say unto me, and what I shall answer when I am reproved” (Hab. 2:1). Man would say, Come down from the watch-tower, use your own judgment; break down the wall of exclusiveness, and be one with all the false doctrine of the land. Satan hates that wall, and hates to see you sit watching before the Lord, to know His mind; and only do His will. O, how much depends on this! — waiting on the Lord in the watch — tower, or doing our own will. But not only must the wall, the hated wall be built up, but we must put on the whole armor of God. O, soldiers of the Lord, ye young men, chosen to stand before Him in these last days, study the Word. Take the whole armor. Be well equipped with the Word of God, spiritual weapons, and shields, in abundance. It is the last conflict, before we rise to meet our Lord. It is well to be informed of the enemy’s intention. Satan is specially occupied with, and determined to destroy, the assemblies gathered to the person of Christ. Such was the object of Sennacherib to take the cities of Judah. Give all diligence, in dependence on the Spirit, not, to help the enemy in his work in any one thing.
Hezekiah spake comfortably to the people, saying, “Be strong and courageous, be not afraid nor dismayed for the king of Assyria, nor for all the multitude that is with him: for there be more with us than with him. With him is an arm of flesh, but with us is the Lord our God, to help us, and to fight our battles.” At such a moment, how important to know with certainty that we are with the Lord, and He with us. No doubt, to some, all this will be utterly unintelligible. But we are meditating on the position of those who desire to do that which is right in the sight of the Lord, in the midst of all that is wrong: wrestling, not with flesh and blood, but with wicked spirits in the heavenlies.
Now we will sit down in our watch-tower, and wait before the Lord. See there the hosts of the foes encamped around, even in the heavenlies. Now, if we look within the enclosed walls of separation to the Lord, what a feeble few! But is the Lord there — is the Holy Ghost there? Yes, He who has conquered the adversary is there. The Holy Ghost sent down is there. All is well, more than all against us  ... . Take courage, be strong in the might of the Lord. It is a remarkable sight, this look from the watch-tower. You see that city enclosed in the walls of separation to Christ. That feeble company has been besieged for nearly ninety years; and, sad to tell it, Satan and his hosts have so deceived Christendom, that nearly all Christendom has besieged that little city. Yea, professing Christians have been notable captains in the attack, through ignorance, have taken service — and not a few runaways have heated the balls red hot. Yea, some of these have become the veriest Rabshakehs. Now what is all this rage against? A feeble little company, who desire to do that which is right in the sight of the Lord, and, whilst answering not a word, place their entire confidence in the Lord. And thus, through His loving care, they abide in the siege. Now, as we sit in the watch-tower, is not this little city a study?
“Thus saith Sennacherib, king of Assyria, Whereon do ye trust, that ye abide in the siege in Jerusalem?’’ Yes! whereon do ye trust? Dreadful have been the destruction of many nations; their gods have not been able to deliver them. In like manner Satan can point to the churches of Asia. Where is the church which was at Antioch, at Ephesus, and all the assemblies in Greece? Rome also? And all the churches of the Reformation? What is their condition now? And are you the feeblest of all; or are you stronger or better than they? Wherein do ye trust, that ye abide in the siege? Solemn questions. But did any of the assemblies of Palestine, Greece, or Europe — or have any of the churches of the Reformation continued in lowly dependence on the guardian care of God the Holy Ghost? No; an arm of flesh, poor puny man, took the place of the Holy Ghost in every one of these — and hence the overthrow. Be it episcopacy, or the popular will, all was of man, and the Holy Ghost set aside. Now, as we look from our watch-tower on that little besieged city, all depends on unfeigned dependence on the Lordship of Christ, and the presence of the Holy Ghost. Mind, all the wall that was broken down must be builded up. Is it not wondrous grace to gather the weakest saints to the person of Christ, separated to Him by that wall, and kept in peace, amidst the rage of Satan, and hosts of deadly, and some, perhaps, only mistaken, foes? So useful to us is the study of this siege, that the Spirit has given us a long account of it in Isaiah 36-37.
As we have noticed here many cities of Judah were taken. This is humbling. We have known it. Many gatherings were gathered years ago, which had not built the wall of separation up to the watch-towers. A most determined attack was made — first, to displace the Holy Ghost, and set up clericalism; afterward to introduce and allow, to refuse to judge, heretical doctrine. Being a young soldier during that early war, I just remember how many gatherings were taken, I believe every one where clericalism had gotten the least footing. I believe every gathering that really trusted the Holy Ghost stood the siege, and Rabshakeh could not deny it.
Yes, what baffled the Assyrians was this trust in the Lord. And now they cry, and lie, threaten, and blaspheme. “But they held their peace, and answered him not a word: for the king’s commandment was, saying, Answer him not.” What dignity in quiet faith! No spirit of defiance and boasting. No, no: rent clothes and sackcloth; deep humility. And though not noticed by Isaiah, yet that little prayer-meeting: “And for this cause Hezekiah the king, and the prophet Isaiah, the son of Amoz, prayed and cried to heaven” (2 Chron. 32:20). Do not forget this: our safety is in prayer in every time of conflict. Some in the siege might not know what to do, when another furious pamphlet cannonade is fired. “Why,” say you, “this is full of the grossest falsehoods.” What did Hezekiah? “Hezekiah received the letter from the hand of the messengers, and read it: and Hezekiah went up unto the house of the Lord, and spread it before the Lord” (Isa. 37:14). And now he pleads with the Lord that dwelleth between the cherubim! It is written, they overcame him by the blood of the Lamb. Yes, the mercy-seat between the cherubim is our place of victory. The enemy, with all his power, was entirely defeated. And whilst we plead there, let us never forget to plead for our dear mistaken brethren, who have been, and are, so sadly deceived by the enemy. Should these lines reach any such, let me beg of you to search the Scriptures, and ask the Lord by the Spirit using that Word, to lead you into the path that is right, in the midst of so much that is wrong. Ah, if even that path leads to the little besieged city within the re-builded walls. The doom of the Assyrian army only illustrates, or foreshadows the certain doom of Babylon the great. (Rev. 17:18). Thus, step by step, how remarkably this history finds its answer in the events of these last clays!
(Continued and to be Continued).

The Coming and Reign of Our Lord Jesus Christ: The Age to Come, or the Millennium, Part 7

Psalm 72:11 teaches that “all kings shall fall down before Him; all nations shall serve Him.” Yet, if we carefully examine other psalms, we find that there are no less than three distinct passages where, in the marginal reading, we find that the submission of some of the nations is only feigned after all, through fear of the power and might of the King of kings. (See Psa. 66:3; 18:44; 81:15, marg.) This is further confirmed in Zechariah 14:16-19: “And it shall come to pass, that every one that is left of all the nations which came against Jerusalem, shall even go up from year to year to worship the King, the Lord of hosts. and to keep the feast of tabernacles. And it shall be, that whoso will not come up of all the families of the earth unto Jerusalem to worship the King, the Lord of hosts, even upon them shall be no rain. And if the family of Egypt go not up, and come not, that have no rain, there shall be the plague, wherewith the Lord will smite the heathen that come not up to keep the feast of tabernacles. This shall be the punishment of Egypt, and the punishment of all nations that come not up to keep the feast of tabernacles.”
This scripture speaks plainly of threatened judgments on nations that refuse to bow to the authority of the one King over all the earth, and is a remarkable instance of the perfect accuracy of God’s Word. Egypt is singled out from the other nations, and threatened with a plague, should they he insubject. The withholding of rain, which would cause a terrible famine, would be no punishment to her; for it is a well-known fact, as mentioned in the passage itself, that a great part of Egypt has no rain now. Crops are obtained by the careful use of the water of the mighty river Nile, which at certain seasons overflows its banks. Thus upon Egypt the Lord threatens a plague.
Another striking and incontestable proof of the unconverted, sinful state of great masses of Gentiles in the millennium, is the fact, that when Satan is loosed out of the abyss at the close of the thousand years, he makes a last effort to overthrow the kingdom of Christ. He goes out “to deceive the nations which are in the four quarters of the earth, Gog and Magog, to gather them together to battle: the number of whom is as the sand of the sea. And they went up on the breadth of the earth, and compassed the camp of the saints about, and the beloved city; and fire came down from God out of heaven, and devoured them” (Rev. 20:7-9). Cog and Magog are here used as symbolical terms of the great mass of nations who will then be led by Satan against the Lord’s people of that day, and must in no way be confounded with Gog, the land of Magog. (Ezek. 38:1-3); the one is a gathering at the close, the other at the commencement of the reign of Christ. They come up on the breadth of the earth, on all parts of it, and compass the camp of the saints about, wherever they may be, far and wide, and also the beloved city, that is, Jerusalem, seeking their destruction. God suddenly pours out fierce and unsparing judgment; fire will come down and devour them. Their deceitful leader, who will have been bound the thousand years in the bottomless pit, or abyss, is then cast into the place of eternal torment, the lake of fire and brimstone where already the two great leaders of the apostasy, the beast and false prophet have been suffering during the same period. All these, as the Scripture plainly declares, shall be tormented day and night forever and ever (Rev. 20:10).
Then, as regards the brute creation, we find that the present enmity existing between different animals will cease, and that instead of preying one upon the other they will dwell together in peace and harmlessness: “The wolf also shall dwell with the lamb, and the leopard shall lie down with the kid; and the calf, and the young lion, and the fatling together; and the little child shall lead them. And the cow and the bear shall feed; their young ones shall lie down together: and the lion shall eat straw like the ox. And the sucking child shall play on the hole of the asp, and the weaned child shall put his hand on the cockatrice’s (adder’s) den. They shall not hurt nor destroy in all My holy mountain; for the earth shall be full of the knowledge of the Lord, as the waters cover the sea” (Isa. 11).
This interesting fact has often, like many other prophetical scriptures, been spiritualized away, and widely distorted from its evident literal weaning. And this is not the only passage which speaks of it, but similar language is used in Isaiah 65:25, “The wolf and the lamb shall feed together, and the lion shall eat straw like the bullock: and dust shall be the serpent’s meat. They shall not hurt nor destroy in all My holy mountain, saith the Lord”. The sole exception to the general blessing mentioned here, is another instance worthy of note, as showing the perfect harmony of every statement of the Word of God — “Dust shall he the serpents meat. In pronouncing the curse upon the serpent in Genesis 3, God said, “Upon thy belly shalt thou go, and dust shalt thou eat all the days of thy life”; so that its condition will remain the same to the end. And “I  ... . will cause the evil beasts to cease out of the land and they shall dwell safely in the wilderness, and sleep in the woods” (Ezek. 34:25).
The curse pronounced upon the earth by the Lord God consequent upon the fall of man (Gen. 3:17) will then be removed; for the Revelation tells us “there shall be no more curse (Rev. 22:3). Thorns and briers shall no longer be brought forth as now; but “instead of the thorn shall come up the fir-tree, and instead of the brier shall come up the myrtle-tree,” which are both evergreens (Isa. 55:13), “and the desert shall rejoice and blossom as the rose. It shall blossom abundantly, and rejoice even with joy and singing” (Isa. 35:1-2). ‘‘I will open,” too, saith the Lord, “rivers in high places, and fountains in the midst of the valleys: I will make the wilderness a pool of water, and the dry land springs of water. I will plant in the wilderness the cedar, the shittah-tree, and the myrtle and the oil-tree; I will set in the desert the fir-tree, and the pine, and the box-tree together” (Isa. 41:18-19).
The earth will then be so wondrously fertile and productive, that the labor necessary to procure a crop in that day will be nothing to be compared to the present toil; for “the plowman shall overtake the reaper, and the treader of grapes him that soweth seed; and the mountains shall drop sweet wine, and all the hills shall melt” (Amos 9:13). A beauteous strain is found in the sixty-fifth Psalm, showing, how richly God will then bless the earth, and how, consequently, it will teem with plenty: ‘‘Thou visiteth the earth, and waterest it: Thou greatly enrichest it with the river of God, which is full of water: Thou preparest them corn, when Thou halt so provided for it. Thou waterest the ridges thereof abundantly; Thou settlest the furrows thereof: Thou makest it soft with showers; Thou blessest the springing thereof. Thou crownest the year with Thy goodness; and Thy paths drop fatness. They drop upon the pastures of the wilderness: and the little hills rejoice on every side. The pastures arc clothed with flocks; the valleys also are covered with corn: they shout for joy, they also sing” (Psa. 65). And “there shall be a handful of corn in the earth upon the top of the mountains; the fruit thereof shall shake like Lebanon” (Psa. 72).
The seasons themselves will not change; for the Lord said after the flood, “While the earth remaineth, seed-time and harvest and cold and heat, and summer and winter, and day and night, shall not cease.” The heavens and the earth will be brought into blessed connection during this marvelous epoch, the dispensation of the fullness of times, when all things shall be gathered together in one in Christ (Eph. 1:10), and all things reconciled; (mark, things, not all persons) (Col. 1:20). “And it shall come to pass in that day, I will hear, saith the Lord, I will hear the heavens, and they shall hear the earth; and the earth shall hear the corn, and the wine, and the oil; and they shall hear Jezereel” (Hos. 2:21-22).
The thousand years having expired then cometh the end, when Christ “shall have delivered up the kingdom to God, even the Father; when He shall have put down all rule and all authority and power. For He must reign till He hath put all enemies under His feet., The last enemy that shall be destroyed is death. For He hath put all things under His feet. But when He saith, All things are put under Him, it is manifest that He is excepted, which did put all things under Him. And when all things shall be subdued unto Him, then shall the Son also Himself be subject unto Him that put all things under Him, that God may be all in all” (1 Cor. 15:24-28).
Hark! the sound of jubilee,
Loud as mighty thunders’ roar.
Or the fullness of the sea
When it breaks upon the shore!
Hallelujah! for the Lord
God omnipotent shall reign!
Hallelujah! let the word
Echo round the earth and main!
Hallelujah!-hark! the sound
From the depth unto the skies,
Wakes above, beneath, around,
All creation’s harmonies!
See, Jehovah’s banner furled,
Sheathed His sword: He speaks-’tis done;
And the kingdoms of the world
Are the kingdoms of the Son!

Correspondence: Synagogue of Satan; Matt. 16:19

Ques. 85. Who are those of the synagogue of Satan, which say they are Jews, and are not? And what do they teach? E. M. A.
Ans. This is symbolic language. The Jews claim to be Jehovah’s people by descent. Many in Christendom do the same, claiming to have the true religion from their forefathers. This recognizes their being in the flesh and under the law and ordinances; and pours contempt on those who honor Christ’s name and Word only. They are the synagogue of Satan, for they are in opposition to the Word of God.
Ques. 86. What were the keys of the kingdom of heaven given to Peter? and when did he use them? Matthew 16:19. C. B.
Ans. The keys are the authority the Lord gave to Peter to act for Him in what bore His name on earth.
In Acts 1 he directed that an apostle be appointed in place of Judas Iscariot.
In Acts 2 he spoke, convicting the Jews of putting Christ to death, and exhorted all to repent and to be baptized in His name. Three thousand were loosed from their sin that day.
In Acts 3 he healed a lame man, and convicted the Jews of the death of Christ, but, alluding to Christ’s prayer on the cross, said, “I wot that through ignorance ye did it, as did also your rulers,” but God by them fulfilled His Word. And now, if they would repent as a nation, Jesus Christ would come back and reign over them, fulfilling all the prophets. This bound their sin upon them as a nation.
In Acts 5 he bound Annanias’ and Sapphira’s sin upon them, and the judgment of God took them away. This is discipline, not eternal judgment (See 1 Cor. 11: 32).
In Acts 8, he, with John, went down to Samaria. The converts there had not received the Holy Ghost, though truly converted and baptized. Laying on of hands is identification with. And they prayed for them and laid their hands on them, declaring thus their oneness with the assembly at Jerusalem. The old feud between Samaritans and Jews is considered gone in the church. Heaven ratifies their act and the converts received the Spirit. But Simon the sorcerer is manifested as an unsaved man, though baptized. The first empty professor is seen in him.
In Acts 10, Peter opened the door to the Gentiles, and commanded them, who had believed and received the Spirit, to be baptized in the name of the Lord. There may be other occasions where he used his authority. Others had authority given them as apostles — John, James, and Paul.

Inspiration of the Scriptures: The Bible - its Unity, Part 1

Every student of Scripture meets with difficulties, and finds much that he cannot understand; but, as we have said before, waiting on God in humility of mind, and in unfeigned dependence on the Holy Spirit, it is astonishing how He clears up the difficulties for us. It is, however, very remarkable how the Lord seems to have anticipated many of the objections which are raised by learned and scientific men, who rely on their own reasonings instead of simply accepting what God has said. In addition to some instances already pointed out, we may notice that the Lord authenticated the doctrine of the descent of the human race from one pair of parents from the beginning of the creation (Mark 10:6); of the taking away of all by the Deluge, except those who were in the ark (Matt. 24:37-39); of the destruction of Sodom and Gomorrah with fire and brimstone; of the miraculous supply of the manna; and the story of Jonah and the fish (some great fish, not necessarily a whale), and his mission to Nineveh and its effects.
In meditating on the gospels it is scarcely possible not to notice the reverence and honor paid by our Lord to “the Scriptures.” Only think of Him as to this when on the cross. After being bodily suspended there for six hours with nails in His hands and His feet, in unmitigated and indescribable pain, with all the sorrow too that pressed upon His sprit, of being betrayed by one, denied by another, and forsaken by all; when lover and friend had been put far from Him, and His acquaintance into darkness; when consciously too bearing our sins in His own body on the tree, the Holy One made sin for us; the wrath of God, the forsaking of God, the judgment of God for our sins falling upon Him, so that His soul was made an offering for sin; so occupied was He with the prefect will of Him who sent Him, that the silence, of the unparalleled crisis was broken by the words, “I thirst”; and why? There remained one little scripture that had not as yet had its fulfillment. What was it? “In My thirst they gave Me vinegar to drink” (Psa. 69:21). We therefore read, “Jesus knowing that all things were now accomplished, that the scripture might be fulfilled, saith, I thirst. Now there was set a vessel full of vinegar; and they filled a sponge with vinegar and put it upon hyssop, and put it to His mouth. When Jesus, therefore, had received the vinegar, He said, It is finished, and He bowed His head, and gave up the ghost” (John 19:28-30). And we may well ask, when pondering such a scene as this, could anything be more perfect as sealing the divine authority of Scripture with His own blood? What true reverence for the sacred writings! What perfect subjection to every word of it! In a moment too of unutterable agony, depths of deepest sorrow, hour of darkness and unutterable woe; and above all, the forsaking of God. His heart broken with reproach. Satan bruising, men deriding in hateful enmity, every bone out of joint, yet manifesting perfect love to the Father, perfect obedience to His Word, perfect subjection to His will, perfect reverence for what is written, and perfect love to all who trust in Him. What perfections cluster around the cross at Golgotha! Happy those who know Him risen and ascended as the object of their faith, and the One in whom they trust. What lessons this unparalleled scene reads to us as to the reverence for Scripture, because it is the will and Word of God; and how truly the Holy Spirit says of Him, that He hath “left us an example, that we should follow His steps.”
(Continued)

Grace Triumphant

“Jesus said, I am the door, by Me if any man enter in he shall be saved” (John 10).
A lady young, of noble mien,
In you grand mansion oft was seen,
The gavest of the gay-
But ah! she in those lightsome hours,
Was captive led by hostile powers,
Who claimed her as their prey.
The world had marked her for its own,
Satan in her had raised his throne,
Sin in bondage held her,
A worldly man, her father was,
Her friends engaged in Satan’s cause.
O, how great her danger!
Her danger great she knew not of,
She thought not of the Lord above,
His Word she cared not for;
Till He by grace arrested her,
Yea, stayed at length her mad career,
She entered at “the door.”
The scales of sin fell from her eyes,
She saw her guilt as mountains rise,
In that momentous hour,
Her soul then quailed with anxious fears
Her eyes were filled with briny tears,
When God displayed His power.
She earnestly deliverance sought,
Through Him who life to light had brought
From all her mighty foes;
She pardon found, and full release
And all her hopes of joy and peace,
She did on Christ repose.
O wondrous love!-O how divine!
The grace that did her soul refine,
And purge away the dross.
Of heavenly food led her to take,
And for her dear REDEEMER’S sake
To count all things but loss.
But earthly friends became her foes,
She was no longer loved by those
Who loved her much before.
Their enmity was not disguised,
They many subtle plans devised,
And tried her oft most sore.
Through grace she triumphed o’er their wiles,
Their anger turned away with smiles,
And spake to them in love.
They wondered at her conduct mild,
But knew not, that which they reviled,
Was given her from above.
The “hope forlorn” they often tried,
They hedged her in on every side.
But then her Lord was near,
Temptations came like to a flood,
But like a rock unmoved she stood,
A stranger then to fear.
Possessions vast her father owned,
He now upon his daughter frowned,
As thus he inly said-
“In pleasure’s path she will not stay,
Nor will she mingle with the gay,
To me she is as dead.
“Much of my wealth to her I’d give,
But as for God she says she’ll live,
My gold shall not be hers.
My noble friends I’ll now invite,
To spend with me a jovial night,
And drive off saintly fears.
“The gay and fair ones then shall Sing,
And loudly shall the welkin ring.
I’ll on my daughter call
To raise a song and swell the mirth-
Should she refuse-despite her birth-
I’ll spurn her from my hall.
And then in poverty and woe,
She to the saintly ones shall go,
And rue her mournful state.”
He knew not of the peace divine
Which made her soul with glory shine,
Nor of her great estate.
And thus her father’s house became
That one in which her Savior’s name,
She boldly must confess.
The festive room with lights was hung,
The worldling’s songs were freely sung,
Her foes around did press,
It now behooved her, too, to sing,
So lifting her heart to heaven’s king,
She thus began her song-
“No room for mirth or trifling here,
For worldly hope, or worldly fear,
If life so soon is gone.”
With voice most sweet and tones most clear,
She sang the song to her so dear,
Of judgment then she sang
Of CHRIST her Savior and her guide,
Of sins that had Him crucified,
Till heaven with praises rang.
Till heaven with praises rang-
I say, She sang-for ere her heavenly lay
Was ended-stillness reigned-
Then every heart was filled with awe,
The beauty of holiness they saw,
Though none to own it deigned.
In solemn silence the room they left,
E’en then of earthly joy bereft,
Conscience bold accusing,
But one there was whose flowing tears,
Proclaimed that though advanced in years
The cross he had not seen.
That one, her much loved father was,
He had despised God’s holy laws
Throughout his former days.
He now his daughter’s teaching sought,
As she God’s truths her father taught,
She sang her Savior’s praise.
Now strive to estimate her joy,
So pure-so free from all alloy.
When she her father heard-
Pleading before the throne of grace,
Seeking her heavenly Father’s face,
And yielding to His Word.
O now portray the wondrous scene,
He who so long content had been
At Satan’s shrine to bow-
Resolved to give his future hours,
To consecrate his wealth and powers,
His all-to JESUS-NOW.
O let us now adore the Lord,
As we His wondrous works record,
Let us His praises sing.
Since He of bliss the fountain is.
Come-let us ever drink at this,
The never failing spring.
Ashamed!-believer never be,
Nor from your foes in terror flee,
Since CHRIST your LORD is near,
He all your foes will put to flight,
In darkest hours He’ll prove your light,
And drive away your fear.

Truths for Young Christians: What the Word of God Enjoins for Young Women

Chapter 23
FOR YOUNG WOMEN.
The last warning heading this paper is against worldliness, and is expressly addressed to women as well as men. In this case the words of Scripture are so terribly pointed and strong that they need no comment. Are you, beloved reader, prepared to class yourself as an enemy of the God who has so loved you? If not, make a clean cut with the world; in its songs and dances, in its parties and pleasures, in its fashions and follies, in its pride and vanity. Be true to Christ, and esteem the cost of being so “greater riches than the treasures in Egypt.” Let the love of Christ be the constraining power to detach you from all that grieves Him.
And now a word or two on what the Word of God enjoins. We find modest apparel, shamefacedness, sobriety, good works, a meek and quiet spirit, all spoken of, and telling us with one voice that the peculiar charm of a woman’s position is in a humble and quiet walk. Her place at the feet of Jesus, is not so public as man’s, but it is as precious to the heart of Christ. And surely that woman is beautiful to the eyes of the Lord (and shall we add, in the sight of the angels. See 1 Cor. 11:10), and in those of every believer, who is adorned with these virtues. If you then, beloved reader, would find favor with God and every godly man, let no grace be lacking, remembering that
“Fairest and best adorned is she
Whose clothing is humility.”
A real difficulty with many Christian young women is how to spend their time. Beyond doubt their sphere pre-eminently is home, and there in a diligent discharge of the daily duties to testify against the unsatisfied restlessness, and the craving for excitement that blight the lives of so many. If early rising is practiced, there is time for the diligent study (not mere hasty reading) of the Bible; and if the home duties do not claim all the time, the sick and poor are always within reach of those who have hearts to search them out. Devotion to Christ thus shown is peculiarly a woman’s sphere, and is, as it were, the pouring of the precious ointment over the members of His body.
All this, however, requires self-denial, as we have said. There must be early rising to gather the manna before the sun is up; and none but those who have tried it can tell the quiet and rest of a day, the first hour of which is spent alone with God.
Let each young reader of these pages then steadfastly resolve to let God have the first hour of her day before the bustle of the house begins, and she will be well repaid what it will cost her.
It is surely unnecessary to warn any reader of these pages against scenes of worldly excitement and pleasure. Balls, parties, and concerts are not to be named in connection with true followers of a rejected Christ. But there are other forms of excitement that often injure the simplicity of a young Christian’s life. There is such a thing as religious excitement, an unhealthy craving after meetings, rushing about from one to another, and often up to very late hours, to the neglect of home duties, and fostering a great distaste for quiet occupations. Meetings are very helpful for the soul that is feeding on Christ, but will never do to take His place, or the place of any home duties we may have given us to do. For young believers no text is more profitable than 1 Timothy 5:4.
In conclusion, do not forget the mighty power you wield of unconscious influence, and although you may never hear of the results until you reach heaven, do not cease to speak of Christ by your dress, your conversation, your ways, your walk, and your good works.
Dear young believers, I would beseech you, do not let your days slip by in the idle listless way in which so many do; seek to be earnest, consistent, devoted followers of Christ, daily studying the Word of God, and working hard for the Lord. Above all, if you would render such a service as Mary in Matt. 26, do not forget the secret of her strength in Luke 10. May the Lord make each of you a true Mary of Bethany.
(Continued)

Scripture Study: Matthew 17, Part 2

Matthew 17
Matthew 17:14-18. Here a new scene shows the incapacity of the disciples to act on what was in Him for their faith to use, also shows His never-failing compassion to the needy father whose son was a lunatic, and possessed by a demon, “Have mercy on my son, I brought him to Thy disciples and they could not cure him.” How slow they were to apprehend their place. He had to rebuke them in the presence of the glory, on the mount, and now again before the world and Satan’s power. O, faithless and perverse generation, how long shall I be with you? how long shall I stiffer you? They cannot profit by His presence, and He must leave them, but He will not fail to meet the need of individual faith, so in the same breath He says, “Bring him hither to Me.” The distressed father gets for his child the desired blessing, for Jesus rebuked the demon, and he departed out of him; and the child was cured from that very hour.
Matthew 17:19-21. Then came the disciples to Jesus apart and said, “Why could not we cast him out?” And Jesus said unto them, “Because of your unbelief: for verily I say unto you, if ye have faith as a grain of mustard seed, ye shall say unto this mountain, Remove hence to yonder place; and it shall remove; and nothing shall be impossible unto you.” If faith was in exercise, however small it might be, the difficulty would be overcome. So He adds, “How be it this kind goeth not out but by prayer and fasting.” Prayer is felt dependence expressed to God; fasting is abnegation of self. It is giving oneself up to divine things, so that for the time bodily desires are set aside. There must be this practical energy of faith to avail ourselves of His power.
Matthew 17:22-23. Again the Lord explains definitely to them what was to happen to Him, and they are very sorry. It was necessary for the glory of God to bring in redemption, yet by the hands of men He would suffer.
Matthew 17:24-27. This tribute was a temple-tax that every godly Israelite would be willing to pay. When the collectors came to Peter, and asked, “Doth not your Master pay tribute?” Peter said at once, “Yes.” And on going into the house, the Lord anticipated him by asking, “What thinkest thou, Simon? of whom do the kings of the earth take custom or tribute? of their own children, or of strangers?” Peter saith unto Him, “Of strangers.” Jesus saith unto him, “Then are the children free. Notwithstanding, lest we should offend them, (stumble them) go thou to the sea, and cast an hook, and take up the fish that first cometh up; and when thou hast opened his mouth, thou shalt find a piece of money: that take, and give unto them for Me and thee.”
What a rebuke to Peter’s forgetfulness of His divine person.
What lowly grace to associate the disciples with Himself as sons of the great King of the temple. What making nothing of Himself to do the Father’s will, and “lest we should stumble them.” He, the Son of God, yet He took this lowly place in grace. He would condescend to pay tribute money for the repairs of the temple, yet in doing it shows Peter that He is Sovereign, Lord of all creation, but how it would bind the heart to Himself when He said, “That give for Me and thee.”
(Continued from page 296).

Hezekiah: Brief Lessons on Church Truth, Part 9

A Brief Outline of Lectures on Hezekiah
Read carefully 2 Chron. 32.
THE UNSETTLED QUESTION.
You may have thus been privileged. The doors opened. The Holy Ghost known and owned in the assembly. The immutable value of the precious blood known. Joy, and bowing, and worship in public. Gathered to take the Lord’s supper as it is written. Testimony in the world. Abundance of fruits. You may know the privileges and responsibilities of the porter — the parcel-carrier for Christ. You may have been preserved through long years of the siege of the city, within the exclusive walls. Great victories of faith and prayer. And yet there may be the unsettled question of Isaiah 38.
“In those days (days of such victories) was Hezekiah sick unto death.” We now come to the inner experiences of the soul — a soul that has not yet learned the death of the flesh. What a sentence on the old man — “Thou shalt die, and not live.” Ah, now he says, “I beseech Thee, O Lord, remember how I have walked before Thee in truth.” Very sorely did Job try this, but it would not do. Neither would it do for Hezekiah; no; there he lay, with his face to the wall; and he wept sore. The Lord is very pitiful; He heard those prayers, He saw those tears, and He granted a new term of life. He also assured him of full deliverance of the city, and He gave him a remarkable sign, that the Lord will do this thing which He hath spoken. “Behold, I will bring again the shadow of the degrees, which is gone down in the sundial of Ahaz, ten degrees backward.” “He restoreth my soul.” “If I wash thee not, thou hast no part with Me.” We speak of the sun rising, or going down; as to fact, it is the earth that has turned from the sun. So with our souls. Our constant tendency to depart from the Lord is like the diurnal motion of the earth. The Lord is ever the same, as we always find, when He restoreth our souls.
Will you read Hezekiah’s own account of his experience in learning this unsettled. question? (Isa. 38:9-20). His heart almost sank in despair.
Is it not very striking, after such public testimony for the Lord? He says, “I shall not see the Lord, even the Lord in the land of the living.” Yes, at such a time, such is the sense of the vileness of the flesh. And Satan now suggests the terrible thought, that afflictions prove that God is against us. He says, “He will cut me off with pining sickness; from day to night wilt thou make an end of me.” The dreadful working of unbelief. “I reckoned till morning, that as a lion, so will he break all my bones.” O, what chattering, and what mourning! “Mine eyes fail with looking upward: O LORD, I AM OPPRESSED, UNDERTAKE FOR ME.” Was it not just so with Job? “I abhor myself, and repent in dust and ashes.” Is not this the very condition of a quickened soul under law? “For we know that the law is spiritual, but I am carnal, sold under sin” (Rom. 7). However earnest the desire of such a soul to keep the righteous requirements of the law, yet no power. “For I know that in me (that is in my flesh) dwelleth no good thing; for to will is present with me; but how to perform that which is good I find not.” Hezekiah says, “Behold, for peace, I had great bitterness.” Behold, reader; until the question of the old man is settled, for peace what bitterness of soul you have had. Yes, bitter herbs indeed. You love the Lord — you long for holiness; but, oh, the bitterness, the loathsome flesh. Did I not hear you saying, Surely I must be a hypocrite: What did that deep groan say: O, shall I ever see the Lord? Are not all these afflictions a proof that He is against me? O, how I loathe, abhor myself! I am oppressed — undertake for me! O, wretched man that I am — no better, no better — who shall deliver me from the body of this death?
This lesson must be learned; and what is the answer? With Hezekiah it is, “What shall I say? He hath both spoken to me, and himself hath done it.” With Paul it is the eye turned from wretched self, the old man, to Christ, and then the joyful exclamation, “I thank God, through Jesus Christ our Lord.” What a deliverance, when we learn the answer to the unsettled question, as to the old man! He hath both spoken Himself, and He hath done it. He has been fully judged for us, made sin for us. In the likeness of sinful flesh, and by a sacrifice for sin, condemned sin in the flesh. Thus we accept the death of the old man, crucified with Christ — buried with Him in death. Judicially there is the end of I. No longer I, but Christ. I thank God, through Jesus Christ our Lord. He hath spoken it — He hath done it. There is generally a little more sobbing at the funeral of the old I. “I shall go softly all my years, in the bitterness of my soul,” says Hezekiah. No, not so, Hezekiah! Not so, deeply exercised soul! No, you will have higher thoughts.
Sweetly now the Spirit whispers in the heart, “But thou hast, in love to my soul, delivered it from the pit of corruption; for Thou hast cast all my sins behind Thy back.” Yes, dear soul, it is true, quite true-rest in it. O, think of the love of God in delivering you from the pit of corruption. Had He left you to your wretched self — ah, the pit, the pit. Thanks be to God: Now a little further discovery for you. It is blessed to be brought to the foot of the Cross yes, to the very grave of Christ — dead and buried with Him. This is the answer in part, full answer, to the old man; death, and the grave of Christ. But do not be too sure that that is all — that you are to remain there. No; “For the grave cannot praise Thee; death cannot celebrate Thee; they that go down into the pit cannot hope for Thy truth.” Is not this also most true? Yes, if Christ be only dead for us; and if we are only dead, and even buried with Him; all is in vain. “If Christ be not raised, your faith is vain! ye are yet in your sins.” (1 Cor. 15:17). “The living, the living,” exclaims Hezekiah, “he shall praise Thee: as I do this day.” Yes, we are not only dead with Thee, Lord Jesus but risen in Thee. “The living, the living, shall praise Thee, as I do this day.”
Thus, through the death of Christ, we have passed through death into life. Old things are passed away, all things new, and all of God. What a new creation!
It is not now bitterness of soul, and doubts, and misery. O, no, says Hezekiah, “The Lord was ready to save me; therefore we will sing my songs all the days of our life in the house of the Lord.” O, that is far better; yes, let us sing His praise with adoring hearts.
The Warning
The business of the ambassadors of the princes of Babylon. If we are left but for a moment, to prove us, alas! all is failure again. What a danger, even in much blessing and prosperity!
“Hezekiah was glad of them, and showed them the house of HIS precious things” (Isa. 39:2). “Mine house;” “my treasures” (Isa. 39:4). Yes, if left to ourselves, it will be, “our precious truth,” “our testimony,” “our table,” “our treasures.” Rich, and increased in goods, and have need of nothing. There is only a step from Laodicea to Babylon. We cannot praise our God and Father too much for the precious truth He gives — for the treasures of His Word — for the restored table of the Lord. But are these treasures to be boastingly shown as ours to the princes of Babylon? If Hezekiah humbled himself, surely we ought to take the lowest place: nay, not take it — it is ours. The Lord keep us near Himself, in lowly dependence, and enjoying the discernment of the Holy Ghost. Thus may He enable us to do that which is right in the sight of the Lord, in the midst of all that is wrong.
Concluded.

The Coming and Reign of Our Lord Jesus Christ: The Eternal State

The Eternal State
We closed in our last paper with the giving up of the kingdom to God, even the Father, by Christ, the Son. This brings us to the end of time, and, the dissolution of the present condition of the heavens and the earth. For “the heavens shall pass away with a great noise, and the elements shall melt with fervent heat; the earth also, and the works that are therein, shall be burned up  ... . Nevertheless we, according to His promise, look, for new heavens and new earth, wherein dwelleth righteousness” (2 Peter 3:10,13).
Another allusion is made to the present heaven and earth in Revelation 20:11. The prophet John sees “a great white throne, and Him that sat on it, from whose face the earth and the heaven fled away; and there was found no place for them.” Then, and not till then, the dead, that is, all who have died out of Christ, impenitent, unbelieving, throughout the whole course of time, small and great, stand before God (or “the throne,” lit.), and are judged according to their works; a perfectly distinct act of judgment from that of the quick in Matthew 25, as we have seen. Only the wicked dead stand here; the saints have already enjoyed a thousand years of bliss with Christ. Books were opened, probably the records of their works. Another, the book of Life, was opened also, but only to show that their names are not there. None can escape this awful tribunal who have died in their sins: the sea, death, hades, all give up their dead; and they were judged every man according to their works. Death and hades, no longer needed, were first cast into the lake of fire; and then, when time exists no longer, their eternal doom is sealed. “Whosoever was not found written in the book of life was cast into the lake of fire.” This is the second death.
God is light, as well as love. “The high and lofty One, that inhabiteth eternity, whose name is Holy.” cannot lower the claims of His holiness. “He will by no means clear the guilty,” is the declaration of the Old Testament. In the New we have brought out the wondrous truths of redemption and salvation, wrought by the gift of His only-begotten Son, to die, the guiltless for the guilty. The justice of God took its course, and its stroke fell upon the Holy and the just One on the cross at Calvary (2 Cor. 5:21). And God has shown His estimate of the perfect sacrifice, in that He has raised Him from the dead, and now declares His righteousness in proclaiming a free pardon to all, and in justifying every one that believeth (Rom. 3:25-26).
All who reject or neglect the wondrous provision of His grace, will surely reap the consequences of their folly in eternal misery in the lake of fire. The strongest language is employed in the Word of God to express the awfulness and eternal duration of the punishment of the wicked. “He that believeth not the Son shall not see life; but the wrath of God abideth on him” (John 3:36). The rich man in hell, crying that he is tormented in this flame, finds no relief to his misery, not even a drop of water, but a great gulf fixed between himself and the place of mercy and blessing (Luke 16:19-31). and no possible means of crossing it. There shall be weeping and wailing and gnashing of teeth (Matt. 13:42; 22:13).
Eternal fire (Jude 7); their worm that never dies (Mark 9:44); fire not quenched (Mark 9:48); everlasting contempt (Dan. 12:2); everlasting punishment (Matt. 25:46); everlasting destruction (2 Thess. 1:9); damned (2 Thess. 2:12); blackness of darkness forever (Jude 13); perdition (2 Peter 3:7); swift destruction (2 Peter 2:1); utterly perish (2 Peter 2:12); no forgiveness (Matt. 12:31, 32); and such — like terms, are surely enough to convince every honest soul of the eternal and awful character of the judgment of the ungodly. The words “perish” and “destruction” are often distorted from their obvious meaning to support the speculations and theories of rationalists, skeptics, and others, and made to mean “annihilation,” whereas there is no hint of such a thing in the Word of God. These very words are constantly used by us to mean otherwise. If we speak of being “perished” with cold, or of some furniture being “destroyed;” do we mean that either we or the furniture are put out of existence? If death is ceasing to exist, how will men weep, and wail, and gnash their teeth?
“The things which are seen are temporal; but the things which are not seen are eternal” (2 Cor. 4:18).
God at the first breathed into man’s nostrils the breath of life, and man became a living soul. His life came from God, and he must spend an eternal existence in joy with Christ, or utter misery and wretchedness and anguish in hell.
Revelation 14:9-11 speaks of ungodly ones, the smoke of whose torment ascendeth up forever and ever, and they have no rest day or night. And these same words, “forever and ever,” or “to the age of ages,” are used to denote the eternal existence of the One upon the throne in Revelation 4:9. (See also 1 Tim. 1:17; Rev. 20:10).
Satan is sowing the world broadcast with his lie, that punishment is not eternal, and thousands are led away by this deceiver of the whole world. My reader, I warn you against it; it undermines the atonement, makes the Word of God a lie, and gives a license to man’s sins and wickedness. Far wiser is it to bow to and believe His Word; then, as a possessor of “eternal life,” the gift of God; go and tell out the glad tidings of salvation, pointing sinners to Christ and His precious blood, which alone can deliver from this awful doom.
“Fear not them which kill the body, but are not able to kill the soul: but rather fear Him which is able to destroy both soul and body in hell” (Matt. 10:28).
Following upon the judgment of the dead at the great white throne, in Revelation 21:1-9, we get the fullest revelation that God has given us concerning that wonderful condition of things that shall succeed dispensations of time, “the eternal state.”
You will remark that in Revelation 21:1 the prophet sees a new heaven and a new earth, but nothing is said about their being then created. Now in Isaiah 65:17 we do find the Lord saying, “Behold, I create a new heaven and a new earth,” etc.; but here it evidently has reference to the millennial period. This is clearly shown by the context, for the same word is used of Jerusalem, “Behold, I create Jerusalem a rejoicing, and her people a joy” (Isa. 65:18), and then follows an ample description of millennial blessing, to the close of the chapter. So entirely different will be the state of things at that day, that God speaks of it as created anew. But in the eternal state there will be a new heavens and a new earth; not a fresh creation, but probably the materials and elements of the old remodeled after dissolution.
Of the present globe we find it recorded in Genesis 1, that darkness was upon the face of the deep, and that God afterward caused the dry land to appear. And we know that even then the greater part of it was left submerged by the ocean to this day; but in the eternal condition there shall be no more sea. The wicked are like the troubled sea, when it cannot rest (Isa. 57:20); and restlessness and wickedness have characterized the whole history of man on the earth; but on the globe, where sea is not, all with be stable, fixed and eternal, and righteousness shall dwell.
The church of the living God is again brought before us in Revelation 21:2. Her thousand years of glory during the reign of Christ will have run out, but still she is seen in her distinctive glory to the age of ages. The holy city, New Jerusalem, is seen by the prophet coming down from God out of heaven, with all the freshness, beauty, and glory of the day of her espousals, “prepared as a bride adorned for her husband.”
A great voice out of heaven pronounces her glorious character and destiny in relation to this new and eternal scene. As the tabernacle or dwelling-place of God she is with men, showing a wonderful connection between heaven and earth in that scene of eternal glory and joy, and also that God has destined that men should live on that new globe. We have nothing told us as to who the men are, but as the wicked on the millennial earth are devoured by fire (Rev. 20: 9), and no mention is made of what becomes of the righteous, the probability is that the men here mentioned may he the millennial saints preserved for this wondrous portion by the power of God.
In Eden, when man was in innocence, the Lord God visited him; both His glory, and also the man Christ Jesus, the Son of God, have dwelt among men since; now the Holy Ghost dwells in men, the saints, and in the church, which is the temple of God; in the coming kingdom, the glory will again dwell in the midst of God’s people; but in the eternal state, the church will be the dwelling-place of God Himself, and be with men, and that forever and forever.
“And He will dwell with them, and they shall be His people, and God Himself shall be with them, and be their God” (Rev. 21:3-4). “And God shall wipe away all tears from their eyes; and there shall be no more death, neither sorrow, nor crying, neither shall there be any more pain: for the former things are passed away.” Every trace of sin, and all its terrible effects, will have gone forever. God Himself will comfort all, and bless with an eternal blessing. Righteousness will no longer only reign as in the kingdom of Christ, but dwell throughout the whole scene.
“And He that sat upon the throne said, Behold, I make (not create) all things new. And He said unto me (John) Write; for these words are true and faithful. And He said unto me, It is done. I am Alpha and Omega, the beginning and the end” (Rev. 21:5-6).
The septic and the infidel may doubt and deny, but God has pledged His immutable Word — “God, that cannot lie” (Titus 1:2). What He has promised, He is able also to perform; for with Him all things are possible; and “these words are true and faithful.” Wondrous pledge of His love to cheer and encourage the hearts of His own.
And then, before closing this short but marvelous revelation of a glorious eternity, we have three classes of persons brought before us in view thereof.
First, His thoughts go out to the poor thirsty sinner with the precious message, “I will give unto him that is athirst of the fountain of the water of life freely” (Rev. 21:6). Not now merely the living streams, but let the thirsty sinner be who he may, God is ready to satisfy that soul thirst from the fountain-head of blessing itself, and that freely. No money, no works, no religiousness of man, could purchase that priceless boon, but freely will God give of it to any and every thirsty one.
Secondly, He turns to His own loved ones, tried and tempted, harassed and perplexed, it may be, by the world, the flesh, and the devil, and cheers the drooping heart, the weary spirit, with the soul-stirring words of comfort, “He that overcometh shall inherit all things; and I will be his God, and he shall be My son” (Rev. 21:7). Faint one, think on this precious promise; shrink not from following your blessed Master. God would have you to be an overcomer; and as thy days, so shall thy strength be. Today may be your last here. “Our light affliction, which is but for a moment, worketh for us a far more exceeding and eternal weight of glory! (2 Cor. 4:17).
Lastly, in one sad catalog, He classes the ungodly, “But the fearful, and unbelieving, and abominable, and murderers, and whoremongers, and sorcerers, and idolaters, and all liars, shall have their part in the lake which burneth with fire and brimstone: which is the second death” (Rev. 21:8). The eternal judgment of the wicked is as sure as the eternal glory of the church; “the Scripture cannot be broken” (John 10:35).
“O the depth of the riches both of the wisdom and knowledge of God! how unsearchable are His judgments, and His ways past finding out! For who hath known the mind of the Lord? or who hath been His counselor? or who hath first given to Him, and it shall be recompensed unto Him again? For of Him, and through Him, and to Him, are all things: to whom be glory forever. Amen” (Rom. 11:33-36).
Rest of the saints above
Jerusalem of God,
Who in thy palaces of love,
Thy golden streets, have trod?
To me thy joy to tell,
Those courts secure from ill,
Where God Himself vouchsafes to dwell,
And every bosom fill.
Who shall to me that joy
Of saint-thronged courts declare,
Tell of that constant sweet employ
My spirit longs to share?
That rest secure from ill,
No cloud of grief e’er stains,
Unfailing praise each heart doth fill,
And love eternal reigns.
Concluded.

Correspondence: Church; John 14:2

Ques. 87. What is the church, and can the term be used for a building? A. B.
Ans. The church is the body of Christ, (Eph, 1:22-23). This was formed by the Holy Spirit at Pentecost. (Acts 2:1 Cor. 12:13). It includes only those who are saved.
The church as a profession or light bearer on earth, is also called The House of God (1 Tim. 3:15), and includes all baptized persons.
If we take away the word “church” and substitute the word “assembly” in every place where it is used in Scripture, we will get the right thought.
There is one exception, in Acts 19:37, which in the original is “idol temples.” It is never used otherwise of a building on earth.
Ques. 88. Does John 14:2 infer that there is a different mansion for each saved person? E. M.
Ans. This passage would not occupy us with thoughts of a grand mansion. It is supposed to be an allusion to the many chambers of the priests in the temple. (Ezek. 40). We are to see in it the love of Christ that has prepared a place with Himself for us. It has no thought of distance or living alone, but of nearness and sweet fellowship in His blest company. He is in the Father’s house. We are the sons (Rom. 8:29); children in the Father’s house; children of the same Father. This is why He says: “In My Father’s house”. We are in the company of the Father and the Son.
“ The Father’s house, the Father’s heart,
All that the Son is given
Made ours-the objects of His love-
And He, our joy in heaven.”
Courtesy of BibleTruthPublishers.com. Most likely this text has not been proofread. Any suggestions for spelling or punctuation corrections would be warmly received. Please email them to: BTPmail@bibletruthpublishers.com.